Download as pdf or txt
Download as pdf or txt
You are on page 1of 190

2012 Musculoskeletal Trauma Self-Assessment Examination by Dr.

Dhahirortho

1
1
2012 Musculoskeletal Trauma Self-Assessment Examination by Dr.Dhahirortho

2
2
2012 Musculoskeletal Trauma Self-Assessment Examination by Dr.Dhahirortho
Q. 1Figure 1 is the radiograph of a 62-year-old woman who fell and sustained a left
hip fracture. A radiographis shown in Figure 1. Which of the following preoperative
risk factors is associated with the highest postoperative mortality rate?

1. Fracture pattern
2. Chronic renal failure
3. Female gender
4. Coronary artery disease
5. Diabetes mellitus

DISCUSSION: In the study by Bhattacharyya and associates in 2002, they retrospectively


reviewed over 43,000 in-patient orthopaedic procedures to identify preoperative risk
factors associated with postoperative mortality. Their study identified five “critical” risk
factors placing patients at increased risk for death. These included chronic renal failure,
congestive heart failure, chronic obstructive pulmonary disease, hip fracture, and age of
older than 70 years. Their study also demonstrated a linear increase in mortality observed
with the increased number of risk factors. The risk factors of diabetes, gender, fracture
pattern, coronary artery disease, peripheral vascular disease, septic arthritis, and
rheumatoid arthritis did not achieve significance. Identification of patients with risk factors
for mortality is important for individualizing treatment plans, accurate prognosis, and
informed consent.

PREFE RESPONSE: 2

2.A 37-year-old man fell from 24 feet and sustained a subarachnoid hemorrhage and
closed femoral shaft fracture. What is most likely to lead to an adverse outcome?

1. Intraoperative hypotension
2. Temporizing external fixation
3. Elevated cerebral perfusion pressure
4. Immediate reamed intramedullary nailing
5. Skeletal traction with intramedullary nailing in 72 hours

DISCUSSION: In patients with femoral fractures and associated closed head injuries, there
have been conflicting studies regarding timing of fracture care and eventual neurologic
outcome. It is known that an episode of hypotension and elevated intracranial pressure will
lower the cerebral perfusion pressure, which is known to be detrimental to the neurologic
outcome. Intraoperative hypoxia may also worsen the neurologic outcome and increased
fluid administration may elevate the intracranial pressure. If early fracture fixation is
necessary, the intracranial pressure should be monitored and the cerebral perfusion
pressure maintained during the procedure. Immediate reamed intramedullary nailing is
appropriate if the patient is hemodynamically stable and the cerebral perfusion pressure is
maintained. If not, external fixation would be appropriate treatment. Temporary skeletal
traction may be appropriate if the intracranial pressure is labile and precludes the patient
from going to the operating room.

PREFERRED RESPONSE: 1

3
3
2012 Musculoskeletal Trauma Self-Assessment Examination by Dr.Dhahirortho
Question 3Figure 3a Figure 3b Figure 3c Figure 3a is the initial radiograph of a 19-
year-old man who sustained a closed clavicle fracture. Figures 3b and 3c show
postoperative radiographs. If the patient had been treated nonsurgically, which of
the following would most likely occur?

1. Normal shoulder strength and function


2. Local sensory deficits
3. Fracture union
4. Infection
5. Malunion

DISCUSSION: Recent studies comparing surgical treatment with nonsurgical management


in displaced clavicle fractures have revealed a decreased rate of malunion and nonunion
with surgery. In addition,significant malunions can lead to functional deficits at the
shoulder. Thus, with open reduction and internal fixation and anatomic or near-anatomic
reduction, there should be a higher likelihood of
normal shoulder strength and function. Infection and local sensory deficits would not be
expected with nonsurgical management, whereas surgical treatment has a small risk of
infection and a high likelihood of sensory deficits from iatrogenic damage to the
supraclavicular nerves.PREFERRED RESPONSE: 5

Question 4-What is the most common anatomic location of the lateral femoral
cutaneous nerve?

1. Deep to the psoas muscle


2. Medial to the femoral vein
3. Under the inguinal ligament
4. Adjacent to the femoral nerve
5. Deep to the iliopectineal fascia

DISCUSSION: The lateral femoral cutaneous nerve most commonly originates from the
lumbar plexus and runs on the surface of the iliacus muscle and enters the thigh by
passing under the inguinal ligament before piercing the fascia lata. Its path can be
variable. PREFERRED RESPONSE: 3

5.Figures 5a and 5b are the radiographs of a 24-year-old obese woman who


sustained a knee dislocation in a fall. Postreduction radiographs and an angiogram
are shown in Figures 5c through 5e. Examination reveals a cold foot with no pedal
pulses. After vascular repair and four-compartment fasciotomy is performed by a
vascular surgeon, and while the patient is still in the operating room, you are
contacted and asked to evaluate the patient. The knee is grossly unstable. What is
the most appropriate initial
orthopaedic management?

4
4
2012 Musculoskeletal Trauma Self-Assessment Examination by Dr.Dhahirortho

1. Spanning external fixation of the knee


2. Open lateral collateral ligament repair and delayed anterior cruciate
ligament/posterior cruciate ligament/medial collateral ligament reconstruction
3. Application of a cylinder cast
4. Arthroscopic anterior cruciate ligament/posterior cruciate ligament reconstruction and
open lateral cruciate ligament/medial cruciate ligament repair
5. Diagnostic knee arthroscopy

DISCUSSION: Knee dislocations are known to have a high risk for vascular injury.
Although the specific treatment of various combinations of ligamentous injuries is
controversial, the need for emergent revascularization is not. In this particular patient, after
vascular repair, the most important initial concern is protection of the vascular repair. A
spanning external fixator, especially in this patient with gross instability, will allow for easier
assessment of vascular status, evaluation of fasciotomy wounds,
and temporary stability of the knee. A cylinder cast can stabilize the knee but will not allow
wound assessment or room for inevitable post-injury/postoperative swelling. Diagnostic
knee arthroscopy is not necessary, and ligamentous repair/reconstruction should be
delayed until the vascular repair is stable. PREFERRED RESPONSE: 1

Question 6 Figures 6a and 6b are the


radiographs of a thin 23-year-old man who
sustained a closed injury to his left arm in a
fall. He has no other injuries and his
neurologic examination is normal. What is
the most appropriate treatment?

1. Intramedullary nailing
2. Hanging arm cast for 6 weeks
3. Shoulder immobilizer for 4 to 6 weeks
4. Open reduction and internal fixation
5. Coaptation splinting with conversion to a
fracture brace

DISCUSSION: The patient is a thin man with


an isolated left humerus fracture. The fracture
has bony apposition and should be amenable
to closed treatment; therefore the most

5
5
2012 Musculoskeletal Trauma Self-Assessment Examination by Dr.Dhahirortho
appropriate treatment is coaptation splinting with conversion to a fracture brace. A hanging
arm cast is not recommended for a transverse fracture because of the propensity to
distract the fragments, especially if left in place for a long period of time. A shoulder
immobilizer is not an appropriate treatment for a humeral shaft fracture. A transverse
fracture line is sometimes considered a relative indication for surgical treatment if the
fragments are distracted, but in this patient, immediate surgical fixation is not warranted in
the absence of other indications for surgical treatment. PREFERRED RESPONSE: 5

Question 7Figure 7Figure 7 is the pelvic radiograph of a 33-year-old man involved in


a high-speed automobile crash. Examination reveals a blood pressure of 90/50 mm
Hg and a pulse rate of 120/min. Radiographs of the chest and lateral cervical spine
are normal. A CT scan of the abdomen does not reveal any intraabdominal bleeding.
What is the most appropriate management for the pelvic fracture?

1. Angiography
2. Application of a pelvic binder
3. Anterior external fixation
4. Anterior external fixation with pelvic packing
5. Open reduction and internal fixation of the pubic
symphysis

DISCUSSION: Since the patient has not had any


mechanical stabilization to the pelvic ring, the first
step in management should be application of a sheet
or binder along with resuscitation. Pelvic binders have
been shown to be effective in decreasing transfusion
needs and are quick and simple to apply. Emergent
external fixation, pelvic packing, or angiography is not indicated unless the patient is
unresponsive to these initial measures. The order in which these measures are used is
controversial and somewhat institution dependent. Repair of the pubic symphysis is
indicated as part of the definitive treatment but should not be done prior to resuscitation
with pelvic binder placement. PREFERRED RESPONSE: 2

Question 8Figure 8What is the most common cause of death in a patient with the
injury shown in Figure 8?

1. Visceral injury
2. Exsanguination
3. Closed head injury
4. Under-resuscitation
5. Disseminated intravascular coagulation

DISCUSSION: The most common identifiable cause


of death in patients with lateral compression
fractures is closed head injury. In contrast, the
identifiable cause of death in patients with
anteroposterior compression injuries is combined
pelvic and visceral injury. Lateral compression injury
results from a lateral impact to the pelvis that rotates the pelvis on the side of the impact
toward the midline. The sacrotuberous and sacrospinous ligaments, as well as the internal
iliac vessels, are shortened and are not subjected to tensile forces. Disruption of large
named vessels (eg, internal iliac artery, superior gluteal artery) is relatively uncommon with
lateral compression injuries. PRE RESP: 3
6
6
2012 Musculoskeletal Trauma Self-Assessment Examination by Dr.Dhahirortho
Question 9A 28-year-old woman with a history of systemic lupus erythematosus
was involved in a motor vehicle crash. She sustained a closed left tibia fracture and
underwent surgery. During surgery, the tourniquet was left inflated while the
surgeon reamed the tibial canal to place the largest diameter nail that could be fit. At
6 weeks follow-up, there is evidence of massive bone necrosis. What event most
likely led to the necrosis?

1. History of steroid use


2. History of systemic lupus erythematosus
3. Over reaming of the tibial canal
4. Reaming of the tibia with the tourniquet inflated
5. Reaming of the tibia with the knee in hyperflexion

DISCUSSION: Karunaker and associates showed in a canine model that there is no


significant difference in the heat generated during reaming with and without a tourniquet.
The factor that made the most difference was related to the size of the reamer used
compared with the diameter of the isthmus.Giannoudis and associates performed a
prospective randomized trial on 34 patients that evaluated the
same thing as the first study with the same methodology, and the conclusions were again
the same. The factor that generated the most heat was using large reamers (11 mm to 12
mm) in a patient with a small isthmus (8 mm to 9 mm). Systemic lupus erythematosus,
steroid use, and knee flexion during reaming have not been shown to be associated with
diaphyseal necrosis after reamed tibial nailing. PREF RESP: 3

Question 10-Figures 10a and 10b are the radiographs of a 33-year-old man who was
involved in a high-speed motorcycle crash. He sustained an isolated injury to the
right lower extremity. On the day of injury, he was treated with open reduction and
internal fixation of the femoral neck and retrograde nailing of the femur.
Radiographs are shown in Figures 10c through 10f. Alternative treatment with a
cephalomedullary device alone would be more likely to lead to which of the
following outcomes?

7
7
2012 Musculoskeletal Trauma Self-Assessment Examination by Dr.Dhahirortho
1. More postoperative pain
2. More rapid healing of the femoral neck fracture
3. Higher union rate of the femoral neck fracture
4. Higher union rate of the femoral shaft fracture
5. Higher rate of malreduction of one of the fractures

DISCUSSION: The patient has ipsilateral fractures of the femoral neck and femoral shaft.
This is not an uncommon scenario, often found in high-energy injuries in younger patients.
There is some controversy as to the best method of fixation with some authors
recommending separate implants for the two fractures,
and some recommending a single antegrade cephalomedullary nail for treatment of both
fractures. The use of a single implant does not increase healing time of the femoral neck
fracture or limit postoperative pain. However, the use of a single implant is associated with
higher malreduction rates of either the shaft or neck component which could lead to
increased rates of nonunion or malunion.-

PREFERRED RESPONSE: 5

Question 11.Figures 11a and 11b show the radiographs of the open fracture of a 46-
year-old man who injured his elbow on his nondominant arm in a motorcycle crash.
On the day of injury, he underwent irrigation and débridement of the fracture. He
was also treated with antibiotics. Which of the following definitive treatment
methods will most likely lead to the best functional outcome?

1. Cast immobilization
2. Intramedullary screw fixation
3. Open reduction and plate fixation
4. Open reduction and internal fixation with tension band wiring
5. Fragment excision and triceps advancement

DISCUSSION: The patient has an open comminuted transolecranon


fracture-dislocation. This occurs when the distal humerus is driven through the proximal
ulna, and it is often associated with comminution of the olecranon and proximal ulna. The
distal fragment translates anteriorly. Results of surgical treatment of transolecranon
fracture-dislocations are best and most reliable when the fracture is reduced anatomically
and plate fixation is used. Nonsurgical management is not indicated in this injury
pattern.Excision of the comminuted fragments and advancement of the triceps will likely
lead to persistent anterior instability of the elbow. Tension band wiring relies on cortical
contact which will not be possible in this fracture. Intramedullary screw fixation is also not
possible because of the significant comminution.

PREFERRED RESPONSE: 3

8
8
2012 Musculoskeletal Trauma Self-Assessment Examination by Dr.Dhahirortho
Question 12.Figures 12a through 12c show the radiographs of the closed fracture of
a 24-year-old man who sustained an isolated injury to his left foot in a motorcycle
crash. He was splinted and, on the following day, he nunderwent open reduction
and internal fixation. Postoperative radiographs are shown in Figures 12d through
12f. What is the most likely complication of this injury?

1. Malunion
2. Nonunion
3. Osteomyelitis
4. Osteonecrosis
5. Posttraumatic arthritis

DISCUSSION: The patient has a talar neck fracture that is associated with several well-
known complications. Posttraumatic arthritis is the most common complication and
osteonecrosis is slightly less common. These two complications are often out of the control
of the orthopaedic surgeon and do not seem to be influenced by the timing of fixation.
Malunion and nonunion are relatively uncommon when an anatomic reduction and stable
fixation can be obtained. Open reduction can help ensure the best possible reduction, and
plate fixation may be a more stable method of fixation, especially useful in preventing
collapse through areas of comminution. Osteomyelitis is rare in closed fractures.

PREFERRED RESPONSE: 5

9
9
2012 Musculoskeletal Trauma Self-Assessment Examination by Dr.Dhahirortho
Question 13When comparing the results of open reduction and internal fixation
(ORIF) versus antegrade intramedullary nailing (IMN) fixation of the humeral
diaphysis in prospective randomized trials, which of the following statements is
most accurate?
1. Union rates are higher with IMN.
2. Reoperation rates are higher with IMN.
3. Shoulder outcomes are similar for ORIF and IMN.
4. Infection rates are higher with ORIF.
5. Radial nerve complications are higher with ORIF.

DISCUSSION: There are relatively few comparative studies of the treatment of diaphyseal
fractures of the humerus in the literature. In a meta-analysis of three prospective
randomized trials comparing ORIF with IMN, open reduction and internal fixation showed a
90% risk reduction of shoulder impingement symptoms and a 75% risk reduction of
reoperation. There is no difference in infection rate, nonunion rate,and radial nerve issues.
PREFERRED RESPONSE: 2

Question 14Which inflammatory marker is most closely tied to a systemic


inflammatory response following orthopaedic injury and treatment?

1. Interleukin 1 (IL-1)
2. Interleukin 6 (IL-6)
3. Interleukin 10 (IL-10)
4. Tumor necrosis factor, alpha
5. D-dimer

DISCUSSION: Significant basic science research has been done on identifying


inflammatory markers associated with systemic inflammatory response following trauma
and musculoskeletal injury. Although not yet clinically applicable, IL-6 has been identified
as a marker that correlates well with musculoskeletal injury (ie, femur fracture) and
treatment of these injuries (ie, intramedullary nailing). IL-1 and IL-10 do not correlate with
treatment of musculoskeletal injury. Tumor necrosis factor, alpha and D-dimer,although
often elevated following trauma, do not correlate with musculoskeletal treatment.
PREFERRED RESPONSE: 2

Question 15Figure 15a Figure 15bFigures


15a and 15b are the radiographs of a 28-
year-old man who fell from a height and
sustained an isolated closed diaphyseal
femur fracture that was treated with reamed
antegrade femoral nailing 8 months ago. He
now reports persistent pain during
ambulation. He smokes one pack of
cigarettes per day but is otherwise healthy.
He denies any infectious history or
symptoms, and laboratory studies show a
normal WBC count, erythrocyte
sedimentation rate, and C-reactive protein.
What is the most appropriate treatment?

10
10
2012 Musculoskeletal Trauma Self-Assessment Examination by Dr.Dhahirortho
1. Bone graft in situ
2. External bone stimulation
3. Reamed antegrade exchange nailing
4. Reamed retrograde exchange nailing
5. Open biopsy to assess for infection followed by delayed surgical stabilization

DISCUSSION: The patient has an uninfected symptomatic nonunion of the femur after
reamed antegrade nailing with an appropriately sized implant. The fracture is well aligned
and has some callus response indicating reasonable vascularity. Auto-dynamization has
occurred via fatigue failure of the distal interlocking screws but the patient remains
symptomatic and the fracture line is evident, consistent with nonunion. Reamed exchange
nailing is preferred because it allows for improved mechanics via a larger diameter nail and
repeat interlock and improved biologics via reaming which is felt to elicit an inflammatory
reaction and generate bone graft in situ. Unfortunately, the results of exchange nailing are
not as good in patients who smoke and smoking cessation should be counseled and
encouraged. The data on external bone stimulation on unhealed fractures of the femur with
an intramedullary nail present are lacking. In light of a benign clinical examination and
history and normal blood work parameters with reference to infection, open biopsy of the
nonunion prior to definitive surgical treatment is unwarranted.The patient has mechanical
instability and bone grafting in situ will not address this issue in terms of promoting
progression to union or allowing for improved function and less pain. PREF RESP: 3

Question 16Figure 16b Figure 16aFigures 16a and 16b show the initial radiograph
and CT scan after the application of a pelvic binder in a24-year-old woman who
sustained a pelvic ring injury in a motor vehicle accident. What does the fractureof
the sacrum best indicate?

1. Pelvic instability
2. Indication to repair the sacrum
3. Higher likelihood of bowel injury
4. Lower likelihood of neurologic injury
5. Lower likelihood of internal bleeding

DISCUSSION: The radiograph and CT scan reveal a fracture of the lateral sacrum due to
avulsion of the sacrospinous and sacrotuberous ligaments. These ligaments are disrupted
in an unstable anteroposterior compression-type pelvic ring injury. Higher grade injuries
have a higher risk of vascular and/or neurologic injuries. PREF RES: 1

11
11
2012 Musculoskeletal Trauma Self-Assessment Examination by Dr.Dhahirortho
Question 17When attempting to treat a proximal tibial metadiaphyseal fracture with
an intramedullary nail, what is the most common angular malalignment?
1. Varus alone
2. Valgus alone
3. Varus and procurvatum
4. Valgus and procurvatum
5. Valgus and recurvatum

DISCUSSION: Fractures of the proximal metadiaphysis of the tibia can be treated


successfully with intramedullary nails but historic rates of malalignment are up to 84%. The
typical deformity is valgus and procurvatum due to the metaphyseal bony anatomy,
eccentric start point, deforming force of the patellar tendon, and implant factors such as
the Herzog curve of the nail. An ideal starting point is mandatory and should be at the
medial border of the lateral tibial eminence on a true AP view and very proximal and
anterior on a true lateral view with appropriate coronal and sagittal trajectory of the entry
reamer. A reduction should be obtained and maintained during reaming, implant insertion,
and interlocking. This can be facilitated via a variety of techniques including intraoperative
external fixation, percutaneous reduction clamps or joysticks, semi-extended positioning,
blocking screws, and ancillary plate fixation. PREFERRED RESPONSE: 4

Question 18Figures 18a through 18c show injuries sustained by a 22-year-old


woman after falling 45 feet while mountain climbing. After being airlifted to the
nearest trauma center, her arterial blood gas was 7.21, pO2 84, pCO2 48, and base
arterial blood gas was 7.21, pO2 84, pCO2 48, and delta base -11 mmol/L. Her Hg is
8.7 and her resuscitation is ongoing. Based on this data, what would be the best
management of her orthopaedic injuries?

1. External fixation of the pelvis, external fixation of the distal femur, and splinting of the
humerus
2. External fixation of the pelvis, external fixation of the distal femur, and intramedullary
nailing of the humerus
3. External fixation of the pelvis, open reduction and internal fixation of the distal femur,
and splinting of the humerus
4. Open reduction and internal fixation of the pelvis, open reduction and internal fixation of
the distal femur, and intramedullary nailing of the humerus
5. Open reduction and internal fixation of the pelvis, open reduction and internal fixation of
the distal femur, and open reduction and internal fixation of the humerus

12
12
2012 Musculoskeletal Trauma Self-Assessment Examination by Dr.Dhahirortho
DISCUSSION: The patient is under-resuscitated and would benefit from minimally invasive
stabilization of the pelvic ring and long bone fractures in a “damage-control” approach.
External fixation of the pelvis and femur can be performed quickly and with minimal blood
loss which should limit the “second hit” associated with more prolonged, invasive surgery.
Upper extremity fractures are best managed acutely with splints in this clinical setting.
Definitive fracture fixation should be delayed until the patient is adequately resuscitated.
PREFERRED RESPONSE: 1

Question 19Figure 19bFigures 19a and 19b are the radiographs of a 32-year-old
woman who has sustained multiple injuries after being struck by a motor vehicle
while riding a bicycle. She is intubated on arrival and remains tachycardic and
hypotensive. Pulses are hard to palpate but the right hand is somewhat cooler to
touch than the left hand. She has a large open wound over the upper arm. What is
the most important predictor of outcome with these injuries?

1. Open wound size


2. Open wound contamination
3. Time to débridement
4. Adequacy of débridement
5. Neurovascular status

DISCUSSION: The patient has lateral translation of the shoulder girdle on the chest
radiograph as measured from the spinous process to the medial border of the scapula with
resultant acromioclavicular widening consistent with the diagnosis of scapulothoracic
dissociation. She also has an ipsilateral open fracture of the proximal humeral diaphysis.
This represents an extremely high-energy injury to the upper extremity with a dismal
prognosis. The overall mortality in the presence of scapulothoracic dissociation is 10%.
Over 90% of patients will have neurologic injury which is often a complete and permanent
brachial plexopathy, and a significant percentage will have associated limb-threatening
vascular injuries. In the presence of a complete vascular and neurologic injury, amputation
must be considered. While important,wound size and contamination, and time or adequacy
of débridement will not likely drive the clinical outcomes in the presence of such a
significant concomitant injury. PREFERRED RESPONSE: 5

13
13
2012 Musculoskeletal Trauma Self-Assessment Examination by Dr.Dhahirortho
Question 20Figures 20a and 20b are the radiographs of a 19-year-old woman who
was involved in a motor vehicle accident. What mechanism of injury is most
consistent with the injury?

1. Vertical shear
2. External rotation
3. Sagittal translation
4. Lateral compression
5. Anterior posterior compression

DISCUSSION: The radiographs show a lateral compression pelvic ring injury with a
displaced superior ramus fracture, or tilt fracture. Tilt fractures are most commonly caused
by a lateral compression mechanism. These injuries are often seen in female patients and
careful examination, including vaginal examination, is required to rule out open fractures.
Lateral compression results in internal rotation,not external rotation, of the pelvic ring. Tilt
fractures are not commonly seen with anterior-posterior compression injuries or vertical
shear injuries. Sagittal translation is not a term used to describe pelvic ring injuries.
PREFERRED RESPONSE: 4

Question 21Figure 21Figure 21 is the radiograph of a 45-year-old woman who was


severely injured in a motorcycle crash.Her injuries include a traumatic subarachnoid
hemorrhage, bilateral pneumothoraces with pulmonary contusions and flail chest,
fracture-dislocation of the left hip, and open fractures of the right distal femur and
proximal tibia. Antibiotics and tetanus are administered in the emergency
department. The patient is intubated and bilateral chest tubes are placed. A closed
reduction is performed on the left hip. After appropriate
resuscitation, what is the most appropriate initial
management of the right knee injury?

1. Skeletal traction
2. Irrigation and débridement of the open fractures
3. Irrigation and débridement and spanning external fixation of
the knee
4. Open reduction and internal fixation of the proximal tibia and
distal femur
5. Percutaneous screw fixation of the articular fragments with
retrograde femoral nailing and antegrade tibial nailing

14
14
2012 Musculoskeletal Trauma Self-Assessment Examination by Dr.Dhahirortho
DISCUSSION: Although the radiographic evaluation is incomplete, the single lateral view
shows a comminuted fracture of the distal femur with suspicion of intra-articular injury and
an ipsilateral proximal tibia fracture. This is an open fracture that requires antibiotics,
débridement, and skeletal stabilization.
The fractures are complicated and the patient is polytraumatized; therefore, rapid but
complete surgical débridement and simple stabilization of the knee with a spanning
external fixator would be the most appropriate management. Definitive surgical
stabilization will likely be complicated and is less desirable during the early post-injury
period. PREFERRED RESPONSE: 3

Question 22Figure 22Which of the following strategies is helpful to avoid the


complication seen in Figure 22?

1. Fibular plating
2. Blocking screws
3. Medial starting point
4. Nailing in the flexed position
5. Cross Kirschner wire fixation prior to nail insertion

DISCUSSION: This is the classic deformity encountered during


intramedullary nailing of a proximal one third tibia fracture: apex
anterior angulation and anterior translation of the proximal
segment.Blocking screws, nailing in the semi-extended position
and a lateral starting point all may help avoid the malalignment
seen with proximal tibial metaphyseal fractures. Fibular plating
may help with distal tibial metaphyseal fracture alignment. Cross
Kirschner wire stabilization is not used in adult fracture
patterns.A temporary unicortical plate, external fixator, or
distractor may be used instead to hold provisional reduction
while the nail is inserted.

PREFERRED RESPONSE: 2

Question 23Figure 23Figure 23 is the radiograph of a 22-year-old woman who was


involved in a motor vehicle collision. She reports isolated pain in her left shoulder.
She is hemodynamically stable, respiring comfortably, and neurovascularly intact.
Based on these findings, which of the following statements regarding treatment is
most appropriate?

1. Union rates are in excess of 95% if treated


nonsurgically.
2. A figure-of-8 brace is superior to a sling for nonsurgical
management.
3. Open reduction and internal fixation increases the
likelihood of a nonunion.
4. Open reduction and internal fixation results in improved
functional outcomes.
5. Open reduction and internal fixation and nonsurgical
management have equivalent outcomes at 1 year.

15
15
2012 Musculoskeletal Trauma Self-Assessment Examination by Dr.Dhahirortho
DISCUSSION: The patient has sustained an isolated, closed, transverse fracture of the
middle third of the clavicle with greater than 100% displacement and greater than 2 cm of
shortening. Whereas the traditional treatment of clavicle fractures has been
overwhelmingly conservative, recent reports suggest that surgical fixation should be
considered for certain injury patterns. The union rates of displaced clavicle fractures are
more recently noted to be approximately 85%, which is lower than the traditional literature.
In a prospective randomized trial of clavicle fractures with greater than 100%
displacement, union rates were higher and functional outcomes were better at all time
points up to 1 year after injury in the surgical group when compared with nonsurgical
management. PREFERRED RESPONSE: 4

Question 24A 19-year-old man underwent intramedullary nailing of a closed tibia


fracture 1 year ago and has never been pain free. While playing football, he was
tackled and sustained the injury shown in Figure 24a.What is the best treatment
option based on the radiographs seen in Figures 24b and 24c?

1. Circular fixator
2. Exchange nailing
3. Iliac crest bone graft
4. Straightening of the leg and casting
5. Removal of the nail and functional bracing

DISCUSSION: This is a young, healthy man with a tibial nonunion and a failed implant. He
requires treatment for the nonunion. In the absence of bone loss and/or infection, the injury
is best treated with removal of the bent nail and a reamed exchange nailing. Casting alone
or functional bracing is not the best option in a patient with an atrophic nonunion. Use of
circular fixators is an option; however, in a young, healthy patient with a fracture that has
bony contact, the first line of treatment is exchange nailing. PREFERRED RESPONSE: 2

Question 25Figure 25cFigures 25a through 25c show the radiographs, including a
stress radiograph, of a 58-year-old woman who twisted her ankle on a step. She has
no history of diabetes or vascular disease. Examination reveals a closed injury with
moderate swelling about the ankle. Her neurologic examination is normal. She hasa
strong dorsalis pedis pulse and tenderness over the lateral malleolus and the
medial side of her ankle. What is the most appropriate management?
1. MRI scan of the ankle
2. Non-weight-bearing cast for 6 weeks
3. Removable walking boot and progressive weight bearing
4. Open reduction and internal fixation of the fibula
5. Open reduction and internal fixation of the fibula with medial ligament repair
16
16
2012 Musculoskeletal Trauma Self-Assessment Examination by Dr.Dhahirortho

DISCUSSION: The patient has a lateral malleolus fracture with an ankle mortise that is
stable to a stress examination; therefore, surgical treatment is not indicated. In a stable
lateral malleolus fracture, strict non-weight-bearing is not necessary, and a removable
walking boot or walking cast can be used along with progressive weight bearing. The
presence of tenderness or swelling medially at the ankle has been shown to be a poor
indicator of medial-sided injury. The clinical utility of MRI scans in ankle fractures is
controversial. Studies have used MRI scans to evaluate the competence of the deltoid
ligament and have shown that the ligament may remain intact even with an increased
medial clear space on a stress examination. In the patient, the stress examination does not
show talar subluxation so the deltoid ligament is not incompetent. PREFERR RESPO: 3

Question 26Figure 26Figure 26 is the radiograph of a 33-year-old woman who was


involved in a high speed motor vehicle crash. Her initial blood pressure is 80/50 mm
Hg and she has a pulse rate of 120 bpm. After hemodynamic stabilization and
temporizing measures have been performed, the patient is cleared for surgery. What
is the most appropriate method of definitive fixation?
1. External fixation
2. Open reduction and internal fixation of the pubic symphysis with a two-hole plate
3. Open reduction and internal fixation of the pubic symphysis with a two-hole plate and
posterior triangular osteosynthesis
4. Open reduction and internal fixation of the pubic symphysis with a multi-hole plate
5. Open reduction and internal fixation of the pubic symphysis with a multi-hole plate and
posterior plate osteosynthesis

DISCUSSION: The patient has sustained an


anterior posterior compression (APC) grade II
pelvic ring injury. Initial management should
consist of pelvic volume reduction with pelvic
binding or sheeting. Once the patient is
hemodynamically stable, the decision for
definitive management should be made. In a
retrospective review of more than 200 patients,
Sagi and Papp investigated plate osteosynthesis
of the pubic symphysis. They found significantly
fewer malunions in the multi-hole plate group and
a trend toward fewer surgeries in the same
group. Typically external fixation should be

17
17
2012 Musculoskeletal Trauma Self-Assessment Examination by Dr.Dhahirortho
reserved for temporary fixation and not a definitive management in stable patients.
Posterior fixation is reserved for injuries with disruption of the posterior ligamentous
constraints, typically APC grade III injuries. Triangular osteosynthesis is a strategy for
fixation of unstable vertical shear fractures that require fixation of the pelvis to the lumbar
spine. PREFERRED RESPONSE: 4

Question 27Figure 27Figure 27 is the radiograph of a 75-year-old woman with a 1-


year history of left arm pain following a fall.What is the most appropriate
management?

1. Thermal imaging
2. Metabolic/endocrine work-up
3. MRI of the arm
4. PET scan
5. Infection work-up

DISCUSSION: The patient should be evaluated for any correctable


metabolic or endocrine abnormality
that may exist prior to any surgical intervention. Infection in the
absence of previous surgery is very unlikely. There are no
radiographic findings suggestive of a malignancy; therefore, MRI
and PET scans are not indicated. Thermal imaging is not in use in
orthopaedics. PREF RESPON: 2

Question 28Of the following variables, which has the strongest influence on external
fixator stiffness?
1. Pin diameter
2. Pin spread
3. Bone quality
4. Stacking a second fixator bar
5. Distance from bone to fixator bar

DISCUSSION: Whereas all of the factors will have an impact on frame rigidity and stability,
the single biggest factor is the pin diameter because it has an exponential effect.
PREFERRED RESPONSE: 1

Question 29Figure 29Figure 29 is the radiograph of a 30-year-old man who


sustained an isolated tibial shaft fracture. What is the most common deformity with
nonsurgical management?

1. Varus
2. Malrotation
3. Valgus
5. Valgus and recurvatum
4. Valgus and procurvatum

DISCUSSION: Studies have shown that approximately 25% of


diaphyseal fractures of the tibia with intact fibulae will go onto varus
malunion if treated nonsurgically. Limb-length discrepancies are also
common.Here the fibula acts as a strut, preventing valgus collapse but
predisposing to varus collapse. Valgus and procurvatum is the typical
deformity in proximal tibial fractures. PREFERRED RESPONSE: 1
18
18
2012 Musculoskeletal Trauma Self-Assessment Examination by Dr.Dhahirortho
Question 30Figures 30a and 30b are the radiographs of a 61-year-old man with
diabetes who fell from a ladder and sustained an isolated closed fracture. After
realignment and splint application, what is the most appropriate next step in
management?

1. CT scan
2. Hybrid external fixation
3. Ankle-spanning external fixation
4. Open reduction and internal fixation within 6 to 8 hours
5. Open reduction and internal fixation within 2 to 3 days

DISCUSSION: The patient has sustained a high-energy severely comminuted AO/OTA C2


fracture of the distal tibia. This injury is notably fraught with soft-tissue complications that
can lead to disastrous clinical results. In general, a staged protocol is now preferred in an
effort to avoid these complications and has shown substantial decreases in infection rates
and wound healing problems. A CT scan is certainly appropriate for preoperative planning
but should be obtained after frame application because the indirect reduction that is
achieved improves one’s ability to understand the fracture characteristics and morphology.
Hybrid external fixation has fallen out of favor because of its limited biomechanic rigidity
and clinical results. Open reduction and internal fixation in the acute phase (6 to 8 hours)
or sub-acute phase (2 to 3 days) is difficult.PREFERRED RESPONSE: 3

Question 31A starting point entry portal that is too lateral on a trochanteric femoral
nail will result in what deforming force?

1. Varus
2. Valgus
3. Flexion
4. Extension
5. Excessive hoop stress

DISCUSSION: The trochanteric entry portal for femoral nail insertion is increasingly being
used by orthopaedic surgeons both for cephalomedullary implants and standard femoral
nailing. In contradistinction to the piriformis fossa, the tip of the trochanter is not co-linear
to the diaphyseal isthmus and an errant start can lead to the introduction of malalignment
and/or iatrogenic comminution at the fracture site. The desired starting point should be at
the tip or slightly medial to the tip of the greater trochanter to avoid varus malalignment
and blow out of the lateral wall. PREFERRED RES: 1
19
19
2012 Musculoskeletal Trauma Self-Assessment Examination by Dr.Dhahirortho
Question 32A 26-year-old man is involved in a high-speed motorcycle accident. He
sustains a grade IIIB open tibia fracture. Examination reveals a large soft-tissue
defect and an insensate foot. What is the expected outcome in this scenario?

1. Equal functional outcome when limb salvage is compared with amputation


2. Worse functional outcome with limb salvage than with primary amputation
3. Better functional outcome when amputation is compared with limb salvage
4. Amputation within 6 months of injury
5. Permanent loss of plantar sensation

DISCUSSION: The Lower Extremity Assessment Project data have shown that absent
plantar sensation is not an indication for primary amputation. When looking at a
comparison between an insensate salvage group and a sensate salvage group at 2 years
follow-up, both groups had an equal proportion (55%) of normal plantar sensation and
functionally both groups were equivalent. Absent plantar sensation at initial evaluation is
not prognostic for long-term plantar sensory status or functional outcome. PREF RESP: 1

Question 33Which of the following clinical scenarios represents the strongest


indication for locked plating technique in a 70-year-old woman?

1. Segmentally comminuted ulnar fracture


2. Simple diaphyseal fracture of the humerus
3. Transverse midshaft displaced clavicle fracture
4. Periprosthetic femur fracture distal to a well-fixed total hip arthroplasty
5. Schatzker 2 fracture of the tibia with severe joint depression and comminution

Locking screw fixation is a relatively new option in the armamentarium of orthopaedic


surgeons treating fractures. The understanding of the biomechanics, implications to
healing, and optimal indications and surgical techniques is still in evolution. A
periprosthetic proximal femur fracture with a stable prosthesis is best treated with open
reduction and internal fixation with locking proximal fixation with or without cerclage cables.
Diaphyseal fractures treated with compression plating or bridge plating can be treated well
with conventional implants unless osteoporosis is severe. An AO/OTA B-type partial
articular fracture is also better suited to standard buttress plating with periarticular rafting
lag screws.Locking fixation is not always required for a transverse displaced midshaft
clavicle fracture. PREF RESPONSE: 4

Question 34A 65-year-old woman with rheumatoid arthritis is involved in a motor


vehicle accident. Her injuries include a right displaced femoral neck fracture, a left
open tibial pilon fracture, a left open tibial plateau fracture, multiple rib fractures,
and bilateral pulmonary contusions. Her vitals signs on admission are a heart rate
of 115 bpm and a systolic blood pressure of 90 mm Hg. Laboratory studies show a
hemoglobin of 10.0 g/dL and a delta base of -6.0 mmol/L. What finding in this patient
is most significantly associated with increased mortality?

1. Heart rate
2. Base deficit
3. Hemoglobin
4. Urine output
5. Systolic blood pressure

The severity of injuries and the lack of physiologic reserve in this and other elderly patients
often result in mortality. Base deficit has shown to be a reliable predictor of mortality even
20
20
2012 Musculoskeletal Trauma Self-Assessment Examination by Dr.Dhahirortho
in normotensive elderly blunt trauma patients. Although tachycardia, low systolic blood
pressure, and low hemoglobin may all contribute to these patients’ mortality, base deficit
may be used as a predictor of mortality and a measure of resuscitation. PREFE RES: 2

Question 35-A fracture of what portion of the coronoid is most often associated with
a terrible triad injury?
1. Tip
2. Rim
3. Base
4. Anterolateral facet
5. Anteromedial facet

DISCUSSION: The most common pattern of cornoid fracture with a terrible triad injury is a
transverse fracture of 2 mm to 3 mm of the tip. The mechanism of injury of a terrible triad
injury is typically valgus and supination. These forces force the radial head against and
then under the capitellum, resulting in a fracture of the radial head. The coronoid is then
driven under the trochlea and sheared off as the valgus force continues. The lateral
collateral ligament typically tears next.

PREFE RESPONSE: 1

Question 36A 45-year-old man sustained the injury shown in Figures 36a and 36b.
The involved side is his dominantside. What is the most appropriate management?

1. Closed reduction
2. Arthroscopic labral repair
3. MRI to evaluate the rotator cuff
4. Stress radiographs to evaluate instability
5. Early motion in a structured physical therapy program

DISCUSSION: This minimally displaced (one-part) proximal humerus fracture is best


treated with nonsurgical management. Early motion and physical therapy should be
instituted to optimize functional results. No reduction is required. There is no indication for
an acute MRI scan. If symptoms exist after healing, one may be obtained. Labral injuries
are not typically associated with this type of injury.Instability is not associated with a one-
part fracture and stress radiographs are not described.

PREFERRED RESPONSE: 5
21
21
2012 Musculoskeletal Trauma Self-Assessment Examination by Dr.Dhahirortho
Question 37Which set of patient characteristics has the highest risk of developing
osteonecrosis after an intracapsular femoral neck fracture?

1. 45-year-old woman with a displaced fracture


2. 55-year-old man with a nondisplaced fracture
3. 70-year-old woman with a nondisplaced fracture
4. 70-year-old man with a displaced fracture
5. 85-year-old woman with a displaced fracture

DISCUSSION: Loizou and associates prospectively studied 1,023 patients who sustained
an intracapsular hip fracture that was treated with internal fixation using contemporary
methods. The overall incidence of osteonecrosis was 6.6%. Osteonecrosis was less
common for undisplaced (4.0%) than for displaced fractures (9.5%) and in men (4.9%)
than women (11.4%) who had a displaced fracture. The incidence of osteonecrosis for
those patients younger than 60 years and who sustained a displaced fracture was
20.6%,compared with 12.5% for those aged 60 to 80 years and 2.5% for those older than
age 80 years. Barnes and associates reported that late segmental collapse was more
common in displaced fractures in women younger than age 75 years than in those older
than age 75 years. PREFERRED RESPONSE: 1

Question 38-When compared with reamed intramedullary nailing for an unstable


diaphyseal tibia fracture, undreamed nailing is associated with which of the
following?

1. Longer surgical times


2. Higher infection rates
3. Lower functional outcome scores
4. Similar union rates in open fractures
5. Higher incidence of pulmonary complications

DISCUSSION: The Investigators Randomized Trial of Reamed versus Non-Reamed


Intramedullary Nailing of Tibial Shaft Fractures (SPRINT) study, a large, randomized,
controlled trial, has shown a benefit of reamed intramedullary (IM) nailing versus
unreamed IM nailing for closed tibial shaft fractures with regard to reoperation rates. No
such association exists for open tibial fractures; ie, union rates are the same for open
fractures. The infection rates are the same, as is functional outcome, and surgical time is
potentially shorter for unreamed nails. The potential pulmonary benefits from unreamed
nailing have never been clinically proven. PREFERRED RESPONSE: 4

Question 39Figures 39a and 39b are the


radiographs of a 45-year-old man with
diabetes who fell 12 feet from a ladder
and sustained an isolated closed injury
to his left leg. Examination revealed
that he was neurovascularly intact and
compartments were soft. A damage
control knee spanning external fixator
was applied and after 2 weeks in the
frame, his blisters have resolved and
his skin now wrinkles. What is the most
appropriate treatment?

22
22
2012 Musculoskeletal Trauma Self-Assessment Examination by Dr.Dhahirortho
1. Conversion to a periarticular ”hybrid” frame
2. Open reduction and internal fixation with a lateral nonlocking plate
3. Open reduction and internal fixation with a lateral locking plate
4. Open reduction and internal fixation with medial and lateral plates
5. Open reduction and internal fixation with posteromedial and lateral plates

DISCUSSION: The patient has sustained a severely comminuted bicondylar fracture of the
tibial plateau.The mechanism and radiographs highlight the high-energy mechanism of the
injury and should warrant aggressive monitoring for compartment syndrome which is
relatively common in this scenario. A staged surgical approach is warranted with
application of a spanning damage control external fixator to maintain length and alignment
while the soft-tissue injury recovers and to allow for surveillance and examination of the
limb. The radiographs reveal a comminuted bicondylar pattern with significant depression
of the lateral articular surface and a split fracture with condylar widening. This element of
the fracture will require direct elevation of the joint surface and reduction/buttress of the
lateral condyle. This is best achieved with a lateral plate with subchondral rafting screws.
The medial articular surface is coronally split and the posteromedial fragment is displaced.
This fragment requires direct reduction and buttress via a separate posteromedial
approach which is frequently performed prior to the lateral approach and fixation. A lateral
buttress plate or a lateral locking plate alone does not reliably capture or adequately
support the displaced posteromedial fragment. A medial and lateral plate construct is less
soft-tissue friendly, particularly if inserted through a single incision. A medial plate would
also fail to give direct buttress to the posteromedial fragment.

PREF RESP: 5

Question 40A patient with an unstable pelvic ring injury has just undergone an
emergent laparotomy and currently has a packed abdomen. Stabilization of the
pelvic ring is performed with an anterior external fixator. What is an advantage of
using an external fixator with pins in the iliac crest rather than pins in the anterior
inferior iliac spine?

1. Greater pelvic ring stability


2. Lower risk of pin tract infection
3. Less reliance on fluoroscopy for pin placement
4. Better ability to control a posterior pelvic injury
5. Less likely to interfere with future incisions for definitive pelvic internal fixation

DISCUSSION: There are relative advantages to both types of these external fixators. A
frame based on the iliac crest is oftentimes easier to place rapidly because it is less
dependent on fluoroscopy. This is also advantageous in this clinical scenario because the
patient may not be on a radiolucent table. A frame with pins in the anterior inferior iliac
spines may be advantageous in that the pin sites will be away from any future needed
incisions if an ilioinguinal approach is needed. There is, however, a higher risk of lateral
femoral cutaneous nerve injury or intra-articular pin placement at the hip joint with this
frame configuration. This technique is generally more dependent on fluoroscopy for pin
placement. Some biomechanic studies have shown advantages to AIIS-based frames but
this does not give a definite clinical advantage because neither frame alone is adequate to
definitively treat an unstable associated posterior pelvic ring injury. There is no known
difference in pin site infection rates between these frame types.

PREFERRED RESPONSE: 3

23
23
2012 Musculoskeletal Trauma Self-Assessment Examination by Dr.Dhahirortho
Question 41What finding would most likely be present on an AP radiograph of a
nondislocated elbow with an anteromedial coronoid fracture?

1. A “fleck” sign
2. The AP radiograph would appear normal
3. Equal joint spaces between the medial trochlea and the coronoid
4. Progressive narrowing of the joint space from lateral to medial between the medial
trochlea and the coronoid
5. Progressive narrowing of the joint space from medial to lateral between the medial
trochlea and the coronoid

DISCUSSION: Because of the fracture of the anteromedial portion of the coronoid, the
medial side of the elbow is unsupported to varus stresses. As a result, the joint line will
narrow from lateral to medial secondary to medial collapse of the joint. There is no “fleck”
sign for this injury. PREFERRED RESPONSE: 4

Question 42Figure 42a Figure 42bFigures 42a and 42b are the AP pelvis and normal
chest radiographs of a 40-year-old man who was a restrained driver in a high-speed
motor vehicle crash. He reports right hip pain in the emergency department. He was
awake and alert, normotensive, and tachycardic. Hemoglobin was 14
gm/dL.Thoracolumbar radiographs are normal. A closed reduction of the right hip
was performed and the patient was monitored in the emergency department. One
hour later, he was transported to the intensive care unit and, en route, he started to
report chest pain and new onset of lower extremity weakness. He was noted to be
tachycardic and hypotensive. Which of the following studies should be obtained?

1. CT scan of the head


2. CT scan of the pelvis
3. CT scan of the lumbar spine
4. CT angiography of the chest
5. Angiography of the right lower extremity

DISCUSSION: Posterior hip dislocation associated with a deceleration injury is associated


with traumatic rupture of the thoracic aorta. Usually the chest radiograph reveals some
abnormality such as mediastinal widening but this sign may be absent. This patient’s
mental status is normal so an intracranial process is not a likely cause of the change in
status. The patient is not reporting back pain. Posterior dislocation is not commonly
associated with intrapelvic bleeding or lower extremity vascular injury. A high index of
suspicion is necessary; however, in light of the patient’s chest pain and hypotension as
well as lower extremity weakness (likely from spinal cord ischemia); therefore, the most
24
24
2012 Musculoskeletal Trauma Self-Assessment Examination by Dr.Dhahirortho
appropriate study is CT angiography of the chest followed by prompt cardiothoracic
surgical intervention. PREFERRED RESPON: 4

Question 43Figure 43 is the radiograph of a 22-year-old right-hand dominant man


who sustained the injury shown 1year ago. He now reports persistent pain with
activity. Which of the following interventions would most reliably relieve the
patient’s symptoms?

1. Functional bracing
2. Compression plating
3. Iliac crest bone grafting
4. Reamed intramedullary nailing
5. Internal electrical bone stimulation

DISCUSSION: This is a hypertrophic


nonunion of the forearm. Vascularity to
the fracture site has been preserved;
however, there is too much mechanical instability leading to failure of healing.
Compression plating alone is all that is necessary to achieve fracture site stability and
union. Iliac crest bone grafting is needed in atrophic-type nonunions. Internal electrical
bone stimulation is used as an adjunct occasionally,but again is usually reserved for
atrophic nonunion types. This patient has failed to respond to nonsurgical management,
and functional bracing will not provide enough mechanical stability to the nonunion site to
promote union. A plate and screw device provides greater mechanical stability than a
reamed intramedullary nail. Furthermore, use of a reamed intramedullary nail has never
been described for the treatment of a nonunion of the forearm. PREFERRED RESP: 2

Question 44Figure 44cFigures 44a and 44b are the radiograph and axial CT scan of a
24-year-old woman involved in a motor vehicle accident. The patient undergoes
surgery and an intraoperative photograph is seen in Figure 44c.The finding
indicated by the arrow is best described as which of the following?

1. Hip subluxation
2. Marginal impaction
3. Posterior wall fragment
4. Displacement of the transverse fracture
5. Osteochondral injury to the femoral head

DISCUSSION: The CT scan and the intraoperative photograph reveal a classic example of
marginal impaction that is frequently seen with posterior wall acetabular fractures as a
result of the hip dislocating and impacting a small rim of cartilage. Recognition of this injury
is important because it requires special attention for restoration of articular congruency.

25
25
2012 Musculoskeletal Trauma Self-Assessment Examination by Dr.Dhahirortho
Although hip subluxation, displacement of the transverse fracture, and osteochondral injury
to the femoral head are all seen with this injury pattern, the arrow points to the marginal
impaction. PREFE RESP: 2

Question 45Which of the following is associated with increased fetal morbidity and
mortality in acetabular fractures during pregnancy?
1. Fetal position
2. Surgical approach
3. Mechanism of injury
4. Fracture classification
5. Trimester of pregnancy

DISCUSSION: Fixation of pelvic and acetabular fractures in pregnancy is not


contraindicated. However,both maternal and fetal morbidity and mortality is increased in
this patient subset. Factors shown to be associated with increased fetal mortality include:
injury severity, mechanism of injury, and maternal hemorrhage. Surgical approach, fracture
classification, fetal position, and the trimester of pregnancy have not been shown to affect
fetal morbidity or mortality. PRE RES: 3

Question 46Which of the following is the major blood supply to the heel pad?
1. Lateral calcaneal artery
2. Lateral malleolar artery
3. Artery of the sinus tarsi
4. Artery of the tarsal canal
5. Medial calcaneal branch of the posterior tibial artery

DISCUSSION: The medial calcaneal branch of the posterior tibial artery is the major
vascular supply to the heel pad. Heel pad avulsions are severe injuries associated with
high-energy trauma and often carry a poor prognosis because of the potential for heel pad
necrosis. The lateral calcaneal artery and the lateral malleolar artery, along with the lateral
tarsal artery, provide perfusion to the lateral flap associated with a standard extensile
approach to the calcaneus. The artery of the tarsal canal is a branch of the posterior tibial
artery, and the artery of the sinus tarsi is a branch of the perforating peroneal artery. Both
provide perfusion to the talus. PREFERRED RESPONSE: 5

Question 47Figure 47a Figure 47bFigures 47a and 47b are the radiograph and CT
scan of a 45-year-old man who was involved in a highspeed motor vehicle accident.
What is the most appropriate treatment?
1. Subtalar arthrodesis
2. Percutaneous screw fixation
3. Closed reduction and cast application
4. Open reduction and internal fixation
5. Non-weight-bearing and early range of motion

26
26
2012 Musculoskeletal Trauma Self-Assessment Examination by Dr.Dhahirortho
DISCUSSION: The radiograph and CT scan show a displaced talar neck fracture that is
best treated with anatomic reduction and stable internal fixation. Neither non-weight-
bearing and early range of motion nor closed reduction and cast application address the
unstable fracture or restore articular congruity. The fracture is displaced with an interposed
fragment in the fracture line and therefore requires open reduction prior to screw
placement. Subtalar arthrodesis should be reserved for talar fractures with severe injury to
the subtalar joint. PREFERRED RESPONSE: 4

Question 48Figures 48a and 48b are the initial radiograph and clinical photograph of
a 21-year-old woman who sustained a severe mangling injury to her left foot in a
rollover ATV crash. After multiple débridements,she underwent definitive
transmetatarsal amputation with split-thickness skin graft. She did have intact
plantar sensation. Figure 48c shows the clinical photograph 10 days after
amputation. Her wounds healed without infection and she did not require further
surgery. At 2 years after injury, which of these factors is most likely to be
associated with improved level of satisfaction?

1. Female gender
2. Use of skin graft
3. Ability to return to work
4. Retained plantar sensation
5. Initial treatment by amputation

DISCUSSION: The LEAP study is a


multicenter prospective study
evaluating multiple aspects of
reconstruction versus amputation in
the treatment of mangled extremity injuries. With regard to patient satisfaction, treatment
variables such as decision for reconstruction versus amputation, or initial presence or
absence of plantar sensation have little impact. In addition, demographic factors such as
age, gender,socioeconomic status, and education level do not predict patient satisfaction.
Instead, the most important predictors of patient satisfaction at 2 years after injury include
the ability to return to work, absence of depression, faster walking speed, and decreased
pain. PR RESP: 3

Question 49Figure 49Which of the following is expected as a sequela with the use of
a knee-spanning external fixator as a temporary method of
stabilization for the injury shown in Figure 49?

1. Diminished distal pulses


2. An increased rate of pulmonary injury
3. Inability to access soft-tissue envelope
4. Transient compartmental pressure changes
5. Longer time to soft-tissue resolution for definitive surgery

DISCUSSION: The use of bridging external fixation about the knee


for severe tibial plateau fractures is associated with all of the
following: swelling reduction, improved blood flow, access to the
soft-tissue envelope, with only transient increases in
intracompartmental pressure during application. It has no effect on
pulmonary injuries. PREFERRED RESPONSE: 4
27
27
2012 Musculoskeletal Trauma Self-Assessment Examination by Dr.Dhahirortho
Question 50Figure 50cFigure 50a is the radiograph of a 25-year-old man who fell off
his bike, landed on his outstretched elbow, and sustained a closed fracture-
dislocation of the elbow. After urgent closed reduction, he has no neurovascular
compromise. Postreduction radiographs are shown in Figures 50b and 50c. What is
the most appropriate management?
1. Transarticular screw fixation
2. Hinged external fixation and medial collateral ligament repair
3. Radial head excision, coronoid open reduction and internal fixation, and lateral collateral
ligament repair
4. Open reduction and internal fixation of the coronoid, radial head arthroplasty, and lateral
ulnar collateral ligament repair
5. Open reduction and internal fixation of the radial head, repair of the lateral collateral
ligament, and excision of the coronoid fragment

DISCUSSION: The radiographs reveal a terrible triad injury of the elbow. This is a
constellation of injuries involving an elbow dislocation, radial head fracture, and coronoid
fracture and it is notorious for persistent instability and redislocation if treatment is
inadequate. After initial closed reduction, surgical management is indicated to prevent or
limit long-term complications. This involves repair or replacement of the injured radial
head, reduction and fixation of the coronoid/anterior capsule complex, and lateral
ligamentous repair. If there is persistent instability, then additional medial ligamentous
repair should be undertaken. Nonsurgical management with bracing or cast immobilization
is inadequate for this injury.Isolated treatment of either the coronoid or radial head, without
addressing the other injury components is inadequate. Hinged external fixation is
associated with a high complication rate and is generally reserved for salvage cases. The
radial head should not be excised in an unstable elbow injury. Transarticular screw fixation
is reserved for salvage situations. PREFERRED RESPONSE: 4

Question 51A 31-year-old man sustained an unstable closed left posterior hip
dislocation in a motorcycle accident. A postreduction radiograph is shown in Figure
51a. 3-D CT scans are shown in Figures 51b and 51c. What is the optimal surgical
approach that will allow for the most appropriate treatment?

28
28
2012 Musculoskeletal Trauma Self-Assessment Examination by Dr.Dhahirortho
1. Surgical dislocation
2. Watson-Jones approach
3. Smith-Peterson approach
4. Kocher-Langenbach approach
5. Extensile iliofemoral approach

DISCUSSION: The radiograph and CT scans show a posterior wall acetabular fracture
with an associated femoral head fracture. As is the case in most of these injuries, the
femoral head fracture is located on the anterior aspect of the femoral head. Surgical
dislocation with a trochanteric flip osteotomy as described by Solberg and associates and
Henle and associates allows for exposure and treatment of the posterior wall fracture as
well as surgical dislocation for treatment of the femoral head fracture. A Smith-Peterson
approach or Watson-Jones approach would allow for anterior exposure and may help to
address the femoral head fracture, but not the posterior wall fracture. A Kocher-
Langenbach approach would allow exposure of the posterior wall fracture, but not the
femoral head fracture. An extensile iliofemoral approach is unnecessary for this injury
pattern. PREFERRED RESPONSE: 1

Question 52What is the most common complication following surgery for a “terrible
triad” elbow fracture-dislocation?
1. Arthritis
2. Infection
3. Re-dislocation
4. Restricted range of elbow motion
5. Posterior interosseous nerve (PIN) palsy

DISCUSSION: Recurrent instability, PIN palsy, infection, and posttraumatic arthritis have
all been reported following these injuries; however, elbow contracture or loss of motion is
nearly universal following these injuries. PREFERRED RESPONSE: 4

Question 53Figure 53 is the radiograph obtained at the time of transfer to the trauma
center of a 41-year-old man who was involved in a motor vehicle accident. What is
the most appropriate initial management?
1. MRI scan
2. CT scan of the pelvis
3. Application of skeletal traction
4. Closed reduction of the right hip
5. Open reduction and internal fixation

DISCUSSION: The radiograph reveals a displaced


transverse posterior wall acetabular fracture, and
the hip is dislocated. On recognition of a hip
dislocation, the hip should be promptly reduced. A
time to reduction of greater than 12 hours has been
associated with adverse outcomes. Although
skeletal traction and a CT scan are essential
elements in this patient’s care, the hip should be
reduced prior to these actions.An MRI scan is not indicated in this patient, particularly with
the hip dislocated. The ultimate surgical treatment for this injury will be open reduction and
internal fixation, but the patient should be stabilized, the hip reduced, and appropriate
imaging obtained before taking the patient to surgery. PREFERRED RESPONSE: 4

29
29
2012 Musculoskeletal Trauma Self-Assessment Examination by Dr.Dhahirortho
Question 54 are the radiographs of a 23-year-old man who fell from a height and
sustained an isolated injury to his right leg. Which of the following is a useful
surgical technique to optimize alignment during intramedullary nailing?

1. Move the starting point slightly medial


2. Move the starting point slightly lateral
3. Hyperflexion of the knee
4. Anterior blocking screw
5. Medial blocking screw

DISCUSSION: Fractures of the proximal


metadiaphysis of the tibia can be treated
successfully with intramedullary nails, but
previous studies showed rates of
malalignment of up to 84%. The typical
deformity is valgus and procurvatum. An
ideal starting point is mandatory and
should be slightly lateral to the medial
border of the lateral tibial eminence on a
true AP view and very proximal and
anterior on a true lateral view with appropriate coronal and sagittal trajectory of the entry
reamer. A medial start point will exacerbate valgus deformity and should be avoided. A
reduction should be obtained and maintained during reaming, implant insertion, and
interlocking. This can be facilitated via a variety of techniques including intraoperative
external fixation, percutaneous reduction clamps or joysticks, semi-extended positioning,
blocking screws that are typically inserted posterior and lateral to the nail, and ancillary
plate fixation. With careful attention to these techniques, more recent studies report low
rates of malalignment. PREFERRED RESPONSE: 2

Question 55A 27-year-old man who was involved in a high-speed motor vehicle
crash arrives at the trauma center with loss of consciousness, multiple posterior rib
fractures, a left scapula body fracture, a left humerus fracture,
bilateral femoral shaft fractures, and an open right ankle fracture-dislocation. Initial
vital signs are a blood pressure of 88/50 mm Hg, a pulse of 120 bpm, and
respirations of 22/min. His injury severity score is 28 and lactate levels are 2.7. CT
scans of the head and abdomen are negative for hemorrhage, and after initial
resuscitation the patient is cleared for surgery. Initial orthopaedic management
should consist of débridement and irrigation of the right ankle with

1. external fixation, intramedullary nailing of bilateral femurs, and intramedullary nailing of


the left humerus.
2. external fixation, external fixation of bilateral femurs, and splinting of the left humerus.
3. external fixation, external fixation of bilateral femurs, and open reduction and internal
fixation of the left humerus.
4. open reduction and internal fixation, intramedullary nailing of bilateral femurs, and open
reduction and internal fixation of the left humerus.
5. open reduction and internal fixation, external fixation of bilateral femurs, and
intramedullary nailing of the left humerus.

30
30
2012 Musculoskeletal Trauma Self-Assessment Examination by Dr.Dhahirortho

DISCUSSION: The transition from early total care (ETC) to damage control orthopaedics
has developed over the past 20 years in orthopaedic polytrauma situations. A vast number
of studies over the past several years have investigated the timing and technique of
orthopaedic intervention in the care of these patients and effects on outcomes. The goal is
to avoid an iatrogenic second hit that can push a patient into adult respiratory distress
syndrome (ARDS) or multiple organ failure (MOF). In a patient who is unstable,the
algorithm shifts to damage control orthopaedics (DCO). In this patient, the minimum
intervention to achieve orthopaedic stability is indicated; therefore, débridement and
irrigation of open fractures, external fixation of lower extremity long bones, and splinting of
upper extremity fractures is appropriate.
There are several parameters that can help classify a patient’s condition and therefore
guide the surgeon’s initial management (see Figure 55). Grade I is stable and the patient
can be cleared for surgery for early total care. Grade II is borderline and these patients are
found along a continuum. In general, the priority of each injury should be determined and
the most important fixed first. The surgeon should continually check the patient’s status
and proceed as long as the patient is stable. Once the patient shows signs of deteriorating
status, DCO should be implemented for the remaining injuries. Grade III is unstable and
DCO should be the initial course of action. Grade IV is in extremis with life-threatening
injuries and DCO should be implemented if possible. In this scenario, answer choices 1
and 2 involve an ETC philosophy and would be inappropriate for this patient who is
unstable with significant chest trauma. In answer choice 3, the humerus is not a higher
priority than the femur fractures and should be one of the last fractures addressed
surgically. Open reduction and internal fixation of the ankle fracture would not be indicated
in an unstable patient. PREFERRED RESPONSE: 2

Question 56-Figures 56a and 56b are the radiographs of an 88-year-old woman who
had a ground level fall.Examination reveals no neurovascular compromise, and the
skin condition is good. The patient is otherwise in good health, does not take
medication, and lives by herself. What is the most appropriate treatment?

1. Sling
2. Total elbow arthroplasty
3. Long-arm cast immobilization
4. Closed reduction, Kirschner wire fixation, and
cast immobilization
5. Open reduction and internal fixation with
medial and lateral column plates

DISCUSSION: Most distal humerus fractures are


treated with open reduction and plate fixation.
There is increasing evidence that in an elderly,
osteoporotic patient with a comminuted or very
distal fracture,total elbow arthroplasty is
31
31
2012 Musculoskeletal Trauma Self-Assessment Examination by Dr.Dhahirortho
associated with better outcomes than open reduction and internal fixation.Nonsurgical
management, ie, a sling or cast immobilization, is generally reserved for patients with
significant medical conditions that preclude surgery. Kirschner wires and casting is the
treatment of choice in children. In adults, however, the periosteum is not as robust and
does not help in maintaining reduction. Stiffness is inevitable with cast immobilization.
PREFERRED RESPONSE: 2

Question 57-is the radiograph of a 58-year-old woman who is right-hand dominant


and has fallen on her flexed right elbow and is seen in the emergency department
reporting isolated episodes of right elbow pain. Examination reveals that the skin is
contused but intact, and her distal neurovascular examination is normal. What is the
most appropriate treatment?

1. Percutaneous pinning
2. Closed reduction and extension casting
3. Fragment excision and triceps advancement
4. Open reduction and internal fixation with plate fixation
5. Open reduction and internal fixation with tension band
wire construct

Musculoskeletal Trauma Self-Assessment Examination


DISCUSSION: The patient has sustained an isolated, closed
fracture of the olecranon without associated
instability. The bone is radiographically osteopenic and the
fracture is displaced, comminuted, and
includes articular marginal impaction. Plate fixation is
preferred in the presence of comminution or
associated transolecranon or radiocapitellar instability. Displaced fractures are generally
treated surgically
in an effort to restore articular congruity, restore extensor function, and to allow for early
mobilization in an effort to maximize functional outcomes. A tension band wire construct is
a commonly used technique but is reserved for simple fracture patterns without
comminution. Excision and triceps advancement can be considered in elderly, low-demand
patients that have small unreconstructable fracture patterns without associated elbow
instability. PREFERRED RESPONSE: 4

Question 58-A 41-year-old man is involved in a high-speed motor vehicle crash and
sustains a closed femoral midshaft fracture and a unilateral pulmonary contusion
with a hemothorax, requiring placement of a chest tube.He has an initial blood
pressure of 90/50 mm Hg. After receiving two liters of crystalloid, he has a blood
pressure of 115/70 mm Hg and a heart rate of 90 bpm. He has normal mentation and
does not require ventilator support. An arterial blood gas reveals that his delta base
is -2 mmol/L. What is the most appropriate treatment for his femoral fracture?

1. Skeletal traction
2. Temporizing external fixation
3. Reamed intramedullary nailing
4. Unreamed intramedullary nailing
5. Open reduction and internal fixation

DISCUSSION: The patient responded to crystalloid resuscitation and hemodynamic


parameters and the base deficit indicate that he is adequately resuscitated for definitive

32
32
2012 Musculoskeletal Trauma Self-Assessment Examination by Dr.Dhahirortho
fracture care. In a resuscitated patient a reamed nail is not detrimental in the setting of a
pulmonary injury and is favorable for fracture union.
An unreamed nail has a higher nonunion rate than a reamed nail for femoral fractures. In a
skeletally mature patient with a midshaft fracture, an intramedullary nail is preferred to
open reduction and internal fixation. In an adult patient, skeletal traction should be
considered only as a temporary treatment prior to surgical fixation of the femoral fracture.
PREFERRED RESPONSE: 3

Question 59-A 20-year-old concert pianist sustained a diaphyseal radius fracture


and underwent open reduction and internal fixation 3 years ago. She is thin and
reports that the plate is irritating her after playing the piano for an hour or more. She
undergoes elective plate removal of the 3.5 mm plate and 2 weeks later she
refractures the radius. Which of the following statements is most accurate?

1. Diaphyseal plate removals are at higher risk of refracture.


2. Postoperative splinting increases the chance of refracture.
3. The patient would not have sustained a refracture if the plate was 4.5 mm.
4. The risk of fracture increased because the plate was removed within 5 years.
5. Waiting 5 years to remove the hardware would have decreased the risk of refracture.

DISCUSSION: The risk of refracture after hardware removal is multifactorial. Multiple


variables have been looked at, such as protective splinting for 6 weeks after hardware
removal, waiting 12 months or more prior to hardware removal, and the location of the
fracture. The one variable that seems to correlate the most with the risk of refracture is a
diaphyseal location of the initial fracture. Large fragment plates (4.5 mm), when removed,
are at higher risk for refracture in the forearm. PREFERRED RESPONSE: 1

Question 60-are the radiographs of a 42-year-old man who was involved in a


motorcycle crash. His vital signs are a pulse of 122 bpm, a blood pressure of 145/88
mm Hg, and a respiratory rate of 24/min. He has some facial trauma but his Glasgow
coma scale score is 14. His delta base is -2mmol/L. His primary and secondary
surveys reveal no other injury. What is the most appropriate initial management?

1. Distal femoral traction for both injuries


2. External fixation of the pelvis and femur
3. External fixation of the pelvis and intramedullary nailing of the
femur
4. Open reduction of the pelvis and external fixation of the femur
5. Nonsurgical management of the pelvis and intramedullary nailing
of the femur

33
33
2012 Musculoskeletal Trauma Self-Assessment Examination by Dr.Dhahirortho
PREFERRED RESPONSE: 3-DISCUSSION: The patient is relatively hemodynamically
stable and is a candidate for early surgical stabilization of his injuries. The femoral shaft
fracture is best managed acutely with definitive intramedullary nailing. The rotationally
unstable pelvic ring injury should be addressed at the time of femoral fracture fixation. In
this patient, it is best managed acutely by anterior external fixation. Distal femoral traction
would not be optimal for either injury.

Question 61-is the radiograph of a 34-year-old woman who was involved in a


rollover motor vehicle accident. On arrival at the trauma center she is hypotensive
and tachycardic. An abdominal CT scan reveals a spleen laceration. The patient
remains hypotensive despite resuscitation and is taken to surgery for an emergent
laparotomy and splenectomy. After surgery her delta base is -9 mmol/L. What is the
most appropriate management of her pelvic ring injury?

1. Application of a pelvic binder


2. Application of skeletal traction
3. Open reduction and internal fixation
4. Placement of percutaneous iliosacral screws
5. Placement of an anterior pelvic external fixator

DISCUSSION: The patient has a displaced iliac


wing and bilateral rami fractures with superior
migration of the right hip. To prevent further
deformity, the patient’s right hip should be placed
into skeletal traction. A pelvic binder may worsen
the deformity because of the iliac wing fracture.
Anterior external fixation is problematic with an iliac
wing fracture. Although the patient will eventually
require open reduction and internal fixation and/or placement of iliosacral screws, she
must be stabilized and resuscitated prior to this treatment. PREFERRED RESPONSE: 2

Question 62-When planning pin placement for external fixation of the tibia, what is
the maximum extent of the knee capsular reflection from the subchondral joint line?
1. 4 mm
2. 6 mm
3. 10 mm
4. 14 mm
5. 20 mm

DISCUSSION: Intracapsular pin placement is a concern for septic arthritis. Reid and
associates and DeCoster and associates have demonstrated that the maximum distal
extent of the knee capsule is 14 mm from the subchondral line and occurs in the
posterolateral region. The recommended placement of external fixation pins is greater than
14 mm from the subchondral line of the proximal tibia. PREFERRED RESPONSE: 4

Question 63- are the radiographs of a 24-year-old left-hand dominant man who felt a
“snap” in the left mid-arm while arm wrestling. Examination reveals an isolated
closed injury. He has significant pain and gross instability about the upper arm. His
compartments are soft and he has good pulses with a wellperfused hand. He is
unable to actively extend his wrist and fingers. Following splinting of the arm, his
examination findings remain unchanged. What is the next most appropriate step in
management?
34
34
2012 Musculoskeletal Trauma Self-Assessment Examination by Dr.Dhahirortho

1. Splintage followed by functional bracing


2. Intramedullary nailing with radial nerve exploration
3. Intramedullary nailing without radial nerve exploration
4. Open reduction and internal fixation with radial nerve exploration
5. Open reduction and internal fixation without radial nerve exploration

DISCUSSION: The patient has sustained an isolated, closed, comminuted fracture of the
humeral diaphysis with an associated radial nerve palsy. In a report of 922 patients,
Sarmiento and associates noted a nonunion rate of less than 2% with overall acceptable
alignment and satisfactory functional results. Even in the presence of a radial nerve palsy,
most humeral shaft fractures should be treated with functional bracing. Whereas open
reduction and internal fixation and intramedullary nailing are viable treatment options, this
fracture is acceptably aligned and is amenable to closed treatment. The absolute
indications for surgical management include vascular injury, severe soft-tissue injury
precluding closed treatment, compartment syndrome, open fracture, and associated
ipsilateral forearm fracture, ie, floating elbow. Relative indications for surgical management
include: segmental fracture, intra-articular extension,transverse fracture pattern with
significant distraction, bilateral fracture, patient factors that preclude closed treatment such
as head injury or morbid obesity, inability to maintain acceptable alignment via closed
means, and polytrauma. PREFERRED RESPONSE: 1

Question 64-are the radiographs of a 62-year-old woman who is seen in the


emergency department after a fall to a flexed knee. She underwent posterior-
stabilized total knee arthroplasty 6 years ago. She has no other injuries and was
previously a community ambulatory without assistance. What is the most
appropriate treatment?

1. Closed reduction and casting


2. Closed reduction and fracture bracing
3. Retrograde intramedullary nailing
4. Minimally invasive plate osteosynthesis
5. Revision total knee arthroplasty with distal
femoral replacement

DISCUSSION: The patient has a displaced


periprosthetic fracture just proximal to a
posterior-stabilized total knee arthroplasty.
Whereas distal bone stock is limited and there
35
35
2012 Musculoskeletal Trauma Self-Assessment Examination by Dr.Dhahirortho
is evidence of radiographic osteopenia, there does not appear to be disruption of the bone-
prosthetic interface and the prosthesis appears to be well fixed. Surgical fixation is
indicated to restore alignment and allow for early range of motion and mobilization in an
effort to optimize functional outcomes. Unless the patient is unable to tolerate surgical
management, closed reduction and casting or functional bracing should be reserved for
stable, minimally displaced or impacted fracture patterns. Whereas retrograde nailing is a
viable treatment option, it can be difficult in the presence of a posterior-stabilized prosthetic
design and distal fixation can be problematic in the setting of poor bone stock or quality.
Minimally invasive plate osteosynthesis using modern-locking periarticular plates allows for
indirect reduction techniques in an effort to optimize the biologic milieu and offers optimal
biomechanical stability. Revisional total knee arthroplasty with a distal femoral replacement
is not a good option for a young patient.
PREFERRED RESPONSE: 4

Question 65-A patient undergoes open reduction and internal fixation of a displaced
radial neck fracture. What position should the forearm be in during the approach
and during fixation?

1. Supinated during the approach and neutral for plate application


2. Neutral during the approach and pronated for plate application
3. Pronated during the approach and neutral for plate application
4. Pronated during the approach and pronated for plate application
5. Pronated during the approach and supinated for plate application

DISCUSSION: Pronating the forearm during the surgical approach decreases the danger
to the posterior interosseous nerve by moving it away from the surgical field. Placing the
plate straight lateral with the forearm in neutral rotation puts the plate in the safe zone that
does not articulate with the proximal radioulnar joint to prevent impingement of the plate
during forearm rotation.
PREFERRED RESPONSE: 3

Question 66-A 45-year-old man sustained bilateral femur fractures in a motorcycle


accident. On admission to the emergency department, the patient is unconscious
with a heart rate of 120 bpm and a systolic blood pressure of 80 mm Hg. A chest
radiograph reveals bilateral pulmonary contusions. After resuscitation with 2 L of
crystalloid, the patient’s heart rate is 115 bpm, the systolic blood pressure is 90 mm
Hg, and the patient’s delta base is -10 mmol/L. What is the most appropriate
treatment for the femoral fractures at this point?

1. External fixation
2. Percutaneous plating
3. Reamed antegrade intramedullary nailing
4. Reamed retrograde intramedullary nailing
5. Unreamed antegrade intramedullary nailing

DISCUSSION: The patient is in shock, has pulmonary contusions, and needs further
resuscitation.Damage control orthopaedics (ie, external fixation of the femoral fractures) is
rapid, safe, and assists in patient resuscitation. Although the patient may ultimately
undergo definitive treatment with reamed intramedullary nailing or percutaneous plating, to
do so at this time would not further assist resuscitation and may increase pulmonary
dysfunction. Unreamed intramedullary nailing is not indicated in this patient.
PREFERRED RESPONSE: 1

36
36
2012 Musculoskeletal Trauma Self-Assessment Examination by Dr.Dhahirortho
Question 67-What approach should be chosen for the injury seen in Figure 67?
1. Stoppa
2. Hardinge
3. Ilioinguinal
4. Watson Jones
5. Kocher-Langenbeck

DISCUSSION: The injury shown is a transverse


plus posterior wall fracture of the acetabulum.
The anterior and posterior columns are
fractured as one piece, the ischiopubic
segment, and need to be reduced along with
the posterior wall. The Kocher-Langenbeck
approach will allow visualization of the
transverse component through the joint and also direct exposure of the posterior wall. The
ilioinguinal approach would not allow visualization or access to the posterior wall. The
Stoppa, Watson Jones, and the Hardinge approaches do not allow adequate visualization
of the posterior column which is needed to reduce the posterior wall. The other option is
the extended iliofemoral approach which would allow excellent visualization and ability to
reduce this fracture. PREFERRED RESPONSE: 5

Question 68-are the radiographs of a 58-year-old right-hand dominant woman who


fell from a standing height directly onto her left shoulder and now reports left
shoulder pain and is unable to elevate her arm. She has a normal sensory
examination. The patient refuses any type of surgical intervention.What factor will
have the greatest impact on her outcome at 1 year?
1. Age
2. Bone quality
3. Hand dominance
4. Initial angulation of fracture
5. Use of a physical therapy program

DISCUSSION: In a review of over 1,000 proximal humerus fractures, Court-Brown and


McQueen looked at outcomes of impacted varus fractures. These accounted for 13% of
the fractures in their review. All impacted varus fractures, with the exception of two, were
treated nonsurgically and were followed for1 year. They determined that the age of the
patient was the major factor in overall outcome. Good results can be expected with
younger patients, but results deteriorate with advancing age. In regards to angulation of
the fracture, they found no correlation between increased varus angulation and shoulder
function. Most patients had good or excellent results no matter how much final varus was
achieved.
They also noted that physical therapy did not have a significant impact on outcome. In their
study, poor results in patients who lacked therapy were more related to the advanced age
of the patients. Hanson and associates prospectively evaluated nonsurgical management
of 124 proximal humerus fractures for 1year. They found that displacement of the fracture
only gradually influenced the constant scores and that the DASH revealed the patients on
average had not fully recovered at 1 year. They also noted that 97.6% of employed
patients were able to return to work and that employed patients on average had
significantly lower differences in side-to-side constant and DASH scores. Solid bony union
was seen in 93% at 1 year and fracture consolidation was seen in 98%. The predicted risk
of delayed and nonunion was 7% with smoking increasing the nonunion risk by 5.5 times.
PREFERRED RESPONSE: 1

37
37
2012 Musculoskeletal Trauma Self-Assessment Examination by Dr.Dhahirortho
Question 69-is the radiograph of a 52-year-old right-hand dominant man who fell
while skiing. He was initially treated at a mountainside clinic where he was placed in
a sling. He now reports moderate shoulder pain but has no other complaints. What
is the most appropriate management?

1. Hemiarthroplasty
2. Total shoulder arthroplasty
3. Open reduction and internal fixation
4. Sling for 6 to 8 weeks followed by mobilization
5. Sling followed by early mobilization within 2 to 3 weeks

DISCUSSION: The patient sustained a four-part valgus


impacted fracture of the proximal humerus with a
significantly increased head-shaft angle and resultant
displacement of the greater and lesser tuberosities.In an
effort to restore anatomic alignment and optimize
functional outcomes, open reduction and internal fixation
is indicated. A valgus impacted pattern has been noted to
have improved outcomes and lower
rates of osteonecrosis. In addition, this patient has
adequate bone stock and all efforts should be made to
salvage his shoulder. A sling will not change the overall
alignment and is therefore not the most appropriate treatment option. Hemiarthroplasty is
reserved for unreconstructable patterns, and total shoulder arthroplasty would only be
considered in the acute scenario if there was significant preexisting glenoid wear or
inflammatory arthropathy. PREFERRED RESPONSE: 3

Question 70-Pelvic packing for a hemodynamically unstable patient with a pelvic


ring fracture is best described by which of the following techniques?

1. Placing a pelvic external fixator followed by packing the pelvis with lap pads via a
subumbilical incision
2. Placing lap pads for packing via a subumbilical incision in the angiography suite
3. Placing lap pads for packing using the lateral window of the ilioinguinal approach
(anterior approach to the internal iliac fossa)
4. Packing the retroperitoneum with lap pads after exploration of the abdomen by the
general surgeons
5. Direct exploration of the pelvic vasculature via a midline incision followed by packing
with lap pads

DISCUSSION: For the technique of pelvic packing patients are placed supine on an
operating room table. For rotationally and/or vertically unstable fracture patterns, an
external fixator is then placed to stabilize the pelvis so that the volume of the pelvis is
decreased and the packing has counterforce acting against it.
An approximately 6 cm to 8 cm midline incision is made extending upwards from the pubic
symphysis and heading toward the umbilicus. The rectus fascia is then divided in the
midline. The bladder is retracted to one side and three lap pads are packed deep to the
pelvic brim. The bladder is retracted to the other side and three more lap pads are placed
on that side as well. The first sponge is placed at the level of the sacroiliac joint, the
second anterior to the first sponge, and the third in the retropubic space lateral and just
deep to the bladder. All should be placed below the level of the pelvic brim. The fascia is

38
38
2012 Musculoskeletal Trauma Self-Assessment Examination by Dr.Dhahirortho
then closed. If the patient is hemodynamically unstable after stabilization, then packing of
the pelvis angiography should be considered. PREFERRED RESPONSE: 1

Question 71-A previously healthy man who weighs 70 kg (154 lb) sustains an acute
blood loss of 2 liters after a motorcycle crash. Which of the following statements
about physiologic parameters is unique to this amount of blood loss?

1. Pulse pressure will be widened.


2. Urine output will be at the lower limits of normal.
3. Tachycardia will be present, but with no change in systolic blood pressure.
4. Systolic blood pressure will be decreased with a narrowed pulse pressure.
5. Systolic blood pressure will be maintained with an elevated diastolic blood pressure.

DISCUSSION: The normal adult blood volume is approximately 7% of body weight. For
example, a man weighing 70 kg has a circulating blood volume of approximately 5 liters. A
blood loss of 2 liters places the patient in a class IV hemorrhage of more than 40% blood
volume loss. Signs and symptoms of class IV hemorrhage include marked tachycardia of
greater than 140 bpm, a significant decrease in blood pressure, and a very narrow pulse
pressure. Urinary output is negligible, and mental status is markedly depressed. The skin
is cold and pale. Physiologic parameters associated with a class II hemorrhage include:
urine output that is at the lower limits of normal; the presence of tachycardia but with no
change in the systolic blood pressure; and maintenance of the systolic blood pressure with
an elevated diastolic blood pressure. A widened pulse pressure correlates with a class I
hemorrhage. PREFERRED RESPONSE: 4

Question 72- are the radiographs and CT scans of a 45-year-old man who fell 10 feet
from a ladder and sustained an injury to the right knee. Examination reveals no
open wounds and the skin was in good condition with moderate swelling and no
fracture blisters. The patient is neurovascularly intact.What is the most appropriate
treatment?

1. Hinged knee brace and non-weight-bearing for 6 weeks


2. Percutaneous screw fixation
3. Open reduction and internal fixation with a laterally applied nonlocking plate
4. Open reduction and internal fixation with posteromedial and lateral plates via one
anterior approach
5. Open reduction and internal fixation with posteromedial and lateral plates via dual
incisions

39
39
2012 Musculoskeletal Trauma Self-Assessment Examination by Dr.Dhahirortho
DISCUSSION: The patient has a bicondylar tibial plateau fracture with metadiaphyseal
separation,depressed lateral articular surface, and a medial articular coronal split. This
fracture is ideally treated with open reduction and internal fixation. A nonlocking lateral
plate may not be able to protect against varus collapse through the metaphysis as well as
a locking plate. However, neither plate can reliably deal with reduction and reliable fixation
of the medial articular fracture. Recent studies have shown the relative frequency of the
so-called “posteromedial fragment,” and have recommended supplemental fixation of the
medial articular surface because standard lateral implants may not be able to gain screw
purchase in the posteromedial bone. Other studies have shown good fracture reduction
and maintenance of reduction with low complication rates using the two-incision technique
with double plating. Placing dual plates via one anterior incision is associated with soft-
tissue complications. PREFERRED RESPONSE: 5

Question 73-are the radiographs of a 57-year-old woman who twisted her ankle
stepping off a curb and now has lateral foot pain. Examination reveals intact skin
and point tenderness over the base of the fifth metatarsal. What is the most
appropriate treatment?

1. Fragment excision
2. Intramedullary screw fixation
3. Non-weight-bearing cast for 6 weeks
4. Percutaneous Kirschner wire fixation
5. Immediate weight bearing with a postoperative shoe

DISCUSSION: The patient has an avulsion fracture of the fifth metatarsal base. Unlike a
true Jones-type fracture, this is amenable to immediate weight bearing. Intramedullary
screw fixation may be indicated in select patients with a Jones fracture but not with this
fracture type. Fragment excision can be considered in the presence of a symptomatic
nonunion of this fracture, but is not indicated acutely. Percutaneous pinning is not
indicated.

PREFERRED RESPONSE: 5

40
40
2012 Musculoskeletal Trauma Self-Assessment Examination by Dr.Dhahirortho
Question 74-Which of the following factors is associated with improved outcomes
following surgery for hip fractures?

1. Immediate surgical intervention


2. Early discharge to a skilled nursing facility
3. Choosing spinal versus general anesthesia for surgery
4. Choosing total hip arthroplasty instead of hemiarthroplasty for a displaced femoral neck
fracture
5. Correction of metabolic abnormalities prior to surgical intervention

DISCUSSION: Many studies have looked at patient outcomes following hip fracture
surgery. While early surgery in these patients is recommended, medical optimization prior
to surgical intervention is warranted in all cases. Anesthetic type and discharge status
have not been proven to alter patient outcomes. Total hip arthroplasty has improved
function at 1 year compared with hemiarthroplasty; no changes in mortality have been
reported. PREFERRED RESPONSE: 5

Question 75- are the CT scans and radiographs of a 56-year-old man who is a
restrained driver involved in a motor vehicle accident in which his car is struck at 35
mph. He has pain to the right buttock and groin regions. Examination reveals that he
is neurologically intact and has no evidence of other injuries. What is the most
appropriate management at this time?

1. Anterior pelvic external fixation


2. Iliosacral screw fixation on the right side
3. Open reduction and internal fixation of the sacrum
4. Bed rest for 1 to 2 weeks, followed by non-weight-bearing on the right side for 4 weeks
5. Immediate mobilization and weight bearing as tolerated on the right lower extremity

41
41
2012 Musculoskeletal Trauma Self-Assessment Examination by Dr.Dhahirortho
DISCUSSION: The patient has a lateral compression-type pelvic fracture with an
incomplete fracture of the sacrum. This is a stable pattern that will tolerate immediate
weight bearing with little risk of displacement. Anterior pelvic external fixation is
occasionally indicated for a lateral compressiontype pelvic fracture to reduce a severe
internal rotation deformity, but that is not present in this patient.Iliosacral screw fixation
may be indicated if the patient has pain that prevents mobilization with this injury or in the
case of a sacral fracture that is complete or comminuted, which can indicate a higher risk
of displacement. PREFERRED RESPONSE: 5

Question 76-How is the fracture pattern shown in Figures 76a through 76c best classified?

1. Moore 1
2. Moore 3
3. Schatzker 3
4. Schatzker 6
5. OTA type 41C

DISCUSSION: This fracture is best classified as a “rim avulsion pattern”: Moore type 3. A
Moore 1 is a posteromedial shear fracture associated with subluxation of the tibiofemoral
joint. This classification is useful for fractures associated with knee instability and patterns
that do not fit into the Shatzker classification. A Shatzker 3 is a pure depression type, and
a type 6 has metaphyseal-diahyseal dissociation.OTA type C are complete articular
fractures. PREFERRED RESPONSE: 2

Question 77-are the radiographs of a 45-year-old man who fell into a ditch and
sustained a twisting injury to the left lower extremity. Examination in the emergency
department reveals normal neurologic function, no skin compromise, and palpable
pedal pulses. The patient has no other complaints and has an otherwise normal
examination. What additional imaging study is recommended?

1. MRI of the left knee


2. Arterial duplex of the left leg
3. CT scan of the left ankle
4. CT scan of the chest, abdomen, and pelvis
5. CT scan of the cervical, thoracic, and lumbar spine

42
42
2012 Musculoskeletal Trauma Self-Assessment Examination by Dr.Dhahirortho

DISCUSSION: Spiral distal tibia fractures are frequently associated with intra-articular
fracture extension,usually involving the posterior malleolus. This may or may not be visible
on the radiographs. A CT scan of the ankle is recommended to identify this associated
injury. This is especially important when considering intramedullary nail fixation of the
distal tibia fracture because a previously nondisplaced intraarticular fracture may become
displaced as the nail is inserted to its final depth. Anteroposterior screw fixation prior to
nailing may be useful in these cases. With the patient’s history, there is no indication of
thoracic, abdominal, pelvic, or spinal trauma. There are no signs of vascular injury and
preoperative MRI is not indicated. PREFERRED RESPONSE: 3

Question 78-Long-term alendronate (Fosamax) use for osteoporosis has been


associated with which of the following?
1. Scurvy
2. Detached retina
3. Uterine carcinoma
4. Osteonecrosis of the femoral head
5. Diaphyseal femoral insufficiency fractures

DISCUSSION: Alendronate is a bisphosphonate that inhibits the ruffled border of the


osteoclast. When used long term, this class of medication prevents the normal bone
remodeling process. Long-term use has recently been shown to be associated with
insufficiency fractures of the femur. Osteonecrosis of the jaw has been described but not in
other anatomic locations. Scurvy occurs because of a lack of vitamin C and use of
bisphosphonates is not associated with uterine cancer or a detached retina.
PREFERRED RESPONSE: 5

Question 79-are the radiographs of a 78-year-old right-hand dominant man who fell
at home and sustained an isolated injury to his right shoulder. He lives alone and is
independent with his activities of daily living. Examination reveals a closed injury
and a normal neurologic examination. What is the most appropriate management?
1. Shoulder hemiarthroplasty
2. Hanging arm cast for 6 to 8 weeks
3. Shoulder immobilizer for 6 weeks, followed by range-of-motion exercises
4. Open reduction and internal fixation of the proximal humerus with a locking plate
5. Sling, followed by pendulum exercises with elbow range of motion within 1 to 2 weeks
43
43
2012 Musculoskeletal Trauma Self-Assessment Examination by Dr.Dhahirortho

DISCUSSION: The patient has an isolated proximal humerus fracture in acceptable


alignment, thus surgical treatment is not indicated. Multiple studies have shown the
benefits of earlier mobilization and therapy in patients who are treated nonsurgically for
proximal humerus fractures. A hanging arm cast can be used in proximal humerus
fractures, but typically would not be recommended for a length of 6 to 8 weeks because a
prolonged time in a hanging arm cast has the disadvantage of immobilizing the elbow
during that time as well. PREFERRED RESPONSE: 5

Question 80-A 38-year-old woman is polytraumatized in a motor vehicle crash. She


has multiple injuries including a unilateral femur fracture. The patient is felt to be
borderline and, although she is currently stable,she could potentially deteriorate
quickly. Which of the following parameters has been suggested as an indicator of
which patients would benefit from damage control?

1. Normothermia
2. Hemoglobin of less than 9 g/dL
3. Unilateral lung contusion evident on CT only
4. Injury severity score of greater than 40 without thoracic injury
5. Injury Severity Score of less than 18 with a pulmonary contusion

DISCUSSION: Polytraumatized patients can be classified as stable, unstable, borderline,


or in extremis.Management of the borderline patient is controversial because it is unclear
which patients can safely undergo early definitive surgical stabilization of fractures, and
which patients would benefit from temporizing “damage control” stabilization to allow
adequate resuscitation and physiologic stabilization prior to definitive treatment. Although
the question of damage control versus early total care is unresolved, there are several
clinical parameters that have been suggested for use in deciding who should be treated
with early damage control. These include Injury Severity Score of greater than 40, Injury
Severity Score of greater than 20 with thoracic trauma, multiple injuries with severe
pelvic/abdominal trauma and hemorrhagic shock, bilateral femoral fractures, pulmonary
contusion noted on radiographs,hypothermia of less than 35 degrees C), and a head injury
with an Abbreviated Injury Score of 3 or greater. A hemoglobin of 9 g/dL is not included in
these suggested parameters. PREFERRED RESPONSE: 4

44
44
2012 Musculoskeletal Trauma Self-Assessment Examination by Dr.Dhahirortho
Question 81-A 37-year-old man is evaluated in the emergency department after a
motor vehicle accident at 40 mph. He reports low back and buttock pain and
numbness in his perineum. After initial radiographic evaluation of his pelvis, he is
advised to attempt weight bearing but is unable to because of severe pain. A CT
scan of the pelvis is ordered. The radiographs and CT scan are shown in Figures
81a through 81d. What study should be obtained next?

1. CT scan of the abdomen


2. Judet views of the pelvis
3. MRI scan of the lumbar spine
4. Lateral radiograph of the sacrum
5. Electromyography of the lumbosacral plexus

DISCUSSION: The patient has a U-type fracture of the sacrum. The best way to visualize
this fracture is with a lateral view of the sacrum or sagittal reformatted images of the CT
scan. Standard pelvic radiographs often miss this injury. A high index of suspicion must be
maintained for a transverse fracture component (H- or U-type fracture) in a patient with
bilateral sacral injuries, especially without any anterior pelvic ring fractures. This injury
occurs by a different mechanism than pelvic ring disruptions,thus the sacrum will fracture
oftentimes without associated anterior pelvic injuries. These injuries have a high rate of
associated neurologic injury. Treatment of these injuries varies based on neurologic
compromise and displacement. PREFERRED RESPONSE: 4

Question 82-are the radiographs of a 52-year-old woman who fell down the stairs
and sustained an acute hemarthrosis of the elbow. What is the most common
complication following surgical treatment of this injury?

1. Elbow contracture
2. Complex regional pain syndrome
3. Posterior interosseous nerve palsy
4. Bridging heterotopic ossification of
the elbow
5. Elbow instability requiring
ligamentous reconstruction

DISCUSSION: The patient has a


displaced capitellum fracture that
requires surgical intervention. Wheras
complex regional pain syndrome,
posterior interosseous nerve palsy,

45
45
2012 Musculoskeletal Trauma Self-Assessment Examination by Dr.Dhahirortho
bridging heterotopic ossification of the elbow, and elbow instability requiring ligamentous
reconstruction are seen as sequelae of various traumatic elbow injuries, elbow contracture
is frequently seen following this specific injury. PREFERRED RESPONSE: 1

Question 83-The vessel that is exposed crossing the interval used for an anterior
approach to the hip between the tensor fascia lata and the sartorius muscle is a
branch of what artery?

1. Deep femoral artery (profunda)


2. Superficial femoral artery
3. Superior gluteal artery
4. Descending recurrent femoral artery
5. External iliac

DISCUSSION: The ascending branch of the lateral femoral circumflex artery crosses the
surgical field between the tensor fascia lata and the sartorius muscles and has to be
ligated during this approach. It is a branch of the profunda femoris artery.
PREFERRED RESPONSE: 1

Question 84-Three years following repair of a subtrochanteric femur fracture, a 26-


year-old man has a draining sinus shown in Figure 84a. Radiographs are shown in
Figures 84b and 84c. Management should consist of which of the following?

1. Hip disarticulation
2. Infectious diseases consultation and long-term suppressive antibiotics
3. Incision and drainage, removal of hardware, excision of heterotopic bone, and culture-
directed
antibiotics
4. Excision of heterotopic bone and radiation therapy to prevent recurrence
5. Excision of heterotopic bone and administration of nonsteroidal anti-inflammatory drugs
to prevent recurrence

DISCUSSION: This is a case of chronic osteomyelitis with infected hardware and a healed
fracture. Limb salvage should be attempted in the patient. The presence of a chronic
draining sinus requires surgical débridement . Removal of the implant and as much of the
infected heterotopic bone will reduce the bacterial load. Culture-directed antibiotics are
started after deep cultures are obtained. Infectious diseases consultation is obtained
46
46
2012 Musculoskeletal Trauma Self-Assessment Examination by Dr.Dhahirortho
following surgical intervention. Long-term suppressive antibiotics are not the treament of
choice in healthy patients who can tolerate a surgical procedure. PREFERRED RESP: 3

Question 85-Which of the following is the best predictor of mortality after a patient
has sustained a pelvic ring injury?
1. Gender
2. Comorbidities
3. Fracture pattern
4. Use of angiography
5. Shock on presentation

DISCUSSION: Starr and associates demonstrated that age and shock on presentation
were predictors of mortality after pelvic ring injury. Smith and associates showed that the
amount of blood transfusions in the first 24 hours was also predictive of mortality. Gender,
fracture pattern, use of angiography, and comorbidities do not correlate directly with
mortality. PREFERRED RESPONSE: 5

Question 86-What is the best way to determine whether a radial head implant is too
thick intraoperatively?

1. Visually assess the radiocapitellar joint.


2. Visually assess widening of the lateral ulnohumeral joint.
3. Assess widening of the radiocapitellar joint on an AP radiograph.
4. Assess the elbow for concentric reduction on a lateral radiograph.
5. Assess widening of the medial ulnohumeral joint on an AP radiograph.

DISCUSSION: Widening of the medial ulnohumeral joint on an AP radiograph is only


visible after overlengthening of the radial head by 6 mm or more. At least in this cadaver
study, the most sensitive method was to visually assess the lateral aspect of the
ulnohumeral joint with the radial head resected and then with the trial radial head in place.
This method allows detection of any overlengthening. PREFERRED RESPONSE: 2

Question 87-are the radiographs of an 18-year-old pedestrian who was struck by a


car. During intramedullary nailing, it is difficult to maintain proper alignment. Poller
blocking screws placed in the proximal fragment at which position(s) relative to the
nail can help prevent the typical deformity?

1. Anterior only
2. Anterior and medial
3. Anterior and lateral
4. Posterior and medial
5. Posterior and lateral

47
47
2012 Musculoskeletal Trauma Self-Assessment Examination by Dr.Dhahirortho
DISCUSSION: This is a proximal one third tibial shaft fracture. Typically nailing of this
fracture creates a valgus and procurvatum malalignment that must be addressed. This can
be difficult when using an intramedullary nail in the wide metaphyseal bone of the proximal
tibia. To help direct and center the nail in the metaphysis, blocking screws can be used.
Blocking screws should be placed where the nail should not travel. If the nail was passed
with the proximal fragment in this position, it would occupy the lateral and posterior
aspects of the metaphyseal fragment. To prevent this, blocking screws should be placed in
the lateral and posterior aspects of the proximal fragment. PREFERRED RESPONSE: 5

Question 88-What mechanism of injury is most likely to cause a fracture of the


anteromedial facet of the coronoid?
1. Extension and axial load
2. Varus and posteromedial rotation
3. Valgus and posteromedial rotation
4. Varus and posterolateral rotation
5. Valgus and posterolateral rotation

DISCUSSION: The mechanism of injury in a fracture of the anteromedial facet of the


coronoid is typically a varus and posteromedial rotation force on the forearm which is the
opposite of a terrible triad injury. First, the lateral collateral ligament is injured and then the
medial coronoid is compressed against and then under the medial trochlea. PRE RESP: 2

Question 89-a is the radiograph of a 24-year-old man who was involved in a motor
vehicle accident. A closed reduction is performed and a post-reduction CT scan is
shown in Figure 89b. What is the next most appropriate step in management?

1. Total hip arthroplasty


2. Removal of loose bodies
3. Protected weight bearing
4. Assessment of hip stability
5. Open reduction and internal fixation

DISCUSSION: The radiograph and CT scan show a posterior hip dislocation with an
associated posterior wall acetabular fracture. The next step in management is assessment
of hip instability. As suggested by Tornetta, assessment of hip instability with dynamic
stress views is helpful to determine which posterior wall fractures are unstable and
therefore require open reduction and internal fixation. Although protected weight bearing
may be correct if the hip is stable, stability needs to be determined first. The CT scan
reveals a small fragment in the cotyloid fossa. However, in this location, the presence of a
loose body alone does not require surgical treatment. Hip instability needs to be assessed
before determining if this fracture should be treated with open reduction and internal
fixation. Total hip arthroplasty is not appropriate for a 24-year-old patient with a small
posterior wall acetabular fracture.

PREFERRED RESPONSE: 4

48
48
2012 Musculoskeletal Trauma Self-Assessment Examination by Dr.Dhahirortho
Question 90-are the radiographs of the right leg of a 30-year-old man who sustained
a crush injury to his right chest, abdomen, and right leg after being pinned under a
hydraulic jack. He has a blood pressure of 170/90 mm Hg. He is intubated and
sedated secondary to his pulmonary injury. Six hours later he has a swollen lower
leg. Examination reveals significant swelling but palpable pulses.Compartment
pressures ranged from 32 to 41 mm Hg. What is the next step in management?

1. MRI of right leg


2. Venous doppler of right leg
3. Four-compartment fasciotomy
4. Follow-up examination the next day
5. Serial examinations with compartment
pressures

DISCUSSION: The patient is at risk for a


compartment syndrome. Management
should consist of close follow-up with serial
examinations and repeat compartment
pressure measurements as long as the
patient cannot give a good clinical
examination. MRI scan of the leg is not
needed acutely because the scenario is
suggestive of a crush injury and the most
likely problem is muscle injury. Venous
doppler,although important to discern the
possibility of a venous occlusion, is not the
most pressing issue. Fourcompartment
fasciotomy may become necessary but
based on the available data is not indicated at this time. The current pressure difference
between the diastolic blood pressure and his compartment pressure is almost 50 mmHg,
suggesting the microcirculation is open. PREFERRED RESPONSE: 5

Question 91-is the radiograph of a 20-year-old man who kicked a door while
intoxicated. At the emergency department, his leg is placed into a long-leg cast.
After 2 hours, he reports increasing pain, numbness, and tingling in his toes. What
is the most appropriate initial treatment?

1. Elevate leg on pillows


2. Administer IV morphine
3. Observation of the patient
4. Bivalve and spread the cast
5. Apply ice to the lower extremity

DISCUSSION: The patient appears to have some


indications of a compartment syndrome: increasing
pain and signs of nerve compression. Tibia
fractures also should heighten the suspicion for a
compartment syndrome. Two basic mechanisms of
compartment syndrome are that an increase in
volume occurs in an enclosed space or there is a
decrease in size of the space. In this situation, both
are likely occurring; postfracture swelling is
occurring within a closed space and if a cast is in
49
49
2012 Musculoskeletal Trauma Self-Assessment Examination by Dr.Dhahirortho
place that may constrict the space even more. One way to increase the available space for
swelling would be to bivalve and spread the cast.If the extremity has been casted, then it is
vitally important that the cast is bivalved and the surrounding soft dressings under the cast
be removed so that all external compression of the compartment has been eliminated. In
the face of compartment syndrome, elevation of the limb, masking the pain with
morphine,application of ice, or observation alone are all inappropriate.
PREFERRED RESPONSE: 4

Question 92-is the radiograph of a 45-year-old man who was thrown from his horse
and now reports groin pain. Which of the following is the most common long-term
sequelae of this injury?

1. Gait abnormality
2. Sexual dysfunction
3. Chronic low back pain
4. Quadriceps weakness
5. Posttraumatic osteoarthritis

DISCUSSION: The radiograph reveals an anterior posterior compression injury to the


pelvic ring which is commonly seen after horseback riding injuries. In a large series of
patients with this type of injury,18 of 20 patients had sexual dysfunction after sustaining
this injury. Posttraumatic osteoarthritis of the sacroiliac joints may occur, but is less
common in this type of injury. Chronic low back pain, gait abnormalities, and quadriceps
weakness are not typically seen with this type of injury. PREFERRED RES: 2

Question 93-A 23-year-old woman is involved in a motorcycle accident. She


sustains bilateral femur fractures(Abbreviated Injury Score [AIS]=3), an intra-
abdominal injury (AIS=3), facial fractures (AIS=2), and a pulmonary injury (AIS=2).
What is her Injury Severity Score (ISS)?

1. 13
2. 18
3. 22
4. 27
5. 35

DISCUSSION: The ISS is calculated as the sum of the squares of three highest AIS
scores from the six body regions, thus this patient’s ISS score is 22. The ISS does
correlate with mortality, but the ISS does not score multiple injuries to the same body
region, hence the bilateral femur fractures score the same as a unilateral fracture. The
New Injury Severity Score (NISS) was developed because of this shortcoming of the ISS.
The ISS is used in studies to characterize patient injury severity, with a value of 18 or
above indicating polytrauma in many studies. PREFERRED RESPONSE: 3
50
50
2012 Musculoskeletal Trauma Self-Assessment Examination by Dr.Dhahirortho
Question 94-is the initial lateral radiograph of the foot of a 55-year-old woman who
felt a pop in her left foot as she stepped off the curb. She subsequently had severe
heel pain and could not bear weight.Examination in the emergency department
revealed a bony prominence over the posterior aspect of the heel with blanching of
the surrounding skin. What is the most appropriate orthopaedic management?

1. Immediate cast immobilization with maximum plantar flexion


2. Immediate surgical treatment with percutaneous reduction and screw fixation
3. Immediate open reduction and internal fixation via an extensile lateral approach
4. Short leg splint, elevation, and delayed open reduction and internal fixation
5. Short leg splint, elevation, and conversion to cast immobilization when soft-tissue
swelling has resolved

DISCUSSION: The patient has a calcaneal tuberosity fracture, similar to the tongue-type
fracture except the fracture line exits posterior to the posterior facet. The Achilles tendon is
attached to the displaced fragment and pulls the fragment proximally. These are relatively
uncommon fractures, but have the same (or greater) potential as tongue-type fractures for
soft-tissue compromise and necrosis.Immediate management with reduction and fixation is
indicated to prevent heel ulceration and secondary complications such as deep infection.
Fracture fixation generally does not require an extensile approach or plate fixation, and
may benefit from decreasing the forces acting on the displaced fragment by supplemental
gastrocnemius recession. PREFERRED RESPONSE: 2

Question 95-A 24-year-old man is involved in a motor vehicle accident at 60 mph. He


sustains multiple injuries including an intra-abdominal injury requiring a
splenectomy and a closed right femoral shaft fracture.Which variable will best
indicate the patient’s resuscitation status when deciding whether to proceed with
definitive care of the fracture at the conclusion of the laparotomy?

1. Heart rate
2. Hematocrit
3. Base deficit
4. Urine output
5. Systolic blood pressure

DISCUSSION: A metabolic parameter such as the base deficit or lactate level has been
shown to better reflect the resuscitation status and survival after trauma. Normalization of
hemodynamic parameters does not accurately reflect the resuscitation status and a patient
51
51
2012 Musculoskeletal Trauma Self-Assessment Examination by Dr.Dhahirortho
can be in compensated shock (occult tissue hypoperfusion) despite normalization of the
heart rate and blood pressure. The use of temporizing measures with delayed definitive
fracture treatment has been shown to decrease systemic complications in these patients
with occult hypoperfusion. PREFERRED RESPONSE: 3

Question 96-A 27-year-old woman who was an unrestrained driver in a head-on


collision sustained the following injuries: bilateral supracondylar femur fractures, a
left talus fracture, multiple left metatarsal fractures, a right distal radius fracture,
and a left open elbow fracture-dislocation. Which of the following serologic
inflammatory markers drawn in this patient has been shown to be a reliable measure
of systemic inflammatory response, correlating with injury severity and outcome?
1. IL-6
2. IL-8
3. IL-10
4. C-reactive protein
5. Tumor necrosis factor-alpha

DISCUSSION: In response to trauma, the body demonstrates a systemic inflammatory


response that varies in intensity according to the severity of the injuries sustained.
Increased production of proinflammatory cytokines serves to activate the host immune
system. The activation of systemic inflammatory response may lead to remote end organ
damage with neutrophil demargination and disruption of the vascular endothelium.
Disturbances in microcirculation exacerbate local tissue hypoxia, and parenchymal
necrosis may ensue. Furthermore, the systemic inflammatory response serves as the
basis for the development of adult respiratory response distress syndrome (ARDS) and
multiple organ failure following trauma.Several serologic inflammatory markers have been
investigated for their potential usefulness in measuring and monitoring the inflammatory
response to a major trauma. These markers include IL-1, IL-6, IL-8,-IL-10, and C-reactive
protein and tumor necrosis factor-alpha. For many of these markers, serum concentration
is noted to increase following severe trauma, but the extent and duration of elevation have
been too variable to allow for clinical application. IL-6 has been shown to be a reliable
measure of systemic inflammatory response, correlating with injury severity and outcome.
It has been recommended that measurement of IL-6 concentration be traced to evaluate
the severity of the inflammatory response.In this way, it may be possible to clarify the risks
associated with secondary procedures such as fracture fixation and to determine when
these procedures should be performed. PREFERRED RESPONSE: 1

Question 97- are the radiographs of a 27-year-old man involved in a motorcycle


crash who sustained a right proximal humerus fracture. Which of the following is
most associated with osteonecrosis?

52
52
2012 Musculoskeletal Trauma Self-Assessment Examination by Dr.Dhahirortho
1. Intact medial hinge
2. Metaphyseal head extension of fracture of less than 8 mm
3. Tobacco use
4. Age
5. Angular displacement of the head of 30 degrees

DISCUSSION: Hertel and associates demonstrated factors that are predictive of


osteonecrosis of the humeral head after fracture. Factors shown to be predictive of
osteonecrosis include: fractures consisting of four fragments, angular displacement of the
head (greater than 45 degrees), the amount of displacement of the tuberosities
(displacement of greater than 10 mm), glenohumeral dislocation, and head-split
components. Factors associated with good prognosis include: length of the metaphyseal
head extension(calcar segments of greater than 8 mm), the integrity of the medial hinge,
and the basic fracture pattern.When the above criteria (anatomic neck, short calcar,
disrupted hinge) were combined, positive predictive values of up to 97% could be obtained
for osteonecrosis. However, the degree to which this osteonecrosis impacts long-term
outcome is unclear and should not be the only indication for proximal humeral arthroplasty.
PREFERRED RESPONSE: 2

Question 98-Clinical staging of osteomyelitis using the Cierney-Mader classification


system takes into account which of the following factors?

1. Age and gender of patient


2. Fracture type and type of bacteria
3. Host status and extent of infected bone
4. Immune status and chronicity of infection
5. Bacterial resistance and source of infection

DISCUSSION: The Cierney-Mader classification system takes into account three types of
patients with osteomyelitis: (A) healthy, (B) those with comorbidities, and (C) a host in
whom treatment will lead to greater morbidity than the infection. Furthermore, the disease
is addressed based on its complexity: type I-medullary, type II-superficial, type III-localized,
and type IV-diffuse. PREFERRED RESPONSE: 3

Question 99-are the radiographs of a 76-year-old woman who sustained an injury to


her dominant arm in a fall. Which of the following is the most common complication
seen following treatment with a locked plate and screw construct?

1. Osteomyelitis
2. Osteonecrosis
3. Posttraumatic osteoarthritis
4. Deltoid heterotopic ossification
5. Screw penetration of the
articular surface

53
53
2012 Musculoskeletal Trauma Self-Assessment Examination by Dr.Dhahirortho
DISCUSSION: The most common complication reported following use of a locked plate
construct for a displaced proximal humerus fracture is screw penetration of the humeral
head (16% to 30%). Heterotopic ossification can be seen following proximal humerus
fracture and repair and is associated with a deltoid split approach. The rate of
osteonecrosis following a valgus impacted three-part fracture is 5% to 10%.Posttraumatic
osteoarthritis is not seen frequently following surgical repair of these fractures. Infection is
uncommon after this surgery, and chronic osteomyelitis is rare. PREFERRED RESPO: 5

Question 100-A 63-year-old woman with osteopenia is struck by a motor vehicle and
sustains a Schatzker 2 (AO/OTA Type B) fracture of the lateral tibial plateau. She has
1.5 cm of joint depression and 7 mm of condylar widening. What is the most
appropriate surgical fixation for this injury?

1. Lateral nonlocking plate


2. Percutaneous screws
3. External fixation
4. Lateral locking plate
5. Medial and lateral plating

DISCUSSION: The patient has a significantly displaced partial articular fracture of the tibial
plateau.Surgical treatment is preferred in an effort to restore the axis of the knee, achieve
an articular reduction,and allow for repair of commonly associated soft-tissue injuries such
as meniscal tears. This requires direct reduction, and fixation should provide subarticular
support, interfragmentary compression, and buttress. This is best achieved with an
undercontoured lateral nonlocking plate. Screws alone are unlikely to be adequately stable
in this patient. Locking plates do not provide buttress effect when used in pure locking
mode. In addition, locking plates add significant incremental cost to the procedure.
PREFERRED RESPONSE: 1

RESPONSES FOR QUESTIONS 101 THROUGH 104


1. Open reduction and internal fixation with absolute stability
2. Open reduction and internal fixation with relative stability
3. Closed reduction with casting
4. Reamed locked intramedullary nailing
5. External fixation
For each of the following fractures choose the best method for definitive fixation.

Question 101-A 25-year-old man fell off a golf cart and injured his right forearm. The
radiographs show both a radial shaft and an ulna fracture. The fractures are highly
comminuted, and there is a small laceration of about 1cm over the fracture site.

Question 102-A 45-year-old man was skiing when he sustained a direct blow to the
lateral side of his left knee.Radiographs reveal a left split depression tibial plateau
fracture.

Question 103-A 19-year-old woman was involved in a high-speed motor vehicle


accident. She has an obvious deformity of her right thigh. Radiographs reveal a
transverse mid-shaft femur fracture.

Question 104-A 22-year-old woman injured her ankle when she fell off a ladder.
Radiographs reveal a displaced large posterior malleolus fracture of about 45% of
the joint.

54
54
2012 Musculoskeletal Trauma Self-Assessment Examination by Dr.Dhahirortho
PREFERRED RESPONSE: 2
PREFERRED RESPONSE: 1
PREFERRED RESPONSE: 4
PREFERRED RESPONSE: 1

DISCUSSION FOR QUESTIONS 101 THROUGH 104:


Basic understanding of fracture care requires a fundamental knowledge of the principles
regarding absolute and relative stability. Compression plating and anatomic reduction of
articular fractures are examples of absolute stability. Bridge plating, external fixation,
casting, and intramedullary nailing are all examples of relative stability. Both bone forearm
fractures have long been treated with open reduction and internal fixation even in the light
of open wounds. Results have been excellent with plate fixation.Recently, intramedullary
nails that are contoured and locked have been used in the treatment of both bone forearm
fractures, but they are not reamed. It is well established that with restoring the proper
radial bow,length, and alignment, optimal function can be achieved. Open reduction and
internal fixation allows this achievement. In cases where comminution exists, absolute
stability may have to be sacrificed so as to not strip small comminuted bone fragments.
Therefore, a bridging technique is worthwhile. External fixation can be used as a
temporary technique until the soft tissues are more amenable to definitive fixation. Cast
treatment is not indicated in adult forearm fractures. Locking nails for forearm use are not
reamed. With regards to articular fractures, anatomic reduction and rigid stabilization are
required to achieve the best results and allow for fracture healing. This environment also
allows for the best chance of the cartilage repair process to form “hyaline-like” cartilage.
Open reduction and internal fixation with absolute stability is the mainstay of treatment for
partial articular fractures such as split depression tibial plateau fractures and posterior
malleolus fractures involving greater than about 25% to 30% of the joint. The gold standard
for the treatment of a closed femur fracture is a reamed intramedullary locked nail. Results
are uniformly excellent. This can be done without stripping of the soft tissues such as in
open reduction and internal fixation. External fixation can be used as a temporary device in
patients in extremis for damage control reasons.

CLINICAL SITUATION FOR QUESTIONS 105 THROUGH 107


Figure 105 is the radiograph of a 72-year-old woman who
fell down the steps and injured her left leg.Because she
was unable to get up on her own or reach a telephone, she
was not discovered until 24hours after the incident and was
then brought to the emergency department by ambulance.
The patient is a community ambulator, who drives and lives
alone. Her medical history includes hypertension, type 2
diabetes mellitus, and hypothyroidism. She takes enalapril,
levothyroxine, and rosiglitazone. She is obese (BMI>30).
Examination reveals that the left lower extremity is
shortened and externally rotated.She has pain with
movement of the extremity. There is no other bony
tenderness in either the upper or contralateral lower
extremity.
Question 105-Which of the following imaging studies must be obtained for this
patient?
1. Duplex scan of both lower extremities
2. MRI scan of the hip
3. Traction internal rotation radiograph of the hip
4. Frog lateral of the hip
5. CT scan of the abdomen and pelvis
55
55
2012 Musculoskeletal Trauma Self-Assessment Examination by Dr.Dhahirortho
PREFERRED RESPONSE: 1
Question 106-What is the most appropriate protocol for surgical management of this
patient?
1. Must be performed within the first 24 hours
2. Should be delayed until a stress test is performed
3. May occur only following optimization of medical conditions
4. Must be postponed until normalization of the BMI
5. A delay of several days after admission results in greater technical ease

PREFERRED RESPONSE: 3
Question 107-Treatment of the patient’s injury is best accomplished by which of the
following?

1. Cephalomedullary nail
2. Closed reduction and percutaneous pinning
3. Open reduction and internal fixation
4. Total hip arthroplasty
5. Hemiarthroplasty
PREFERRED RESPONSE: 4
DISCUSSION FOR QUESTIONS 105 THROUGH 107:
This scenario is a common one for the presentation of a displaced intracapsular hip
fracture. Injured elderly patients not found for an extended period of time prior to hospital
admission are at greater risk for the development of a venous thromboembolism and
should have a screening Doppler. Whereas a tractioninternal rotation radiograph is
valuable in further delineating the fracture pattern, it is not necessary in making the
diagnosis. The frog lateral radiograph should be avoided in hip fracture management
because it causes unneeded pain and risks displacement if none exists. An MRI scan is
not indicated in a displaced fracture, but may be useful in diagnosing nondisplaced
fractures. A CT scan is not warranted in the absence of clinical signs of intra-abdominal
trauma or a high-energy mechanism. Surgical treatment is warranted in this ambulatory,
independent patient. Whereas many studies have looked at optimal timing of surgery, no
definitive time frame has been elucidated. Most authors agree that surgery is urgent and
should follow optimization of all comorbid medical conditions. Technical aspects of the
surgery are not dependent on time from injury. Given that this is a displaced femoral neck
fracture in an elderly patient,the options of fracture repair, which include intramedullary
nailing, closed reduction and pinning, and open reduction and internal fixation, are
associated with high rates of complications and revision surgery.
Of the two arthroplasty choices, total hip arthroplasty has been shown to yield the best
clinical and functional results with a lower rate of reoperation. Hemiarthroplasty is reserved
for low functioning or minimally ambulatory patients.

56
56
2012 Musculoskeletal Trauma Self-Assessment Examination by Dr.Dhahirortho
CLINICAL SITUATION FOR QUESTIONS 108 THROUGH 110
A 23-year-old man was injured in a high-speed motorcycle accident. He sustained bilateral
pulmonary contusions, a closed left femoral fracture, an open, comminuted, contaminated,
diaphyseal left tibia fracture without bone loss, and a small subdural bleed. His blood
pressure in the trauma bay was 85/50 mm Hg and did not respond to initial volume
resuscitation. He has a heart rate of 122/min and a core temperature of 34.7°C.

Question 108-What is the optimal initial treatment for his orthopaedic injuries?
1. Irrigation and débridement of the open fracture and reamed intramedullary nailing of the
femoral and tibial fractures
2. Irrigation and débridement of the open fracture, reamed intramedullary nailing of the
femur,and external fixation of the tibia
3. Irrigation and débridement of the open fracture and external fixation of both fractures
4. Irrigation and débridement of the open fracture, a reamed femoral nail, and an
unreamed tibial nail
5. Irrigation and débridement of the open fracture in the ICU and a calcaneal traction pin

PREFERRED RESPONSE: 3
Question 109-Definitive fixation of the tibia is ideally
1. uniplanar external fixation.
2. a reamed intramedullary nail.
3. an unreamed intramedullary nail.
4. either a reamed or an unreamed intramedullary nail.
5. bridge plating.

PREFERRED RESPONSE: 4
Question 110-Four months after injury, the tibia is showing evidence of slow healing
on radiographs. What is the optimal treatment for this potential nonunion?
1. Convert to circular external fixation
2. Exchange nailing and autograft
3. Exchange nailing and bone morphogenetic protein
4. Tibial plate
5. Observation until 6 months after injury

PREFERRED RESPONSE: 5

DISCUSSION FOR QUESTIONS 108 THROUGH 110:


This patient is unstable and is not a good candidate for Early Total Care (ETC) and
therefore should be managed by the tenets of Damage Control Orthopaedics (DCO).
Débridement and external fixation is preferable for this patient. Intramedullary nails would
be a component of ETC. Calcaneal traction is not considered ideal because it does not
allow the patient to travel as easily. The S.P.R.I.N.T. study concluded that while reamed
nails may offer benefit in closed fractures, there was no difference between reamed or
unreamed nails in the treatment of open fractures of the tibia. Uniplanar external fixation
and tibial plating are not considered the best options for open tibia fractures. Additional
findings of the S.P.R.I.N.T. study conclude that delaying surgical intervention for at least 6
months after injury may reduce the need for reoperation.

57
57
2012- -Foot and Ankle --Self-Assessment Examination by Dr.Dhahirortho

58
1
2012- -Foot and Ankle --Self-Assessment Examination by Dr.Dhahirortho

59
2
2012- -Foot and Ankle --Self-Assessment Examination by Dr.Dhahirortho
Question 1Figure 1 is the radiograph of a 48-year-old man. He is of normal height and
weight, medically healthy,and in good physical condition. What is the best treatment
option?
1. Short-leg non-weight-bearing cast in plantar
flexion
2. Excision of the fragment and reattachment of
the Achilles tendon into the calcaneus
3. Immediate open reduction and internal
fixation
4. Open reduction and fixation when swelling
reduces
5. Percutaneous reduction and Kirschner wire
fixation

DISCUSSION: Immediate open reduction and internal fixation of this fracture is required
to prevent necrosis of the overlying soft tissue. Because of the power and proximal pull of
the triceps surae,nonsurgical management is not indicated with avulsion fractures of the
calcaneus. It leaves a large void that will not fill in with bone, leaves the Achilles tendon
weak, and has a high complication rate, especially skin breakdown. The Achilles tendon is
securely attached to the fractured tuberosity. Bone-to-bone healing is more reliable than
detaching the Achilles tendon from the tuberosity and reattaching it to the remainder of
the calcaneus. Because of the size of the avulsed fragment, it will be difficult to correctly
tension the tendon if the fractured piece is excised. Percutaneous Kirschner wire fixation
is not strong enough to provide a stable fixation of the tuberosity, especially in view of
the power of the Achilles tendon contracture.
PREFERRED RESPONSE: 3

Question 2A 45-year-old man with a valgus recurvatum malunion of the distal tibia
undergoes a multiplanar osteotomy. A stable 2 x 2 cm eschar develops perioperatively
over the anterior tibia just distal to the osteotomy. Eight weeks after surgery, the
patient reports the insidious development of increasing pain, erythema, and
intermittent drainage from the eschar area; he denies fevers or other constitutional
signs. Current radiographs are shown in Figures 2a and 2b. Surgical débridement
reveals no gross purulence but intraoperative soft-tissue cultures yield methicillin-
resistant Staphylococcus aureus. The surgical implant was visible deep within the
wound. What is the most appropriate method of treatment?

60
3
2012- -Foot and Ankle --Self-Assessment Examination by Dr.Dhahirortho

1. Tibial saucerization and conversion to external


fixation
2. Repeat surgical débridement, intravenous
antiobiotics, and implant removal
3. Intravenous antibiotics and hyperbaric therapy
4. Intravenous antibiotics and a negative pressure
dressing
5. Wound coverage, antibiotic suppression, and
implant removal after bone healing

DISCUSSION: The patient should be managed with


wound coverage and antibiotic suppression with
implant retention until the osteotomy has definitively
healed, followed by implant removal after bone
healing. The patient has developed a deep infection in an area with a limited soft-tissue
envelope, and the implant is assumed to be colonized since it is visible within the wound.
Because the implants are stable, there is no need to convert to external fixation. The
osteotomy is not likely healed at 8 weeks postoperatively, which precludes implant
removal. Intravenous antibiotics and hyperbaric therapy alone will not be sufficient
treatment. The wound location over the anterior tibia precludes use of a negative
pressure dressing, particularly with an exposed indwelling implant. PRE RES: 5

Question 3 Figures 3a and 3b are the current AP and oblique radiographs of a 44-year-
old man who underwent nonsurgical management of a left ankle fracture 6 months
ago. What is the most appropriate course of management?

1. Arizona brace
2. Ankle arthroscopy with drilling of the talar
osteochondral lesion
3. Medial ankle arthrotomy and débridement
with correction of the fibular malunion
4. Ankle arthrodesis
5. Ankle and subtalar arthrodesis

DISCUSSION: The radiographs reveal a fractured


malunited, shortened fibula with deltoid
61
4
2012- -Foot and Ankle --Self-Assessment Examination by Dr.Dhahirortho
instability.Corrective osteotomy with fibular lengthening has shown positive results.
Nonsurgical management in an active, healthy patient will lead to rapid deterioration of
the ankle joint. Without evidence of arthritis, a joint-sacrificing procedure should not be
used. PREFERRED RESPONSE: 3

Question 4 Figures 4a and 4b are the radiographs of an isolated injury. What is the next
most appropriate step in management?
1. CT
2. MRI
3. Closed reduction and casting
4. Application of a spanning external fixator
5. Immediate open reduction and internal
fixation (ORIF)

DISCUSSION: Successful treatment of a pilon


fractures requires a complete understanding
of the fracture configuration. This information is not available using radiographs alone;
therefore, CT is used to define the fracture anatomy but only after stabilization and
distraction of the fracture via external fixation. MRI does not adequately show the detail
of the bone fragments. Immediate ORIF is contraindicated because of the high rate of
soft-tissue complications with this treatment regimen, whereas closed treatment has a
high rate of poor outcomes because of arthritis. Delayed ORIF is the recommended
treatment, but this occurs after temporary stabilization and CT scanning. PRE RESPO: 4

Question 5 Figure 5 shows the deformity that developed in a 49-year-old woman who
had previously undergone a bunion correction. The patient’s great toe is easily
corrected to a neutral position but tends to spring back to a varus position. She reports
pain in the first metatarsophalangeal joint and has difficulty wearing most shoes. What
is the most appropriate management plan?

1. 1-2 toe taping and closed toe shoes


2. Split extensor hallucis longus tendon transfer
3. Great toe fusion
4. Medial soft-tissue release and lateral capsule
plication
5. Metatarsal osteotomy, medial capsule release, and
split extensor hallucis longus tendon transfer
62
5
2012- -Foot and Ankle --Self-Assessment Examination by Dr.Dhahirortho
DISCUSSION: Osteotomy and tendon transfer is the management of choice. The previous
bunion correction resulted in excessive translation of the metatarsal head. The
orthopaedic surgeon must first correct the bony deformity and allow the proximal
phalanx to sit in a congruent position. The next step is to reconstruct the soft-tissue
components and this can be done by releasing the medial capsule,and transferring part of
the extensor hallucis longus tendon into the proximal phalanx, under the intermetatarsal
ligament laterally. All three procedures are needed to adequately correct this deformity.
A great toe fusion is indicated for an uncorrectable deformity or in an older patient.

PREFERRED RESPONSE: 5

Question 6 A 28-year-old man has a progressive drop-foot deformity secondary to


Charcot-Marie-Tooth disease.Examination reveals no tibialis anterior or peroneus
brevis function. He has a 5-degree equinis contracture. Tibialis posterior and flexor
digitorum longus are 5/5 strength. There are no fixed deformities of any joints. What is
the most appropriate surgical option?

1. A gastrocnemius lengthening and transfer of the tibialis posterior tendon to the


dorsum of the foot
2. A gastrocnemius lengthening and transfer of the peroneus brevis to the dorsum of
the foot
3. Transfer of the flexor digitorum longus to the dorsum of the foot
4. An ankle fusion and transfer of the tibialis posterior to the dorsum of the foot
5. A triple arthrodesis to stabilize the drop-foot deformity

DISCUSSION: At this point, the deformities are supple and fusions are not indicated. The
tibialis posterior is the force couple or antagonist of peroneus brevis. With no peroneus
brevis, the tibialis posterior is not only a deforming force, pulling the foot into inversion,
but it is also the strongest muscle to use as an ankle dorsiflexor. By transferring it, the
deforming force is removed and converted into an ankle dorsiflexor. With the equinis
contracture, the gastrocnemius should be lengthened to allow the transferred tendon to
dorsiflex the ankle beyond neutral.

PREFERRED RESPONSE: 1

63
6
2012- -Foot and Ankle --Self-Assessment Examination by Dr.Dhahirortho
Question 7 Figures 7a and 7b are the weight-bearing radiographs of a 17-year-old girl
who has great toe pain with push-off and stiffness 1 year after undergoing a proximal
crescentic osteotomy for hallux valgus. Motion at the first metatarsophalangeal joint
includes approximately 20° of dorsiflexion. What is the most appropriate treatment?

1. Proximal phalanx osteotomy


2. Double metatarsal osteotomy
3. Plantar flexion metatarsal osteotomy
4. Distal biplanar metatarsal osteotomy
5. Capsular release and aggressive physical therapy

DISCUSSION: The patient has progressed to a dorsiflexion malunion of the first


metatarsal. The absence of implants suggests that smooth pin fixation was likely used,
likely contributing to the malunion. The patient should be managed with a plantar flexion
metatarsal osteotomy. The joint stiffness is likely a result of the malunion acting as a
mechanical block to dorsiflexion, thus a capsular release would not be sufficient. The
distal biplanar metatarsal osteotomy and double metatarsal osteotomy are used for
hallux valgus deformities associated with an increased distal metatarsal articular angle.
The proximal phalanx osteotomy is used for supplemental correction of associated hallux
valgus interphalangeus. PREFERRED RESPONSE: 3

Question 8 Figures 8a and 8b are the preoperative radiographs of a 47-year-old woman


who is being treated for a supple pes plano abductovalgus deformity. She is unable to
perform an ipsilateral single leg heel raise. Which of the following is the most likely
soft-tissue procedure performed in combination with the bony surgery?

1. Flexor digitorum longus tendon transfer


2. Extensor hallucis longus tendon transfer
3. Spring ligament release
4. Peroneus longus tendon repair
5. Lateral collateral ligament reconstruction
64
7
2012- -Foot and Ankle --Self-Assessment Examination by Dr.Dhahirortho
DISCUSSION: Flexor digitorum longus tendon transfer and augmentation has similar
dynamic function to the posterior tibial tendon. Other reported tendon transfers for this
procedure include flexor halluces longus and peroneus brevis, but not extensor hallucis
longus nor peroneus longus. Spring ligament release accentuates the flatfoot deformity
whereas conversely, spring ligament repair/reconstruction is another recognized soft-
tissue procedure that may be combined with bony surgery for treatment of flexible
acquired flatfoot deformity. Lateral collateral ligament reconstruction addresses lateral
ankle instability which the patient does not have. PREFERRED RESPONSE: 1

Question 9 Figures 9a and 9b are the radiographs of a 32-year-old woman who has right
foot pain after falling down a few steps. For the best long-term outcome, initial
treatment should include which of the following?

1. Splinting with non-weight-bearing as the definitive


treatment
2. Walking boot
3. Closed reduction and casting
4. Percutaneous pinning
5. Primary open reduction and internal fixation (ORIF)

DISCUSSION: The radiographs show a displaced Lisfranc injury. The outcome of treatment
is dependent on achieving an anatomic reduction and stabilization, which is only possible
with primary ORIF. Some studies indicate primary fusion may provide superior short-term
results compared with ORIF. Closed treatment (reduction with casting or splinting) will
not achieve or maintain the reduction, whereas delayed treatment by secondary fusion
after arthritis occurs yields inferior outcomes to primary ORIF.

PREFERRED RESPONSE: 5

65
8
2012- -Foot and Ankle --Self-Assessment Examination by Dr.Dhahirortho
Question 10 Figure 10 is the radiograph of a middle-aged woman who has had midfoot
pain for the past several years without antecedent trauma. What is the most likely
etiology of her condition?
1. Osteomyelitis
2. Kohler disease
3. Rheumatoid arthritis
4. Primary osteoarthritis
5. Osteochondritis dissecans

DISCUSSION: The radiograph shows isolated degeneration in the talonavicular joint that is
symmetric.The symmetry of the degeneration is characteristic of an inflammatory
arthritis. In the absence of trauma,isolated arthritis in this joint is uncommon. The
navicular is normal sized, ruling out Kohler disease(as well as the patient being in the
wrong age group). There are no erosions indicative of osteomyelitis.Osteochondritis
dissecans appears as focal osteochondral lesions, which are not present in the
radiograph. PREFERRED RESPONSE: 3

Question 11Figure 11a is the radiograph of a 45-year-old woman with a moderate


bunion deformity. A Chevron osteotomy was performed and after 6 weeks the patient
was doing reasonably well. Six months later she reports increasing pain and stiffness in
her toe. Clinically the toe is reasonably straight, but she has significant calluses and
overload under the second and third metatarsals. A follow-up radiograph is shown in
Figure 11b. The patient wants to be free of pain. What is the most appropriate
treatment?

1. Revision bunion repair with a Lapidus


procedure
2. Keller’s excision arthroplasty
3. Moberg osteotomy of the proximal
phalanx
4. Metatarsophalangeal joint fusion with
an autologous bone block
5. Allograft replacement of the metatarsal
head

66
9
2012- -Foot and Ankle --Self-Assessment Examination by Dr.Dhahirortho
DISCUSSION: The patient developed osteonecrosis of the metatarsal head with
shortening of the first metatarsal and subsequent overload of the lesser metatarsals. The
most reliable option is to perform a metatarsophalangeal joint fusion with an autologous
bone block to restore length. A revision bunion repair will not address the arthritic
changes. A Keller’s excision arthroplasty will further aggravate the lesser metatarsal
overload. An allograft replacement of the metatarsal head has a very low predictability
rating and is highly experimental. PREFERRED RESPONSE: 4

Question 12 Figure 12 is the radiograph of a patient with type 2 diabetes, a body mass
index of 42, and an Hgb A1c of 8. What is the most appropriate management for this
injury?
1. Fracture boot immobilization
2. Casting the ankle in its current position
3. Closed reduction and definitive casting
4. Closed reduction and application of external fixation
5. Open reduction and internal fixation (ORIF)

DISCUSSION: Several recent studies have shown that while there is an increased risk of
complications following ORIF of displaced ankle fractures in diabetic patients compared
with nondiabetic patients,the overall risks of treatment are less than that associated with
nonsurgical treatment in diabetics. There is also the possibility that ORIF of unstable
ankle fractures may forestall the development of Charcot changes in the ankle, although
this is not definitively known. Extra rigid fixation may be required because of the patient’s
size and poorly controlled diabetes. Nonsurgical management is associated with poorer
functional outcomes (due to arthritis secondary to poor reduction of the fracture) and a
higher rate of skin breakdown, due to the need for higher skin pressures from the use of
highly molded casting used to maintain a closed reduction. PREFERRED RESPONSE: 5

Question 13A 28-year-old construction worker with a body mass index (BMI) of 31
sustained a Weber C fracture 3 years ago. An open reduction and internal fixation was
performed, but he developed degenerative changes in the ankle as seen in Figure 13.
Management consisting of bracing, shoe modifications, and other modalities has failed
to provide relief. He is symptomatic enough that he wants definitive treatment. What is
the best treatment option at this time?

67
10
2012- -Foot and Ankle --Self-Assessment Examination by Dr.Dhahirortho

1. Bipolar allograft replacement of the tibial plafond and


talar dome
2. Low profile total ankle arthroplasty
3. Interpositional soft-tissue replacement
4. Arthroscopic ankle débridement
5. Ankle fusion

DISCUSSION: Ankle fusion will provide the most reliable


pain relief and function for this young manual laboror. At
his young age and with a BMI of 31, both total joint
arthroplasty and allograft replacement are controversial. An interpositional graft could be
an option, but there is not enough evidence in the literature to recommend it at this
time. The radiographs show degenerative changes that are too far advanced for an
arthroscopic ankle débridement to be of any benefit. PREFERRED RESPONSE: 5

Question 14 Figures 14a through 14c are the MRI scans of a 37-year-old woman who
sustained a traumatic laceration to the anterior aspect of the ankle. The wound was
closed in the emergency department. On examination,she has a foot drop and
ambulates with a steppage gait. With successful surgical repair, what is the most
common long-term residual?

1. Numbness in the foot


2. Persistent foot drop
3. Persistent ankle pain
4. Decreased dorsiflexion strength
5. Use of an ankle-foot orthosis for ambulation

DISCUSSION: Anterior tendon disruption has been described in association with direct
trauma, gout,inflammatory arthritis, local steroid injections, and diabetes. When a
68
11
2012- -Foot and Ankle --Self-Assessment Examination by Dr.Dhahirortho
rupture is accurately diagnosed in younger, healthy, active patients, surgical repair has
been recommended. Surgical repair results in improved patient satisfaction; however,
isokinetic testing has shown decreased dorsiflexion and inversion strength compared
with the uninvolved side. Numbness can result from missed superficial nerve laceration.
Persistent foot drop and use of an ankle-foot orthosis are more frequently seen in chronic
missed injuries or with nonsurgical management. PREFERRED RESPONSE: 4

Question 15 Figures 15a and 15b are the radiographs of an active 65-year-old woman
who has a 3-year history of increasing foot pain and flattening of the left foot. Inversion
strength is 5+ and does not reproduce her symptoms. Bracing and nonsteroidal anti-
inflammatory drugs have failed to provide adequate relief. She has a supple hindfoot
and normal heel cord flexibility. What is the most appropriate treatment?

1. Lateral column lengthening and flexor digitorum longus transfer


2. Talonavicular arthrodesis
3. Medial Lisfranc arthrodesis
4. Subtalar arthrodesis
5. Triple arthrodesis
DISCUSSION: The patient has degenerative changes with collapse at the tarsometatarsal
joints; therefore,a fusion of the Lisfranc joint is indicated. The other responses are
hindfoot procedures that will not address the tarsometatarsal joint. PREFER RESPONSE: 3
Question 16 Figure 16 is the lateral radiograph of a 40-year-old laborer who sustained a
displaced intra-articular calcaneus fracture that was treated nonsurgically 1 year ago.
He now reports pain with ankle dorsiflexion,as well as subfibular impingement. What is
the most appropriate surgical treatment?
1. Lateral wall ostectomy
2. In situ subtalar arthrodesis
3. Triple arthrodesis
4. Distraction subtalar arthrodesis with lateral wall
ostectomy
5. Ankle and subtalar arthodesis
69
12
2012- -Foot and Ankle --Self-Assessment Examination by Dr.Dhahirortho
DISCUSSION: The patient has a calcaneal fracture malunion, with symptomatic subtalar
arthritis and anterior ankle and lateral subfibular impingement. Distraction subtalar
arthrodesis addresses subtalar arthritis and anterior impingement and lateral wall
ostectomy relieves symptoms of lateral impingement.The other procedures do not
address all facets of the patient’s symptoms.

PREFERRED RESPONSE: 4

Question 17 Figures 17a and 17b are the radiographs of a 56-year-old man who has foot
pain, swelling, and a limp after being injured in a motorcycle accident 2 months ago.
The patient is a noninsulin-dependent diabetic,has palpable pulses, and has protective
sensibility of his feet. What is the most appropriate management?

1. Cast immobilization and non-weight-bearing for 6


weeks
2. Closed reduction and percutaneous screw fixation
3. Open reduction and internal fixation
4. Lisfranc arthrodesis of the medial and lateral column
5. Lisfranc arthrodesis of the medial column and
Kirschner wire fixation of the lateral column

DISCUSSION: Whereas most fracture-dislocations of the Lisfranc joint are best treated
with formal open reduction and internal fixation, primary fusion may be indicated in
those fractures with severe intraarticular comminution, complete ligamentous
dislocations, or delayed treatment. Fusion of the lateral column is not necessary because
leaving it unfused has not been shown to adversely affect results. Closed reduction and
percutaneous pinning does not allow for accurate reduction of the joint in a displaced
fracture-dislocation. Cast immobilization would be appropriate for a diabetic who is
insensate or someone whose medical conditions preclude surgery.

PREFERRED RESP: 5

70
13
2012- -Foot and Ankle --Self-Assessment Examination by Dr.Dhahirortho
Question 18Figures 18a and 18b are the radiographs of an obese 75-year-old man with
a rigid acquired flatfoot deformity. What is the best treatment option?

1. Double calcaneal osteotomy


2. Medial calcaneal osteotomy and tendon
transfer
3. Lateral column lengthening and tendon
transfer
4. Subtalar arthrodesis
5. Triple arthrodesis

DISCUSSION: For stage III adult-acquired flatfoot deformity characterized by dysfunction


of the posterior tibial tendon, rigid valgus deformity of the hindfoot, and arthritic changes
of the hindfoot joints,arthrodesis is the favored procedure. In an overweight patient with
degenerative changes affecting the subtalar and Chopart joints, triple arthrodesis is the
best treatment option. Subtalar arthrodesis only addresses the talocalcaneal joint and
continues to render the patient symptomatic in the talonavicular and calcaneocuboid
joints. Advanced stage III disease precludes reconstructive procedures involving calcaneal
osteotomy and tendon transfer. PREFERRED RESPONSE: 5

Question 19 Which of the following occurs frequently after nonsurgical management of


displaced intra-articular fractures of the calcaneus?
1. Return to normal function
2. Narrowing of the calcaneus
3. Lengthening of the calcaneus
4. Plantar flexion of the talus
5. Peroneal tendinitis

DISCUSSION: Peroneal tendinitis and stenosis are typically seen following nonsurgical
management and results from lateral subfibular impingement, whereby the displaced,
expanded lateral wall subluxates the peroneal tendons against the distal tip of the fibula
or might even dislocate the tendons. Nonsurgical management of displaced calcaneal
fractures offers little chance for return to normal function because of the development of
a calcaneal malunion. The articular surface is not reduced, the heel remains shortened

71
14
2012- -Foot and Ankle --Self-Assessment Examination by Dr.Dhahirortho
and widened, the talus is dorsiflexed in the ankle mortise, and the displaced lateral wall
causes impingement and binding of the peroneal tendons. PREFERRED RESPONSE: 5

Question 20 What is the most appropriate tendon transfer and augmentation for
surgical treatment of a chronic Achilles tendon rupture?
1. Flexor hallucis longus
2. Extensor digitorum longus
3. Peroneus longus
4. Tibialis anterior
5. Posterior tibialis

DISCUSSION: Flexor hallucis longus tendon transfer provides a biomechanically strong


structure to bridge the defect in surgical treatment of a chronic Achilles tendon rupture.
Although the flexor halluces longus remains the optimal tendon for transfer, others have
described tendon transfer and augmentation with the plantaris, peroneus brevis,
posterior tibial, and flexor digitorum longus. Tibialis anterior and extensor digitorum
longus reside in the anterior compartment of the leg and would be out-of-phase
transfers. PREFERRED RE: 1

Question 21 Which of the following ligaments is commonly attenuated or torn in


patients with an adult-acquired flatfoot deformity?

1. Superomedial calcaneonavicular
2. Dorsal cuneonavicular
3. Dorsal intercuneiform
4. Dorsal calcaneocuboid
5. Anterior tibiofibular

DISCUSSION: The spring ligament (calcaneonavicular) extends from the anterior aspect of
the sustentaculum tali to the plantar medial surface of the navicular, supporting the
plantar medial margin of the talar head. The spring ligament has two components:
superomedial calcaneonavicular ligament and inferior calcaneonavicular ligament, the
former of which is commonly attenuated or torn in patients with an adult-acquired
flatfoot deformity. The distal portion of the deltoid ligament blends into the spring
ligament and is also frequently attenuated in patients with this foot deformity. Dorsal
intertarsal ligaments and the anterior tibiofibular ligament of the ankle are not affected.
PREFERRED RESP: 1
72
15
2012- -Foot and Ankle --Self-Assessment Examination by Dr.Dhahirortho
Question 22 What is the most important measure to take to reduce the risk of frostbite
of the toes while hiking in extreme temperatures?
1. Stop often for recovery breaks.
2. Drink enough warm liquids.
3. Reduce thermal heat loss from shoes.
4. Use triple socks.
5. Adequately "carbo load" before the start.

DISCUSSION: Several studies showed the most reliable method to reduce the risk of cold
exposure injury is to reduce thermal heat loss. This can be done with a combination of
protective socks and shoes, and reducing moisture in the shoes. PREFERRED RESPONSE: 3

Question 23 Which of the following factors has been shown to increase the risk of
peroneal tendon pathology in patients who have undergone posterior plating of lateral
malleolar fractures?
1. Use of cut or trimmed plates
2. Use of straight (uncontoured) plates
3. Use of locked plating
4. Low plate placement with a prominent screw head in the distal hole
5. Low antiglide plate placement

DISCUSSION: Low plate positioning with a prominent screw head in the most distal hole
of the plate was shown to be correlated with peroneal tendon lesions. Distal plate
placement in the absence of prominent screws was not associated with tendon lesions.
Trimmed plates, locked plates, and uncontoured plates have not been shown to increase
the risk of peroneal tendon pathology. PREFERRED RESPONSE: 4
Question 24 A patient who sustained an Achilles tendon rupture does Internet research
on his injury and its treatment before seeing an orthopaedic surgeon. The patient
would like to have surgical repair of the tendon rupture using the technique shown in
Figure 24. What can the surgeon tell the patient regarding the possible benefits of the
use of this pictured technique versus an open technique for the repair of acute Achilles
tendon ruptures?
1. Decreased rates of rerupture
2. Decreased rates of sural neuropathy
3. Statistically significantly greater calf circumference
4. Decreased local scarring
5. Overall the same rate of postoperative complications
73
16
2012- -Foot and Ankle --Self-Assessment Examination by Dr.Dhahirortho
DISCUSSION: Trials comparing the results of open repair of acute Achilles tendon rupture
to repairs done in a limited open fashion show no difference in rerupture rate, sural
neuropathy, or calf circumference.The scarring observed was much less in the group
treated in a limited open fashion. There was a significantly greater number of
postoperative complications seen in the group treated in an open fashion compared with
those treated with a limited open procedure. PREFERRED RESPONSE: 4

Question 25 What is the most common complication with an anterior ankle arthroscopy
using a standard lateral arthroscopy portal?
1. Injury to the superficial peroneal nerve
2. Injury to the tibial nerve
3. Vascular injury to the dorsalis pedis
4. Reflex sympathetic dystrophy
5. Deep infection

DISCUSSION: The most common complication is an injury to the superficial peroneal


nerve at the lateral portal. Infection in ankle arthroscopy happens very infrequently.
Vascular injury with an anterior scope is very rarely reported. Synovial fistulas are also
reported as somewhat common. PREFERRED RESPONSE: 1

Question 26 Figures 26a and 26b are the radiographs and MRI scan of a 15-year-old boy
who reports midfoot pain for the past 4 months despite no history of injury. The
patient plays soccer and is eager to get back to activity.What is the most appropriate
treatment to return the patient back to full activity?

1. Open reduction and internal fixation


2. Restricted weight bearing in a short-leg cast
3. Weight bearing as tolerated in a fracture boot
4. An orthotic and a bone stimulator
5. A carbon fiber orthotic 74
17
2012- -Foot and Ankle --Self-Assessment Examination by Dr.Dhahirortho

DISCUSSION: The patient has a navicular stress reaction as evidenced by the history and
MRI scan findings. This is an acute injury as revealed by the significant edema in the
navicular on the MRI scan.Management should consist of restricted weight bearing in a
short-leg cast, but possibly a boot. With weight bearing, this fracture could displace and
injure the talonavicular joint. A bone stimulator is a good option in conjunction with
immobilization. Surgery is indicated when there is a fracture line that extends across two
cortices, or across one cortice if there is displacement or cystic changes. This reaction has
no fracture line and thus can be treated nonsurgically. PREFERRED RESPONSE: 2

Question 27A 35-year-old woman has a 6-month history of plantar fasciitis. Which of
the following orthoses has been shown to be effective in the treatment of chronic
plantar fasciitis?
1. Full-length accommodative insert
2. Semi-rigid insert with a mild medial arch support and a 5° medial post
3. Night splint
4. Cavus foot orthotic
5. Visco gel heel cushion

DISCUSSION: Of the possible responses, only the night splint has been shown to be
effective in the treatment of chronic plantar fasciitis. The role of inserts in plantar fasciitis
is controversial with limited scientific data. Although a cavus foot orthotic can be of
benefit with respect to plantar fascia symptoms, it is an indirect benefit of
accommodating the plantar flexed first ray and has not been scientifically proven.
PREFERRED RESPONSE: 3

Question 28 Which of the following nerves is most susceptible to iatrogenic injury


during bunion surgery?
1. Terminal branch of the superficial peroneal nerve
2. Terminal branch of the saphenous nerve
3. Dorsolateral cutaneous nerve
4. Medial plantar hallucal nerve
5. Deep peroneal nerve

DISCUSSION: The dorsomedial cutaneous nerve, which is the terminal branch of the
superficial peroneal nerve, is most susceptible to iatrogenic injury, primarily due to the
location of surgical incisions. The dorsolateral cutaneous nerve is typically a branch of the
75
18
2012- -Foot and Ankle --Self-Assessment Examination by Dr.Dhahirortho
deep peroneal nerve; the medial plantar hallucal nerve is a branch of the medial plantar
nerve. The terminal branch of the saphenous nerve provides sensation to the
dorsomedial hindfoot. PREFERRED RESPONSE: 1

Question 29 A patient falls off a roof and sustains the fracture shown in Figure 29. What
is the most likely complication that results from injury to the structure that is located at
the arrow?
1. Paresthesias on the plantar aspect of the foot
2. Pain or popping with great toe flexion
3. Loss of the arch
4. Inability to flex the lesser toes
5. Spring ligament rupture

DISCUSSION: The arrow points to the sustentaculum tali, which


is fractured off the tuberosity of the calcaneus. The flexor
hallucis longus (FHL) tendon runs directly under this structure.
An injury to this structure could cause stenosis around the FHL tendon which would cause
pain with great toe flexion.Paresthesias on the plantar aspect of the foot refers to the
medial plantar nerve. Loss of the arch refers to the plantar fascia which attaches at the
calcaneal tuberosity. Inability to flex the lesser toes refers to the flexor digitorum longus
tendon which runs superior to the sustentaculum. The spring ligament runs from the
navicular to the calcaneus, but does not attach under the sustentaculum tali. PREFE RE: 2

Question 30 shows the radiograph of a 27-year-old patient who has had a medial
forefoot prominence since he was a child. Over the past 6 years he notes progressive
pain in the first metatarsophalangeal joint. Modified shoe wear, custom orthotics, and
use of pads and toe spacers have failed to provide relief.He continues to experience
daily pain that affects both employment and recreation activities. Clinical examination
reveals good maintenance of first metatarsophalangeal joint motion and no evidence of
first tarsometatarsal joint hypermobility. What is the most appropriate treatment?

1. Continued nonsurgical management


2. Austin Chevron osteotomy
3. Biplanar distal first metatarsal Chevron osteotomy
4. Proximal first metatarsal osteotomy
5. Lapidus procedure

76
19
2012- -Foot and Ankle --Self-Assessment Examination by Dr.Dhahirortho
DISCUSSION: The hallux valgus deformity consists of a congruent joint with a moderately
severe abnormal distal metatarsal articular angle (DMAA). As such, the procedure that
will best correct the deformity is a biplanar distal first metatarsal Chevron osteotomy.
The patient has undergone an extended course of nonsurgical management with multiple
modalities; therefore, further nonsurgical management is unlikely to relieve his pain. An
Austin Chevron osteotomy will not correct the abnormal DMAA. He does not have an
increased 1-2 intermetatarsal angle so a proximal first metatarsal osteotomy will not
produce the desired correction. No hypermobility of the first tarsometatarsal joint is
noted so a Lapidus procedure is not indicated.

PREFERRED RESPONSE: 3

Question 31 A 24-year-old man dislocated his right knee in a motorcycle accident 1 year
ago. At the time, an anterior cruciate, posterior cruciate, medial collateral, and lateral
collateral ligament repair was done, but it was also noted that he sustained a complete
transection of the peroneal nerve. A primary nerve repair was done, but he has not
recovered any dorsiflexion of the ankle and continues to have a drop foot. Other than
using an ankle-foot orthosis, what is the best surgical option to regain maximum
function?

1. Sural nerve cable grafting of the peroneal nerve


2. Transfer of the peroneus longus to the tibialis anterior tendon
3. Transfer of the tibialis posterior to the dorsum of the foot
4. Transfer of the extensor hallucis longus to the tibialis anterior tendon
5. Ankle fusion to eliminate the need for an ankle dorsiflexor

DISCUSSION: With no recovery of dorsiflexion power 1 year after a peroneal nerve repair,
it can be assumed that the nerve will not recover. The peroneus brevis and extensor
hallucis longus are supplied by the peroneal nerve, so they will be nonfunctional. A nerve
grafting after an initial repair is less reliable than a transfer of the tibialis posterior tendon
in restoring active dorsiflexion to the ankle. An ankle fusion should not be the first choice
for an active, young patient.

PREFERRED RESPONSE: 3

77
20
2012- -Foot and Ankle --Self-Assessment Examination by Dr.Dhahirortho
Question 32Figures 32a and 32b are the radiographs of a 34-year-old woman who has a
painful ankle following an attempted fusion of her ankle 6 months ago. Infection work-
up was negative. The subtalar joint is pain free with manipulation. What is the most
appropriate treatment?

1. Removal of hardware and an intramedullary rod tibiocalcaneal fusion


2. Bone grafting of the nonunion
3. Use of an external bone stimulator
4. Removal of the plate and screws and the addition of an internal bone stimulator
5. Revision fusion with stable compression fixation and bone graft

DISCUSSION: Because the subtalar joint looks normal, and there is enough talus to work
with, the subtalar joint should be spared and only an ankle fusion performed, especially
in view of the patient’s young age. There is clearly a distraction at the fusion site and the
distal fixation is loose. The patient needs a formal revision with a transfibular approach
with compression screws. A simple bone grafting,removal of hardware, and/or a bone
stimulator will not be sufficient. PREFERRED RESPONSE: 5

Question 33Which of the following is associated with tarsal tunnel syndrome?


1. Adult-acquired flatfoot
2. Insertional Achilles tendinitis
3. Hypermobile first ray
4. Metatarsal stress fracture
5. Gastrocnemius contracture
78
21
2012- -Foot and Ankle --Self-Assessment Examination by Dr.Dhahirortho
DISCUSSION: Of the possible answer choices, only adult-acquired flatfoot is associated
with tarsal tunnel syndrome. The so-called "heel pain triad" includes adult-acquired
flatfoot, plantar fasciitis, and tarsal tunnel syndrome, in which failure of the dynamic and
static supports of the medial longitudinal arch increase traction on the tibial nerve.
PREFERRED RESPONSE: 1

Question 34 A 50-year-old man is having difficulty walking. He has a history of an injury


6 months ago. Examination reveals that he is unable to toe off and has increased ankle
dorsiflexion compared with the uninjured side.He has a positive Thompson test and a
palpable defect in the Achilles tendon. An MRI scan shows a 4-cm defect in the Achilles
tendon. What type of surgical reconstruction should be planned?
1. Primary end-to-end repair
2. V-Y advancement with or without flexor hallucis longus augmentation
3. Flexor hallucis longus tendon transfer
4. Flexor digitorum longus tendon transfer
5. Peroneus brevis tendon transfer

DISCUSSION: The patient has a chronic Achilles tendon rupture with a 4-cm defect. A V-Y
advancement avoids sacrifice of a normal muscle tendon unit and will fill a defect of 2 cm
to 5 cm. The flexor halluces longus may be used to augment a V-Y advancement,
particularly if there is significant scarring or atrophy of the gastrocnemius soleus complex.
Primary end-to-end repair is not an option because the distance is too great with the
contracture of the gastrocnemius soleus complex. A flexor hallucis longus tendo transfer
is indicated for defects of greater than 5 cm. Peroneus brevis tendon transfer is able to
augment the Achilles tendon but will compromise eversion strength. PREFE RESPONSE: 2

Question 35 An 18-year-old man sustained a traumatic laceration of the common


peroneal nerve when glass fell on the outer part of his leg 1 year ago. He has used a
molded foot and ankle orthosis for the past 10 months,but would now like surgical
intervention. Electromyography shows no function in the anterior or lateral
compartments. He has 5/5 muscle strength of the superficial and deep posterior
compartments. What is the most appropriate treatment?
1. Gastrocsoleus recession
2. Subtalar fusion
3. Split anterior tibial tendon transfer
4. Split posterior tibial tendon transfer
5. Flexor hallucis longus tendon transfer
79
22
2012- -Foot and Ankle --Self-Assessment Examination by Dr.Dhahirortho
DISCUSSION: In a patient with a drop foot and with 5/5 muscle strength of the posterior
tibial tendon, a split posterior tibial tendon transfer would be the most appropriate
treatment option based on the options presented. The deep peroneal nerve innervates
the anterior tibial tendon. This muscle has been affected by the injury; therefore, the
anterior tibial tendon cannot be transferred. A subtalar fusion would help correct
inversion and eversion deformities, but is not effective for plantar flexion deformities.
The foot drop is caused by a neurologic condition in this patient, not a contracture of the
gastrocsoleus complex.Therefore, a recession would not be beneficial. A flexor hallucis
longus tendon transfer would not take the deforming force and make it a corrective
force. PREFERRED RESPONSE: 4

Question 36Figures 36a and 36b are the AP and lateral radiographs of a 65-year-old
woman who has a dislocated second toe and a prominent bunion. Besides repairing the
bunion, what procedures are recommended to address the fixed second hammertoe
and the resulting metatarsalgia?

1. Proximal interphalangeal joint (PIP) resection arthroplasty


2. PIP resection arthroplasty, extensor tendon lengthening, and Weil osteotomy
3. Flexor to extensor tendon transfer and metatarsal head excision
4. PIP fusion and DuVries arthroplasty
5. Metatarsal head excision and proximal phalanx resection

DISCUSSION: The patient has a subluxated second metatarsophalangeal (MTP) joint, but
no evidence of second metatarsal head destruction. The patient also has a fixed claw toe.
The claw toe is repaired with a PIP resection arthroplasty or a PIP fusion. The likelihood of
80
23
2012- -Foot and Ankle --Self-Assessment Examination by Dr.Dhahirortho
completely correcting the MTP joint dislocation with just these two procedures, however,
is small and the addition of the Weil osteotomy is more likely to allow decompression of
the joint and complete relocation of the MTP joint. The flexor to extensor transfer is
indicated for a flexible hammertoe. A metatarsal head excision is a salvage option of the
toe that still cannot be reduced after the Weil osteotomy, but this may lead to transfer
lesions.The DuVries arthroplasty will not help reduce the MTP joint. Combined metatarsal
head excision and proximal phalanx resection would be extreme and is not
recommended. PREFERRED RESPONSE: 2

Question 37 During gait evaluation of a 25-year-old patient who had polio at age 5, it is
noted that the right foot slaps
the floor at heel strike, and the toes extend during the swing phase. Examination
reveals a flexible cavus foot, claw toes, and an equinus deformity. The patient has tried
various orthoses and would like surgical correction if possible. What is the most
appropriate treatment?
1. Calcaneal osteotomy, Achilles tendon lengthening, metatarsal osteotomies
2. Calcaneal osteotomy, Achilles tendon lengthening, extensor hallucis longus transfer
to the first metatarsal neck, flexor digitorum longus to extensor digitorum longus
transfer of the lesser toes
3. Calcaneal osteotomy, plantar fascia release, Achilles tendon lengthening, tibialis
posterior transfer to the dorsum of the foot, flexor digitorum longus to extensor
digitorum longus transfer of the lesser toes
4. Triple arthrodesis, Achilles tendon lengthening, extensor hallucis longus transfer to
the first metatarsal neck, flexor digitorum longus to extensor digitorum longus transfer
of the lesser toes
5. Plantar fascia release, Achilles tendon lengthening, extensor hallucis longus transfer
to the first metatarsal neck, tibialis posterior transfer to the dorsum of the foot, flexor
digitorum longus to extensor digitorum longus transfer of the lesser toes

DISCUSSION: Weakness of the tibialis anterior can be noted with a tendency of the foot
to slap the floor at heel strike. Extension of the toes during the swing phase of gait may
be due to the toe extensors attempting to substitute for weakness of the tibialis anterior.
Because this patient is young and has flexible deformities, avoiding arthrodesis is
recommended and soft-tissue procedures are recommended to balance the foot. The
plantar fascia release helps decrease the cavus. Transfer of the tibialis posterior tendon
to the dorsum of the foot is necessary to provide dorsiflexion and limit the slapping of the
foot on the floor.Transfer of the extensor hallucis longus to the metatarsal neck
81
24
2012- -Foot and Ankle --Self-Assessment Examination by Dr.Dhahirortho
addresses the claw toe deformity of the great toe and the flexor digitorum longus
transfer provides additional dorsiflexion assist. Because the patient has a flexible
deformity, osteotomies are unlikely to be needed. PREFERRED RESPONSE: 5

Question 38 A 45-year-old man has a grade 4 hallux rigidus secondary to a turf toe
sustained as a football player in high school. He is an avid golfer and plays tennis on
occasion. His activities are severely limited because of pain in his great toe and
nonsurgical management has failed to provide relief. His goal is to be pain free,
continue with his activities, and require no further orthopaedic care in the future. What
is the best
treatment option for this patient?
1. Keller’s excision arthroplasty
2. Bipolar replacement
3. Cheilectomy and débridement
4. Great toe metatarsophalangeal (MTP) fusion
5. Fascia lata interposition graft

DISCUSSION: There has been some research about interpositional grafting, whether
autologous or allograft, but there is no convincing evidence at this point that there is a
better functional outcome than with a great toe MTP fusion. A Keller procedure is not
indicated for a young, active person. A joint replacement of the great toe similarly has not
proven to be a reliable option for younger, active people.Cheilectomy will not provide
reliable pain relief in grade 4 arthrosis of the first MTP joint.PREFERRED RESPONSE: 4

Question 39Figure 39 is the radiograph of a 67-year-old woman with rheumatoid


arthritis who reports an 8-month history of increasing pain, swelling, and deformity.
Anti-inflammatory drugs, orthotics, and extra-depth shoes have failed to provide relief.
What is the next most appropriate step in treatment?

1. First metatarsophalangeal joint arthrodesis and lesser


metatarsal head resections
2. First metatarsophalangeal joint replacement and lesser
metatarsal head resections
3. Keller arthroplasty and lesser metatarsal head resections
4. Distal Chevron osteotomy and lesser metatarsal head resection
5. Lapidus procedure and Weil osteotomies

82
25
2012- -Foot and Ankle --Self-Assessment Examination by Dr.Dhahirortho
DISCUSSION: The patient has a severe rheumatoid forefoot deformity involving all
metatarsophalangeal joints. Coughlin and Mann have found that 90% of patients have
excellent and good results with combined first metatarsophalangeal fusion and lesser
metatarsal head resection. Keller Arthroplasty does not provide a stable platform for
walking and is associated with recurrent deformity and pain. The first
metatarsophalangeal joint replacement has not been shown to provide reliable long-term
results.Osteotomies may be indicated in patients without erosive joint changes. The
Lapidus procedure is an arthrodesis of the first tarsometatarsal joint, which would not
address the patient’s arthritic first metatarsophalangeal joint. PREFERRED RESPONSE: 1

Question 40 Figures 40a and 40b are the radiographs of a 53-year-old woman. If her
symptoms warrant, what is the most appropriate surgical management?

1. Plantar release
2. Dorsal cheilectomy
3. Interpositional soft-tissue arthroplasty
4. Arthrodesis
5. Prosthetic replacement

DISCUSSION: Surgical indications for management of hallux rigidus involve pain, reduced
range of motion, and degenerative changes at the first metatarsophalangeal joint. Using
the radiographic classification of hallux rigidus, grade 0 denotes normal or minimal joint-
space narrowing without osteophytes, whereas grade 1 denotes minimal joint-space
narrowing with primary dorsal spurring. In general, plantar release yields adequate
clinical results in the surgical management of grade 0 arthritis.Dorsal cheilectomy is used
in the surgical management of grade 1 or 2 hallux rigidus, and arthrodesis or resection
arthroplasty is used for grade 3 arthritis of the first metatarsophalangeal joint. Treatment
with prosthetic arthroplasty is controversial given the increased difficulty with salvage
procedures after possible failure. PREFERRED RESPONSE: 2

83
26
2012- -Foot and Ankle --Self-Assessment Examination by Dr.Dhahirortho
Question 41Figures 41a through 41c are the radiographs and Figure 41d is the biopsy
specimen of a 14-year-old girl who has had increasing foot pain for several months.
What is the most likely diagnosis?

1. Infection
2. Giant cell tumor
3. Unicameral bone cyst
4. Aneurysmal bone cyst
5. Hemangioma
DISCUSSION: Aneurysmal bone cysts frequently occur in the first two to three decades of
life. Patients report pain and a slow-growing lesion. Radiographs show an expansile lesion
with septae or striations.Treatment is usually curettage and grafting of the lesion. In the
foot, unicameral bone cysts are seen most frequently in the calcaneus, and are usually
incidental findings rarely requiring treatment. Infection or acute osteomyelitis typically
shows lucency of bone, periosteal reaction, and a permeative pattern on radiographs.
Patients often have systemic complaints as well. Giant cell tumor is usually seen in the
epiphysis of long bone with radiographs revealing a radiolucent lesion with a small rim of
reactive bone. PREFERRED RESPONSE: 4

Question 42 Figures 42a through 42c are the MRI scans of a 42-year-old woman who
has a 1.5-cm medial ankle mass.She has pain when shoes compress the area. A positive
Tinel’s sign is noted over the tarsal tunnel. What is the most likely diagnosis?

1. Astrocytoma
2. Neurilemoma
3. Neurofibroma
4. Lipoma
5. Ganglion

84
27
2012- -Foot and Ankle --Self-Assessment Examination by Dr.Dhahirortho
DISCUSSION: Neurilemoma (Schwannoma) is a benign tumor of nerve sheath (Schwann
cell) origin.It is usually a solitary, well-encapsulated lesion located on the surface of a
peripheral nerve. Careful excision without damaging the underlying nerve is the
treatment of choice. Neurofibroma is a spindle cell tumor arising within a peripheral
nerve. Due to its location, it can interfere with distal nerve function.Neurofibromas can
be solitary or multiple. A portion of these patients have von Recklinghausen’s
disease.Because of the invasive nature of the tumor, resection requires removal of the
affected nerve, resulting in distal nerve dysfunction. This lesion does not show the MRI
characteristics of either a lipoma or a ganglion.
PREFERRED RESPONSE: 2

Question 43Figures 43a and 43b are the MRI scans of a 54-year-old woman who reports
a 2-year history of progressive shooting and burning-type pain in the posteromedial
ankle. What is the most appropriate management?

1. Needle aspiration
2. Tarsal tunnel release
3. Incisional biopsy
4. Surgical excision
5. Referral to an orthopaedic surgeon
specializing in oncology

DISCUSSION: The MRI scans show a bright, homogeneous mass (white arrow) on the T2-
weighted images consistent with a ganglion cyst, which is likely emanating from the
flexor digitorum longus tendon sheath. Because of the neuritic symptoms from the tibial
nerve, the patient is best managed with surgical excision. Whereas needle aspiration can
provide temporary relief by mass decompression, the location of the lesion in this
instance, adjacent to the tibial nerve and posterior tibial artery, makes this option less
optimal. Although the cyst is deep to the flexor retinaculum, which necessitates a tarsal
tunnel release to access the lesion, release of the flexor retinaculum alone will not likely
provide full resolution of symptoms. Incisional biopsy is indicated for potentially
malignant lesions, which also makes referral to an orthopaedic surgeon specializing in
oncology in this case unnecessary.
PREFERRED RESPONSE: 4

85
28
2012- -Foot and Ankle --Self-Assessment Examination by Dr.Dhahirortho
Question 44Figures 44a through 44c are the MRI scans of a 45-year-old man who has an
enlarging mass on the right foot and has difficulty wearing shoes. What is the most
appropriate management for this tumor?

1. Amputation
2. Marginal excision
3. Radical excision
4. Local excision with adjuvant chemotherapy
5. Radiation therapy
DISCUSSION: A lipoma in the foot frequently presents as a dorsal foot mass. The MRI
appearance of the lesion is homogenous with density of subcutaneous fat on all
sequences. There is no enhancement of the lesion with administration of contrast. The
mass is consistent with a simple lipoma. Treatment for a simple lipoma is marginal
excision. Amputation, radical excision, and adjuvant therapies are most appropriate for
malignant tumors. PREFERRED RESPONSE: 2
Question 45 Figures 45a through 45e are the MRI scans, gross specimen, and histology
of the specimen of a 19-yearold man who has an enlarging mass in the second
interspace. He reports forefoot pain that is worse with athletic activity. Radiographs
show erosive changes of the third metatarsal head. What is the most common
complication associated with incomplete excision?

1. Metastatic disease
2. Malignant degeneration
3. Recurrence
4. Pathologic fracture
5. Infection

86
29
2012- -Foot and Ankle --Self-Assessment Examination by Dr.Dhahirortho

DISCUSSION: Giant cell tumor of the tendon sheath often arises from the synovial lining
of tendon sheaths. This lesion is frequently found in the hand and foot. The lesion is slow
growing and can invade adjacent structures. In the foot, wearing shoes or increased
activity can cause pain. Incomplete or piecemeal excision can lead to recurrence. PR RE: 3

Question 46 Figures 46a and 46b are the radiographs of a 20-year-old collegiate varsity
athlete who reports lateral foot pain. What is the most appropriate management at this
time?

1. Rest and nonsteroidal anti-inflammatory drugs


2. Orthosis and non-weight-bearing status
3. Orthosis, weight bearing as tolerated, and use of a
bone stimulator
4. Short-leg cast
5. Internal fixation

87
30
2012- -Foot and Ankle --Self-Assessment Examination by Dr.Dhahirortho
DISCUSSION: Fractures of the fifth metatarsal proximal metaphyseal-diaphyseal junction
(Jones fracture)generally occur in young athletic patients and have relatively high rates of
delayed union or nonunion with nonsurgical management. The fracture occurs in the
hypovascular zone between the insertion of the peroneus brevis and tertius. These
tendons cause a shearing across the fracture site, preventing stability and healing.
Nonsurgical functional bracing or casting may lead to a high rate of delayed union and
nonunion. Internal fixation in the high-level athlete leads to the most predictable healing
of the fracture in a timely fashion. The use of bone stimulators for this fracture is
controversial.

PREFERRED RESPONSE: 5

Question 47 The lesion in Figure 47 would most likely cause which of the following
symptoms?
1. Anterior tarsal tunnel paresthesias
2. Burning and numbness on the bottom of the foot
3. Heel pain that is worst in the morning
4. Night pain in the heel
5. Loss of toe extension

DISCUSSION: The lesion would most likely cause burning and numbness on the bottom of
the foot,symptoms related to tarsal tunnel, or entrapment of the tibial nerve and its
medial and lateral plantar nerves. The MRI scan shows a benign ganglion cyst that is in
the tarsal tunnel. This would irritate the nerves and cause pain in the medial heel and
plantar foot. Anterior tarsal tunnel paresthesias refers to the anterior tarsal tunnel that is
anterior to the ankle and involves the deep peroneal nerve. Heel pain that is most severe
in the morning refers to plantar fasciitis. Loss of toe extension is incorrect because the
mass may affect toe flexion, not extension. Night pain in the heel is more common with a
malignant tumor.

PREFERRED RESPONSE: 2

88
31
2012- -Foot and Ankle --Self-Assessment Examination by Dr.Dhahirortho
Question 48 A 42-year-old woman sustained an open grade 3B tibial shaft fracture with
a severe degloving injury involving the anterior and lateral compartments 1 year ago.
She underwent multiple débridements,definitive fracture treatment, and flap coverage.
She now reports that she has difficulty ambulating.Examination includes a 20°
equinovarus contracture, 2+ dorsiflexion, 2+ eversion, 5+ inversion, and 5+ plantar
flexion strength. She has a supple forefoot and intact sensation throughout. Figures 48a
through 48c are current weight-bearing radiographs. Attempted surgical correction
should include Achilles lengthening, calcaneal osteotomy, and

1. flexor hallucis longus to peroneal transfer.


2. posterior tibial tendon transfer.
3. split anterior tibial tendon transfer.
4. first metatarsal osteotomy.
5. ankle arthrodesis.

DISCUSSION: The patient has developed an equinovarus contracture from overpull of the
posterior tibial tendon, presumably due to the degloving injury and lack of protective
splinting. In this instance, the patient is best treated with an Achilles lengthening,
lateralizing calcaneal osteotomy, and posterior tibial tendon transfer. Flexor hallucis
longus to peroneal transfer will not restore dorsiflexion. Ankle arthrodesis would allow
deformity correction through the ankle joint but does not address the remainder of the
foot,such that the patient would likely have trouble clearing the floor during the swing
phase of gait. A first metatarsal osteotomy is indicated in the instance of a forefoot-
driven cavus foot deformity. A split anterior tibial tendon transfer is indicated for
correction of ankle and hindfoot varus contractures, and requires an intact and functional
anterior tibial tendon, making it contraindicated in this instance. PREFERRED RESPONSE: 2

89
32
2012- -Foot and Ankle --Self-Assessment Examination by Dr.Dhahirortho
Question 49 A 45-year-old woman with type 2 diabetes (BMI 38, Hgb A1c 7.4) has a
grade II ulcer under the first metatarsal head. Previous treatment with a custom
orthosis and total contact casting has provided only temporary healing. Her ankle-
brachial index is 0.95, she has no foot deformity, and there is no evidence of infection.
What is the next most appropriate step in management?
1. Custom-molded Plastizote orthotics
2. Gastrocsoleus recession and peroneus longus to brevis tendon transfer
3. Resection of the first metatarsal head
4. First-ray amputation
5. Transmetatarsal amputation

DISCUSSION: Ulceration in neuropathy is a consequence of increased pressure. When it


occurs in the forefoot, it is frequently associated with a tight heel cord. In addition,
lesions under the first metatarsal often have an associated overpull of the peronus
longus, plantar flexing the first metatarsal. Before bony resections are contemplated, a
fractional lengthening of the heel cord (gastrocsoleus recession) as well as a peroneal
longus to brevis tendon transfer will decrease the forefoot pressures and leads to
decreased ulcer recurrence rates. Orthotics alone will not be successful because they
would have been used after the initial trial of total contact casting. PREFER RESPONSE: 2

Question 50 The MRI scan of the ankle shown in Figure 50 reveals a tear of what
structure?
1. Superficial peroneal retinaculum
2. Posterior tibial tendon
3. Peroneus longus tendon
4. Peroneus brevis tendon
5. Anterior talofibular ligament (ATFL)

DISCUSSION: The MRI scan shows a tear of a tendon


behind the fibula. The peroneus brevis is deep to the
longus (closer to the fibula). There should only be two
structures behind the fibula, but the peroneus brevis
tendon is shown as two distinct structures which is clearly
a longitudinal tear. The posterior tibial tendon would be posteromedial to the tibia. The
superficial peroneal retinaculum is behind the fibula, but would show up as a tear if the
peroneal tendons were dislocated, or lateral to the fibula. The ATFL runs from the
anterior fibula to the talus. PREFERRED RESPONSE: 4
90
33
2012- -Foot and Ankle --Self-Assessment Examination by Dr.Dhahirortho
Question 51 What is the most common pathogen for soft-tissue infection of the foot
caused by a puncture wound?

1. Staphylococcus aureus
2. Pseudomonas aeruginosa
3. Eikenella corrodens
4. Pasteurella multocida
5. Vibrio species

DISCUSSION: Staphylococcus and Streptococcus species are the most common causes of
soft-tissue infections in the foot due to punctures. Pseudomonas is the most common
cause of osteomyelitis of the foot due to puncture wounds. Pasteurella and Eikenella are
seen in animal and human bites, respectively.Vibrio species are found in marine
environments. PREFERRED RESPONSE: 1

Question 52 A 38-year-old man with a congenital pes cavus deformity reports lateral
foot pain that has become increasingly debilitating. He has calluses over the lateral
column and 3/5 muscle strength of the lateral compartment muscles. Nonsurgical
management has failed to provide relief. In surgery, he undergoes a plantar fascial
release, peroneus longus to brevis transfer, dorsiflexion osteotomy of the first
metatarsal,and a Dwyer osteotomy. He has a hyperextended deformity of the first
metatarsophalangeal joint. What tendon transfer will help to address this deformity?

1. Flexor hallucis longus


2. Extensor hallucis longus
3. Extensor hallucis brevis
4. Extensor digitorum longus
5. Tibialis anterior

DISCUSSION: In cavus foot reconstructions with a hyperextended deformity of the first


metatarsophalangeal joint, a first-toe Jones procedure is indicated. This is an
interphalangeal joint fusion of the first toe with an extensor hallucis longus tendon
transfer. The flexor hallucis longus, extensor hallucis brevis, extensor digitorum longus,
and tibialis anterior tendons are not of adequate length or in the correct direction to
correct this deformity. PREFERRED RESPONSE: 2

91
34
2012- -Foot and Ankle --Self-Assessment Examination by Dr.Dhahirortho
Question 53 What is the most common complication following surgical treatment of a
displaced talar neck fracture?
1. Osteonecrosis
2. Varus malunion
3. Posttraumatic arthritis
4. Fracture delayed union/nonunion
5. Wound dehiscence/delayed wound healing

DISCUSSION: The most frequent complication is posttraumatic arthritis. With talar neck
fractures,osteonecrosis is relatively common, occurring in up to 50% of patients. Fracture
nonunion occurs in 10%to 12% of patients. Varus malunion can occur with medial
comminution. Wound dehiscence and deep infection are much less frequently
encountered. PREFERRED RESPONSE: 3

Question 54A 44-year-old woman with forefoot pain has pain with weight bearing
during toe-off. She reports the pain is worse when she is barefoot and better when
wearing tennis shoes. She has no numbness or tingling. Examination reveals increased
pain with second toe dorsiflexion and plantar flexion. Traction to the second toe
decreases pain with motion. She has no pain with medial lateral forefoot
compression.Radiographically, her second metatarsal is longer than the first. What is
the most likely diagnosis?

1. Second metatarsal stress fracture


2. Second metatarsophalangeal (MTP) synovitis
3. Second to third web space neuroma
4. Second flexible hammer toe
5. Transfer metatarsalgia

DISCUSSION: A test for metatarsophalangeal synovitis is pain with motion that is


decreased with traction and motion. A patient with a neuroma typically has less pain
when barefoot, does not have pain with MTP motion, and often has pain and a click with
medial lateral forefoot compression. A metatarsal stress fracture would cause pain with
weight bearing. The drawer test is the most useful test for diagnosis of instability of the
metatarsophalangeal joint. Transfer metatarsalgia could be considered but typically does
not cause pain with toe motion or have a positive drawer test. PREFERRED RESPONSE: 2

92
35
2012- -Foot and Ankle --Self-Assessment Examination by Dr.Dhahirortho
Question 55 Figure 55 is the radiographs of a 37-year-old patient who reports pain and
swelling over the lateral forefoot (fifth metatarsal) that has become progressively
worse over time. Shoe wear modifications have not been successful. Based on the
radiographs, what is the appropriate treatment at this time?
1. Chevron osteotomy
2. Diaphyseal osteotomy
3. Metatarsal head resection
4. Exostectomy of the lateral eminence
5. Exostectomy of the lateral eminence with a fifth toe
extensor tenotomy and capsular release

DISCUSSION: The most appropriate treatment is a


diaphyseal osteotomy. The patient has a painful
bunionette with a large 4-5 intermetatarsal (IM) angle
(a type 3 bunionette) that is best treated with a
diaphyseal or proximal osteotomy. Exostectomy of the
lateral eminence applies to type 1 bunionettes that can
also have a fifth hammer toe. A Chevron osteotomy is best used for a type 1 or 2
bunionette with a normal 4-5 IM angle. A metatarsal head resection is not indicated as a
primary procedure for a bunionette,especially in a younger patient because it sacrifices
the fifth metatarsophalangel joint. PREFERRED RESPONSE: 2

Question 56 Figures 56a through 56c are the lateral radiograph and MRI scans of a 32-
year-old woman who reports a 3-week history of heel pain, tenderness, swelling, and
onset following an increase in running activity.What is the most likely diagnosis?

1. Plantar fasciitis
2. Atrophic heel pad
3. Achilles tendinitis
4. Retrocalcaneal bursitis
5. Stress fracture of the calcaneus
93
36
2012- -Foot and Ankle --Self-Assessment Examination by Dr.Dhahirortho
DISCUSSION: The sagittal T1-weighted MRI scan reveals a linear streak of low signal
intensity consistent with a stress fracture of the posterior calcaneal tuberosity. The
surrounding area of hypointensity on the T1 MRI scan and the hyperintensity on the T2
MRI scan represent bone contusion, hemorrhage, and edema within the calcaneus, with
an unremarkable radiograph. The plantar fascia, Achilles tendon, retrocalcaneal bursa,
and heel pad all display normal signal in the accompanying MRI scans. PREFERRED RESP: 5

Question 57 A 32-year-old ballet dancer has chronic early stage 2 hallux rigidus. Over
the past year she has been treated with several nonsurgical options, but continues to
be increasingly symptomatic. She decided that she can take a 3-month hiatus in her
career to deal with this problem. What is the most appropriate treatment option?

1. Cheilectomy, débridement, and a Moberg dorsiflexion osteotomy if needed


2. Great toe metatarsophalangeal (MTP) joint fusion
3. Keller excision arthroplasty
4. Unipolar replacement
5. Bipolar replacement

DISCUSSION: A cheilectomy and débridement for a stage 2 hallux rigidus is still the most
accepted procedure. Adding a dorsiflexion osteotomy to the proximal phalanges
increases the total dorsiflexion of the MTP complex. A great toe fusion for a ballet dancer
is a career ending procedure. There is no evidence in the literature that a unipolar or
bipolar replacement will extend a dancer’s career, or that it is superior to a cheilectomy.
A Keller procedure is reserved for low physical demand patients. PREFERRED RESPON: 1

Question 58 Figure 58 is the radiograph of a laborer who has hindfoot and ankle pain.
He is a type 1 diabetic, and has a BMI of 25 and a Hgb A1c of 6. What is the most
appropriate management at this time?

1. Total contact casting


2. Arthrodesis
3. Open reduction and internal fixation
4. Bed rest
5. Standard walking boot

94
37
2012- -Foot and Ankle --Self-Assessment Examination by Dr.Dhahirortho
DISCUSSION: The radiograph shows Charcot changes in the subtalar joint. In the absence
of gross deformity, the initial treatment is nonsurgical, consisting of total contact casting,
with frequent cast changes and progression to weight bearing when swelling subsides
and early consolidation is seen radiographically. A walking boot will not provide sufficient
immobilization, whereas bed rest carries the risk of significant deconditioning in an
otherwise active patient with well-controlled diabetes. Surgery as the initial treatment is
not indicated in the absence of gross deformity or ulceration. PREFERRED RESPONSE: 1

Question 59 A 26-year-old competitive skier sustained an injury to her right ankle and
now reports pain and clicking.Radiographs obtained at the time of the injury did not
show any abnormality. She was diagnosed with an ankle sprain and treated in a short-
leg cast for 6 weeks. While in the cast she was comfortable but the pain and clicking
returned almost immediately after the immobilization was discontinued. Physical
therapy has only made the problem worse. Current MRI scans are shown in Figures 59a
and 59b. What is the most appropriate treatment at this time?

1. A stirrup splint
2. Continuation of cast immobilization until the clicking stops
3. Débridement and repair of longitudinal tears within the peroneal tendons
4. Débridement of the peroneal tendons, fibular groove deepening, and repair of the
superior peroneal retinaculum
5. Excision of the region of the diseased peroneal tendon, tenodesis of the stumps to
the intact tendon, and repair of the peroneal retinaculum

DISCUSSION: The MRI scans show dislocated peroneal tendons. Figure 59b reveals a
convex fibular groove. Because cast immobilization
95 has failed to provide relief,
38
2012- -Foot and Ankle --Self-Assessment Examination by Dr.Dhahirortho
débridement, fibular groove deepening,and repair of the superior peroneal retinaculum
will address all of the patient’s problems. Further immobilization is unlikely to produce
any improvement. A stirrup splint similarly is unlikely to give her relief. Surgical
débridement with repair of tears would not alone prevent the recurrent dislocation.
There is no evidence of longitudinal tears based on the images. Excision of any area of
diseased tendon with tenodesis to the remaining tendon does not address the problem
of dislocation. PREFERRED RESPONSE: 4

Question 60 Figure 60 is the radiographs of a patient who underwent surgery to


alleviate pain under her second metatarsal that is worsened by wearing high heel
shoes. What is the most common complication of the osteotomy shown in the
radiographs?
1. Osteonecrosis
2. Nonunion of the osteotomy
3. Significant transfer lesions
4. Metatarsophalangel (MTP) arthritis
5. Dorsiflexion contracture at the MTP joint

DISCUSSION: The radiographs show a Weil osteotomy


that is used to treat metatarsalgia, which is often
associated with severe claw toes. The most common
complication is a floating toe, or dorsiflexion contracture
at the MTP joint. This is because the interossei muscles move dorsally with respect to the
axis of the MTP joint due to depression of the plantar fragment and that the center of
rotation is altered after the osteotomy. Multiple studies have shown that the floating toe
is the main complication and that the other listed complications rarely occur with this
type of osteotomy. PREFERRED RESPONSE: 5

Question 61Figures 61a and 61b are the radiographs of a 56-year-old woman who
reports medial foot and ankle pain and notes a progressive change in the shape of her
foot over the past year. Her normal activities are limited by pain. Nonsurgical
management has failed to provide relief. Pain is present from the navicular to the
medial malleolus. Single leg heel rise is accompanied by correction of hindfoot valgus
but is painful. What is the best course of treatment?

96
39
2012- -Foot and Ankle --Self-Assessment Examination by Dr.Dhahirortho

1. Débridement of the posterior tibial tendon


2. Transfer of the flexor digitorum longus to the medial
navicular
3. Medializing calcaneal osteotomy with transfer of the flexor
digitorum longus to the medial navicular
4. Medializing calcaneal osteotomy with lateral column lengthening and flexor
digitorum longus transfer to the medial navicular
5. Triple arthrodesis
DISCUSSION: The radiographs reveal significant pes planus with a large degree of talar
head uncoverage.This is a posterior tibial insufficiency with a stage 2 correctable
deformity. Reconstruction requires tendon transfer, lateral column lengthening to
address the talar uncoverage, and a medializing calcaneal osteotomy. The medializing
calcaneal osteotomy corrects the hindfoot valgus and protects the tendon transfer.
Débridement of the posterior tibial tendon has been shown to be effective treatment for
stage I adult-acquired flatfoot deformity but not more advanced stages. Transferring a
tendon without correction of the hindfoot valgus will lead to early failure of the transfer.
This patient does not require a triple arthrodesis because the deformity is supple and she
has no radiographic evidence of arthritis. PREFERRED RESPONSE: 4
Question 62 A 72-year-old woman with a moderately reducible hallux varus has pain in
the first metatarsophalangeal(MTP) joint that is activity related and reports that she
cannot find any comfortable shoes. She wants to know what treatment plan offers her
the most predictable outcome in terms of pain relief, activity, and the ability to get into
shoes?
1. First MTP fusion
2. MTP joint replacement
3. Great toe amputation
4. Keller resection arthroplasty
5. Tendon transfer and capsular release
97
40
2012- -Foot and Ankle --Self-Assessment Examination by Dr.Dhahirortho
DISCUSSION: A great toe fusion is the most appropriate treatment. It is an excellent
procedure for pain relief and it gives a predictable result for return to activity and lack of
recurrence. A soft-tissue correction is not indicated due to patient age and reducibility.
An amputation is not indicated in this case in terms of activity level and is an
unreasonable choice for an otherwise healthy 72-year-old patient. The Keller resection
arthroplasty and the MTP joint replacement allow motion, but they offer unpredictable
results for pain relief, activity, and recurrence. PREFERRED RESPONSE: 1

Question 63 A 43-year-old woman with long-standing rheumatoid arthritis has a large


prominence with soft-tissue swelling under the fifth metatarsal head and over the
lateral eminence of the fifth metatarsophalangeal(MTP) joint. She has minimal hammer
toes with no significant metatarsalgia. Radiographs show a 4-5intermetatarsal angle of
7° and a congruent fifth MTP joint. What is the recommended surgical treatment to
address this problem?
1. Simple exostectomy
2. Metatarsal head excision
3. Distal metatarsal osteotomy
4. Diaphyseal metatarsal osteotomy
5. Simple exostectomy with soft-tissue mass excision

DISCUSSION: Exostectomy with soft-tissue mass excision is the treatment of choice. The
patient has a type 1 bunionette but most of her symptoms are coming from a rheumatoid
nodule that is under the metatarsal head. This is mainly a soft-tissue problem and does
not require any type of osteotomy because the 4-5 intramedullary angle is minimally
elevated. A metatarsal head resection is commonly performed in patients with
rheumatoid arthritis, but is not indicated in this patient because there is a normal fifth
MTP joint and no metatarsalgia. PREFERRED RESPONSE: 5

Question 64 A 15-year-old boy has a unilateral flatfoot that is preventing sporting


activities. After nonsurgical management fails, he undergoes surgery to correct a
calcaneonavicular coalition. What procedure will most likely allow him to return to
sports?
1. No surgical procedure is likely to allow a return to sports
2. Subtalar fusion
3. Arthroereisis
4. Closed manipulation under anesthesia
5. Bar resection with tissue interposition
98
41
2012- -Foot and Ankle --Self-Assessment Examination by Dr.Dhahirortho
DISCUSSION: Restoration of function is the expected outcome from resection of a
symptomatic calcaneonavicular coalition. There is a high likelihood of a return to sports
after a successful bar resection with tissue interposition. Arthrodesis is reserved for
salvage or in adults, whereas arthroereisis is controversial and unproven. Closed
manipulation will not disrupt the coalition enough to improve motion. PREFERRED RES: 5

Question 65Figures 65a through 65c are the weight-bearing radiographs of a 42-year-
old male manual laborer who has a 6-month history of persistent great toe swelling and
pain after undergoing a total joint Arthroplasty for hallux rigidus 9 months ago. He
denies postoperative wound complications, recent fevers, chills, or other constitutional
signs; however, he has never been able to ambulate without pain since his return to
work. Examination reveals moderate diffuse swelling, but no fluctuance or drainage.
Range of motion includes 25° of dorsiflexion. Laboratory studies show an erythrocyte
sedimentation rate of 18 mm/h and a c-reactive protein level of <0.7 mg/L. What is the
most likely source of his symptoms?

1. Septic arthritis
2. Mechanical failure
3. Periprosthetic fracture
4. Aseptic loosening from polyethylene debris
5. Metatarsal shortening/transfer metatarsalgia

DISCUSSION: The patient has developed mechanical failure as evidenced by the lucency
surrounding the proximal phalanx component. His pain has correlated with his return to
work as a manual laborer.Although septic arthritis is a possibility, it is less likely based on
the normal laboratory studies and lack of infectious signs. Periprosthetic fracture is
unlikely because of the lack of a traumatic event or a sudden change in symptoms rather
than a persistent inability to progress his activities. A transfer lesion from metatarsal
shortening would result in pain from mechanical overload at areas adjacent to the first
99
42
2012- -Foot and Ankle --Self-Assessment Examination by Dr.Dhahirortho
metatarsal. Aseptic loosening from polyethylene debris would imply that the implant has
previously been stable and well-fixed, and subsequently loosened over an extended
period of time. PREFERRED RESPONSE: 2

Question 66 A patient with foot pain is noted to have a cavovarus foot. The heel
corrects to slight valgus on Coleman block testing. This finding indicates that the
deformity should correct with which of the following procedures?
1. Triple arthrodesis
2. Subtalar arthrodesis
3. Peroneal brevis lengthening
4. Medializing calcaneal osteotomy
5. Dorsiflexion first metatarsal osteotomy

DISCUSSION: The Coleman block test is used to demonstrate a flexible hindfoot. If the
heel corrects from varus to neutral or slight valgus by bearing weight on a block
supporting the lateral column of the foot, the subtalar joint remains flexible. This
indicates that the varus position is secondary to the plantar flexed first ray or valgus
position of the forefoot. Therefore, the most appropriate procedure is a dorsiflexion first
metatarsal osteotomy. Arthrodesis is indicated in degenerative conditions. The peroneal
brevis does not contribute to the cavus foot deformity. Medializing calcaneal osteotomy
assists in correction of a flexible flatfoot. PREFERRED RESPONSE: 5

Question 67 A 19-year-old woman sustained a displaced talar neck fracture while cliff
jumping. The fracture is managed with open reduction and internal fixation. Which of
the following best describes the findings in the 2-months postoperative radiographs
shown in Figures 67a and 67b, and subsequent treatment plan?

100
43
2012- -Foot and Ankle --Self-Assessment Examination by Dr.Dhahirortho
1. There is a positive Hawkins sign, indicating the patient is unlikely to develop
osteonecrosis.
2. There is a positive Hawkins sign, indicating the patient has developed osteonecrosis.
3. Hawkins sign cannot be determined on radiographs; therefore, MRI is required.
4. No Hawkins sign is visible, and therefore the patient is not likely to develop
osteonecrosis.
5. No Hawkins sign is visible; therefore, the patient should be kept non-weight-bearing
until a Hawkins sign appears

DISCUSSION: The radiographs reveal a positive Hawkins sign, a subchondral lucency in the
talar dome best seen on a mortise radiograph indicating viability of the talar body. Once a
Hawkins sign appears,it is unlikely that that the patient will develop osteonecrosis.
Osteonecrosis is best diagnosed with radiographs. Although MRI can be helpful in
assessing the extent of osteonecrosis, it is unnecessary for purely diagnostic purposes. A
Hawkins sign typically will appear at 6 to 8 weeks after fracture; however,the absence of
a Hawkins sign at that time does not necessarily indicate osteonecrosis. Most authors
agree that even in the absence of a Hawkins sign, weight bearing can commence at 10 to
12 weeks after surgery.

PREFERRED RESPONSE: 1

Question 68 A middle-aged man sustains traumatic loss of the second, third, and fourth
toes in a lawnmower accident.The wound is grossly contaminated with soil. Penicillin is
added to his antibiotic regimen for coverage of what bacteria?

1. Clostridium
2. Acinetobacter
3. Pseudomonas
4. Mycobacterium
5. Staphylococcus aureus

DISCUSSION: In farm or soil-contaminated wounds, including lawnmower injuries,


penicillin is added to broad-spectrum cephalosporin and aminoglycoside therapy to cover
against Clostridium. Psuedomonas is frequently seen after puncture wounds through the
shoes. Acinetobacter is generally a hospital-acquired infection.

PREFERRED RESPONSE: 1

101
44
2012- -Foot and Ankle --Self-Assessment Examination by Dr.Dhahirortho
Question 69 A 35-year-old man sustains a large degloving injury overlying the distal
tibia. The traumatic wound is managed with surgical débridement, followed by
application of a negative pressure dressing. Compared with standard damp-to-dry
dressing changes, use of a negative pressure dressing offers which of the following
advantages?

1. Increased limb vascularity


2. Decreased hospital stay
3. Decreased bacterial count
4. Decreased need for repeat débridement
5. Accelerated granulation tissue formation

DISCUSSION: The development of negative pressure dressings has been a significant


advance in wound management. These devices remove excess interstitial fluid, which
promotes increased local vascularity and, with mechanical deformation of cells from the
negative pressure, accelerates granulation tissue formation. A negative pressure dressing
does not substitute for thorough surgical débridement; it has not been shown to
decrease bacterial counts within the wound or decrease overall length of hospital stays.

PREFERRED RESPONSE: 5

Question 70What is the most common cause of persistent pain after excision of a
Morton neuroma?

1. Tarsal tunnel syndrome


2. Painful plantar scar formation
3. Metatarsophalangeal joint synovitis
4. Presence of an amputation stump neuroma
5. Inadequate resection of an interdigital neuroma
DISCUSSION: The most frequent cause of recurrent pain after neuroma excision is
tethering of the common digital nerve to the plantar skin by plantar-directed branches of
the nerve, possibly preventing retraction of the nerve, due to inadequate resection of the
neuroma. These plantar branches were not found to be present 4 cm proximal to the
transverse metatarsal ligament. Therefore, an effort should be made to cut the nerve at
this level.
PREFERRED RESPONSE: 5

102
45
2012- -Foot and Ankle --Self-Assessment Examination by Dr.Dhahirortho
Question 71 A 40-year-old man with lateral column overload and a cavovarus foot has
failed to respond to nonsurgical management. Examination reveals an Achilles tendon
contracture. With the knee in extension, ankle dorsiflexion is to neutral; with the knee
in flexion, ankle dorsiflexion is to 15°. In addition to correction of the cavovarus
deformity, what is the most appropriate surgical management with regard to the
Achilles tendon contracture?

1. Heel cord lengthening


2. Percutaneous Achilles tendon lengthening
3. Open Achilles tendon lengthening
4. Gastrocnemius recession
5. Gastrocnemius and soleus recession

DISCUSSION: The Silfverskiold test indicates that the patient has an isolated contracture
of the gastrocnemius; therefore, a gastrocnemius recession is indicated. Open Achilles
tendon lengthening,gastrocnemius and soleus recession, and percutaneous Achilles
tendon lengthening are all indicated for management of concomitant gastrocnemius and
soleus contractures. PREFERRED RESPONSE: 4

Question 72 A 28-year-old man reports a 3-month history of foot pain and swelling
after stepping on a nail while working at a construction site. He was wearing rubber-
soled boots at the time he sustained this deep puncture wound. Initial management
consisted of tetanus prophylaxis, superficial wound cleansing, and oral antibiotics.
Imaging shows no evidence of bony infection. What is the most appropriate treatment?
1. IV antibiotics
2. Reinstitution of oral antibiotics
3. CT-guided drainage procedure and IV antibiotics
4. Surgical wound exploration with débridement followed by hydrotherapy
5. Surgical wound exploration with débridement and IV broad-spectrum antibiotics

DISCUSSION: Patients wearing rubber-soled shoes while sustaining deep puncture wound
injuries to the foot may experience complications such as abscess formation and
osteomyelitis. Frequently, there is delayed diagnosis of potential retained foreign bodies.
Therefore, appropriate treatment involves wound exploration, débridement with
removal of the foreign material, and IV antibiotics. A combination of formal surgery and
administration of antibiotics is required for treatment of these deep wound
infections;drainage or antibiotics alone will not suffice. PREFERRED RESPONSE: 5
103
46
2012- -Foot and Ankle --Self-Assessment Examination by Dr.Dhahirortho
Question 73 Figures 73a through 73c are the radiographs of a 14-year-old girl who
sustained an ankle injury in a fall.What ligament is attached to the displaced fragment?

1. Long plantar ligament


2. Anterior talofibular ligament
3. Posterior talofibular ligament
4. Anterior inferior tibiofibular ligament
5. Posterior inferior tibiofibular ligament

DISCUSSION: The patient has sustained a Tillaux fracture. This fracture of the
anterolateral portion of the distal tibia epiphysis occurs in early adolescence. The medial
portion of the growth plate fuses first.The anterior inferior tibiofibular ligament attaches
to the anterolateral portion of the tibial epiphysis and avulses the fragment, usually in
response to an external rotation force. The other named ligaments are lateral in location
but do not attach to the shown tibial fragment. PREFERRED RESPONSE: 4

Question 74 What is the most common associated pathology to look for in patients
with fifth metatarsal stress fractures?
1. Hindfoot varus deformity
2. Lateral ligament instability
3. Talocalcaneal tarsal coalition
4. Peroneus brevis tendon rupture
5. Anterior process calcaneal fracture

DISCUSSION: There is increasing evidence that stress fractures of the fifth metatarsal
could be related to a cavus or cavovarus foot deformity. With a cavus deformity the
hindfoot is in varus, adding to the overload of the lateral border of the foot. Especially in
104
47
2012- -Foot and Ankle --Self-Assessment Examination by Dr.Dhahirortho
chronic or recurring stress fractures, there should be a high index of suspicion of a
hindfoot varus deformity. A peroneus brevis rupture has no specific relationship with fifth
metatarsal stress fractures, but could also occasionally be related to a cavovarus foot. A
talocalcaneal tarsal coalition will most often cause a flatfoot (plano-valgus) deformity,
that in essence "protects" the fifth metatarsal from overload stresses. Anterior process
fractures of the calcaneus have no relation to fifth metatarsal stress fractures.
PREFERRED RESPONS: 1

Question 75 Hallux rigidus can lead to which of the following?

1. Everted gait
2. Increased push-off
3. Haglund deformity
4. Transfer metatarsalgia
5. Flexor hallucis longus rupture

DISCUSSION: With hallux rigidus, restricted dorsiflexion of the first metatarsophalangeal


joint during ambulation may lead to transfer metatarsalgia. In addition, the patient may
have decreased push-off strength and develop an inverted gait. There is no association
with flexor hallucis longus rupture or Haglund deformity of the hindfoot. PREFERESP: 4

Question 76When using a single-incision flexor hallucis longus transfer for


augmentation of a repair for chronic Achilles tendon rupture, which of the following
can be expected?

1. Low functional scores (AOFAS MTP-IP scores)


2. Significantly decreased hallucal phalangeal pressure
3. Transfer metatarsalgia of the lesser metatarsal heads
4. Increased plantar pressure at the first metatarsal head
5. Increased recruitment of the short hallucal flexors

DISCUSSION: The only significant change occurring after transfer of the flexor hallucis
longus for chronic Achilles ruptures and Achilles tendinosis findings was decreased
hallucal phalangeal pressure. No transfer metatarsalgia or increased pressure under the
first metatarsal head was found. AOFAS MTP-IP scores remained high and averaged 96.4
out of 100. PREFERRED RESPONSE: 2

105
48
2012- -Foot and Ankle --Self-Assessment Examination by Dr.Dhahirortho
Question 77 Following surgery for an ankle fracture, which of the following is
considered the most important factor in achieving a satisfactory outcome?
1. Physical therapy
2. Early weight bearing
3. Anatomic alignment
4. Early range of motion of the ankle
5. Calcium and vitamin D administration
DISCUSSION: The only factor that is prognostic for outcomes is the quality of the
reduction. None of the other factors has any effect on the outcome. Early range of
motion or physical therapy may offer temporary effects, but these small advantages do
not last beyond 3 months after surgery. PREFERRED RESPONSE: 3

Question 78 Recurrence of hallux valgus deformity after corrective surgery has been
shown to be related to which of the following?
1. Inversely correlated with presence of bipartite fibular sesamoid
2. Associated with residual increased tibial sesamoid displacement
3. Associated with squared lateral first metatarsal shape
4. Unrelated to preoperative 1-2 intermetatarsal angle
5. Unrelated to preoperative hallux valgus angle
DISCUSSION: Okuda and associates have studied the factors associated with the
recurrence of hallux valgus deformity following correction with proximal first metatarsal
osteotomy. The factors that they found associated with recurrence of deformity
postoperatively are a rounded shape to the lateral first metatarsal head, severe lateral
displacement of the tibial sesamoid, an increased preoperative 1-2 intermetatarsal angle,
and an increased preoperative hallux valgus angle. The reported rate of recurrence of
deformity after proximal first metatarsal osteotomy is 4% to 11%. PREFERRED RESPO: 2
Question 79 A 33-year-old woman has had plantar first metatarsophalangeal joint pain
for 3 years. Examination reveals that she is tender under the medial sesamoid. She has
no swelling or ecchymosis. The first metatarsophalangeal joint motion is equal and
stable bilaterally. Radiographically, there is some fragmentation of the medial
sesamoid with increased density in some of the fragments. Various orthotic and shoe
modifications have failed to provide relief. What is the most appropriate management?
1. First metatarsophalangeal joint arthrodesis
2. Open reduction and internal fixation of the medial sesamoid
3. Medial sesamoid excision
4. Shoe wear and heel heights modifications
5. Corticosteroid injection of the first metatarsophalangeal joint
106
49
2012- -Foot and Ankle --Self-Assessment Examination by Dr.Dhahirortho
DISCUSSION: Medial sesamoid stress fractures with fragmentation that have not
responded to nonsurgical management have done well after sesamoid excision. Excision
of both sesamoids is not recommended. A first metatarsophalangeal joint arthrodesis is
not going to resolve the issue of weight loading on the stress fractured, fragmented
sesamoid. Open reduction can be considered if there are two large fragments without
osteonecrosis. Corticosteroid injection is not going to provide long-term relief.Shoe
modifications have already been tried, without relief. PREFERRED RESPONSE: 3

Question 80 What is the most common organism in osteomyelitis of the foot that
results from a puncture wound in a non-diabetic patient?
1. Proteus
2. Clostridia
3. Pseudomonas
4. Streptococcus
5. Staphylococcus aureus

DISCUSSION: Although Staphylococcus aureus is the most common causative agent for
soft-tissue infection following puncture wounds of the foot, pseudomonas is the most
common organism found when osteomyelitis occurs. PREFERRED RESPONSE: 3

Question 81 A patient underwent an open reduction and internal fixation of a


calcaneus fracture 6 months ago via an extensile lateral approach. He now reports
burning pain on the lateral side of his ankle and foot. A local cortisone injection at the
site of the tenderness, about 7 cm above the lateral heel, provided temporary relief of
the pain. What is the recommended course of management for the persistent burning
pain?
1. Subtalar fusion
2. Neuroplasty of the superficial peroneal nerve
3. Neuroplasty of the sural nerve and implant removal
4. Excision and burial of the sural nerve in deep muscle or vein
5. Electromyography/nerve conduction velocity studies to evaluate local nerve
entrapment versus radiculopathy

DISCUSSION: The patient has a sural nerve neuroma, which is a known complication of
the extensile lateral approach. Of the available choices, excision and burial of the sural
nerve in muscle or vein is the best choice because it gives better pain relief due to the
better blood supply in muscle than bone. Recent authors advocate burying the nerve in
107
50
2012- -Foot and Ankle --Self-Assessment Examination by Dr.Dhahirortho
vein as the best option. Neuroplasty is a possibility (but not of the superficial peroneal
nerve), but the sural nerve is usually very sensitive and often pain relief with a release is
incomplete. Additionally, implant removal is not indicated because of the patient’s
complaints; also,the implants should not be removed at 6 months. A subtalar fusion is the
choice for posttraumatic arthritis from the calcaneus fracture. Electromyography/nerve
conduction velocity studies are reasonable choices if there was an indication the pain
could be coming from the back or there was no clear evidence of a sural nerve neuroma.
PREFERRED RESPONSE: 4
Question 82 Many incisions around the foot and ankle are associated with potential
nerve problems. Which of the following is the most appropriate pairing of surgical
incision and the likely nerve injury?
1. Gastroc recession-injury to the sural nerve
2. Anterior total ankle approach-injury to the tibial nerve
3. Tarsal tunnel release-injury to the deep peroneal nerve
4. Peroneal tendon repair-injury to the superficial peroneal nerve (SPN)
5. Open reduction and internal fixation of the calcaneus-injury to the lateral plantar
nerve

DISCUSSION: A gastroc incision is commonly posterior in the back of the calf and is based
over the sural nerve. All of the other choices are incorrect pairings of incision with the
nerve that can be injured with the approach. The anterior ankle approach can injure the
medial branch of the SPN as well as the deep peroneal nerve. A peroneal tendon repair
uses a posterolateral incision that directly exposes the sural nerve, not the SPN. The
calcaneus incision typically would injure the sural nerve. A tarsal tunnel release is via a
posteromedial approach that can injure the tibial nerve and its branches. PREFE RESP: 1

Question 83 A cavovarus foot reconstruction is planned. Which of the following tendon


transfers will decrease the plantar flexion forces being applied to the first metatarsal
head?
1. Split anterior tibial tendon transfer
2. Peroneal longus to peroneal brevis
3. Flexor digitorum to posterior tibial tendon
4. Flexor digitorum longus to extensor digitorum longus
5. Posterior tibial tendon transfer through the interosseous to the dorsal lateral
cuneiform

108
51
2012- -Foot and Ankle --Self-Assessment Examination by Dr.Dhahirortho
DISCUSSION: Cavus results from muscle imbalances in both the intrinsic and extrinsic
groups.Weakness of the anterior tibialis with strong peroneal longus muscle tone is
believed to be one of the factors causing a plantar flexed first metatarsal. The flexor
digitorum longus to posterior tibial tendon transfer is used for posterior tibial tendon
dysfunction. Posterior tibial tendon transfer to the dorsal foot is used to help correct
weak dorsiflexion. The split anterior tibial tendon transfer is used to help correct
equinovarus deformities or excessive forefoot inversion during the swing phase. Flexor
digitorum longus to extensor digitorum longus transfers are used for correction of flexible
hammer or claw toes.

PREFERRED RESPONSE: 2

Question 84 A tall, thin 17-year-old basketball player and his parents request an
evaluation of his flexible (hypermobile) pes planus/planovalgus foot deformities. As
part of his evaluation, the orthopaedic surgeon notes pectus excavatum,
disproportionately long arms, and scoliosis. In addition to providing treatment of his
feet, what test or evaluation should the patient be referred for?

1. Cardiovascular evaluation
2. Ophthalmologic evaluation
3. MRI of the spine
4. Radiographs of the hip
5. Genetic testing

DISCUSSION: The current diagnostic criteria for Marfan syndrome, called the Ghent
criteria, are based on clinical findings and family history. The role of genetic testing in
establishing the diagnosis is limited,because testing for FBN1 mutations is neither
sensitive nor specific for Marfan syndrome. By making the diagnosis and arranging for
cardiovascular evaluation, the orthopaedic surgeon can help prevent sudden death in
these patients. The cardiovascular manifestations, including dissection and dilation of
the ascending aorta and mitral valve prolapse, are responsible for nearly all of the
precocious deaths of patients with Marfan syndrome. Patients with Marfan syndrome do
have problems with protrusion acetabuli, scoliosis, and opthalmologic problems but the
life-threatening problem that must be considered is the risk of cardiovascular sudden
death.
PREFERRED RESPONSE: 1

109
52
2012- -Foot and Ankle --Self-Assessment Examination by Dr.Dhahirortho
Question 85 A 35-year-old man sustained a Lisfranc dislocation 2 years ago. He was
treated with standard open reduction and fixation. At 4 months, the screws were
removed. He now has increasing pain and discomfort. A current radiograph is shown in
Figure 85. What is the best treatment option?

1. Reduction and fusion of the medial three


tarsometatarsal (TMT) joints
2. Reduction and fusion of all five tarsometatarsal
joints
3. Revision open reduction and internal fixation
with bridge plates to avoid further damage to the
joints
4. Revision open reduction and internal fixation but
leave the screws in indefinitely
5. Soft-tissue interpositional grafts for the
tarsometatarsal joints

DISCUSSION: The most reliable treatment will be a reduction and fusion of the medial
three TMT joints.There is adequate proof in the literature that fusion of all five TMT joints
should be avoided because the fusion rate as well as functional outcome is inferior with
fusion of all five joints compared with fusion of the medial three TMT joints and
preservation of mobility in the 4-5 TMT joints. There is too much deformity and arthritis
to warrant a revision open reduction and internal fixation. An interpositional graft is not
proven to help in this situation because it neither corrects the deformity nor aids in
stability. PREFERRED RESPONSE: 1
Question 86Figures 86a and 86b are the AP and lateral radiographs of an active, healthy
60-year-old man who has had a 1-year history of swelling and pain in the right foot. He
denies any history of trauma. Nonsteroidal anti-inflammatory drugs and an orthosis
have failed to provide relief of his symptoms. What is the most appropriate
treatment?

1. Triple arthrodesis
2. Subtalar arthrodesis
3. Talonaviculocuneiform arthrodesis
4. Pantalar arthrodesis with calcaneal
osteotomy
5. Calcaneal osteotomy and dorsiflexion osteotomy of the first metatarsal
110
53
2012- -Foot and Ankle --Self-Assessment Examination by Dr.Dhahirortho
DISCUSSION: The patient has advanced necrosis of the navicular. Whereas the
pathogenesis is not completely known, some consider it to be related to Kohler disease.
The radiographs are distinctive with sclerosis and narrowing of the navicular, particularly
the lateral aspect. When nonsurgical management with semi-rigid insoles fails, a
talonaviculocuneiform fusion with bone grafting is preferred because it can address
incongruity on both sides of the navicular as well as restore length lost with bony
fragmentation.The other options do not address the naviculocuneiform joint. PRE RESP: 3

Question 87Figures 87a and 87b are the radiographs and MRI scan of a 17-year-old
cross country runner who reports pain in his forefoot around the third and fourth
metatarsals. The pain is mostly on top of the foot and appears to be activity related.
There is minimal swelling on examination and diffuse tenderness over the third and
fourth metatarsal shafts. What is the most appropriate management?

1. Three-phase bone scan


2. Bone density examination
3. Non-weight-bearing short-leg cast
4. Fracture boot with weight bearing as tolerated
5. Limit his miles and repeat radiographs in 2 weeks

DISCUSSION: The most appropriate management is a fracture boot with weight bearing
as tolerated.The radiographs are normal, but the history strongly suggests a metatarsal
stress fracture. The MRI scan clearly shows edema of the third metatarsal which is
consistent with a stress fracture. Patients can be treated successfully with weight-bearing
111
54
2012- -Foot and Ankle --Self-Assessment Examination by Dr.Dhahirortho
immobilization and activity modification. Some physicians treat metatarsal base fractures
with limited weight bearing. Limiting his miles and repeat radiograph in 2 weeks would be
an acceptable option if the MRI scan showing the fracture had not been obtained. A bone
density examination typically will be normal in a teenager, and it does not help in
treatment. A bone scan is a good test to diagnose the fracture, but an MRI scan has
already been obtained. PREFERRED RESPONSE: 4

Question 88The peroneus brevis is the primary antagonist to which of the following
structures?
1. Anterior tibialis
2. Posterior tibialis
3. Peroneus longus
4. Flexor hallucis longus
5. Extensor hallucis longus

DISCUSSION: The primary function of the peroneus brevis is eversion of the foot, thus
acting as the primary antagonist of the posterior tibialis, which inverts the foot, and
secondarily plantar flexes the ankle.The anterior tibialis secondarily inverts the foot and
only acts as a partial antagonist of the posterior tibialis. The peroneus longus plantar
flexes the first ray. PREFERRED RESPONSE: 2

Question 89Lisfranc’s ligament connects which of the following structures?


1. Base of the first metatarsal to the base of the second metatarsal
2. Base of the first metatarsal to the middle cuneiform
3. Base of the second metatarsal to the middle cuneiform
4. Base of the second metatarsal to the medial cuneiform
5. Medial cuneiform to the middle cuneiform

DISCUSSION: The stability of the Lisfranc joint complex results from bony and
ligamentous contributions. The metatarsal cuneiform articulations form a Roman arch
configuration. The second metatarsal is recessed between the medial and lateral
cuneiforms, adding more stability. Strong intermetatarsal ligaments are present between
each of the lateral four metatarsals but are absent between the first and second
metatarsals. In this region, the base of the second metatarsal is joined to the medial
cuneiform by Lisfranc’s ligament. PREFERRED RESPONSE: 4

112
55
2012- -Foot and Ankle --Self-Assessment Examination by Dr.Dhahirortho
Question 90 A football player who injured his right lower extremity during a game
could not get up and reported extreme pain. The initial sideline evaluation showed a
probable anterior cruciate, posterior cruciate,and lateral collateral ligament rupture
with a very unstable knee. He also reports pain in his ankle and is unable to dorsiflex
the ankle. He has limited sensation over the dorsum of his foot. Examination reveals no
swelling of the ankle and no pain with passive range of motion of the ankle. What is the
most likely diagnosis?
1. Tibial nerve injury
2. Associated ankle fracture
3. Acute compartment syndrome
4. Injury to the common peroneal nerve
5. Rupture of the tibialis anterior tendon

DISCUSSION: It is not uncommon to sustain a peroneal nerve injury in association with a


knee dislocation or multi-ligament injury. There should always be a high index of
suspicion for this injury, and the vascular status to the leg should be carefully evaluated.
From the history and examination, there is no indication that the ankle was fractured. A
compartment syndrome will not develop within a few minutes of the injury. It takes
several hours for a compartment syndrome to develop and become symptomatic.The
tibial nerve supplies the plantar aspect of the foot. An acute rupture of the tibialis
anterior tendon in a young person is very uncommon, and it is associated with pain and
localized swelling about the ankle. It is also unlikely that it would lead to sensory loss.
PREFERRED RESPONSE: 4
Question 91 Figures 91a through 91c are the radiographs of a 10-year-old boy who has
a 6-month history of progressive heel pain. The patient is a year-round soccer player
and now experiences pain with most every step. What is the most appropriate
management?

1. MRI
2. Custom orthotics
3. Activity modification
4. Calcaneal epiphysiodesis
5. Percutaneous Achilles
tendon lengthening

113
56
2012- -Foot and Ankle --Self-Assessment Examination by Dr.Dhahirortho
DISCUSSION: The patient has calcaneal apophysitis, an overuse syndrome common in
children ages 9 to12 years. Symptoms are usually the result of excess tension and a tight
heel cord. Management includes activity modification, as well as heel cord stretching,
nonsteroidal anti-inflammatory drugs, icing, and other modalities. Radiographs are
typically negative; MRI is unnecessary. Custom orthotics are not indicated. The condition
is self-limiting, in that the symptoms fully resolve once the apophyses fuses,such that
surgery is rarely indicated. PREFERRED RESPONSE: 3

Question 92 Figures 92a through 92c are the clinical photographs and radiograph of a
22-year-old man who has had a 6-month history of lateral ankle pain following minor
ankle trauma. He has undergone physical therapy,which only made it more
symptomatic. What is the most appropriate management?

1. Arthrodesis
2. Bar resection
3. Immobilization
4. Corrective osteotomy
5. Injection of the peroneal tendons

DISCUSSION: The clinical photographs show a rigid pes planus, with the radiograph
showing a calcaneonavicular coalition. Most patients will have symptomatic
improvement with a short course of immobilization. Only after that has failed would
surgery be contemplated. Although the pain may be along the peroneal tendons,
injecting them will not yield long-term relief because their symptoms are only secondary
to the tarsal coalition. PREFERRED RESPONSE: 3

114
57
2012- -Foot and Ankle --Self-Assessment Examination by Dr.Dhahirortho
Question 93 A 50-year-old woman with a mild flexible planovalgus foot deformity has
lateral hindfoot pain. What is the simplest modification of her shoe wear to help
offload the lateral hindfoot?
1. Medial hindfoot posting
2. Lateral hindfoot posting
3. Rigid foot orthotic
4. Semi-rigid foot orthotic
5. Accommodative foot orthotic
DISCUSSION: Lateral hindfoot pain is often the result of impingement as the planovalgus
foot pronates and abducts. Medial hindfoot posting or wedge will elevate the medial
aspect of the heel and decrease the lateral hindfoot pressures. This may be done in a
dress shoe as well as a tie shoe with a simple heel posting wedge. Adding lateral hindfoot
posting would increase the lateral pressures. An orthotic, whether rigid, semi-rigid, or
accommodative, will support the arch but without medial hindfoot posting, the foot will
often still pronate and abduct with continued lateral hindfoot pressure. PREFE RESP: 1
Question 94 Figures 94a through 94d are the weight-bearing radiographs of a 45-year-
old man who is an avid tennis player and has intermittent lateral midfoot pain. He has
sustained three ankle sprains in the past 10 years and has occasional sensations of
instability. Examination includes mild laxity with lateral ankle ligament testing, normal
ankle and hindfoot motion, a supple forefoot, and no ankle joint line pain. What is the
most appropriate management?

1. MRI
2. Boot immobilization
3. Nonsteroidal anti-inflammatory drugs (NSAIDs) and lace-up bracing
4. Physical therapy and an orthotic with mild arch support and 5° medial posting
5. Physical therapy and an orthotic with lateral forefoot posting and first metatarsal
head recessing
115
58
2012- -Foot and Ankle --Self-Assessment Examination by Dr.Dhahirortho
DISCUSSION: The patient has lateral column overload and functional ankle instability due
in part to the subtle cavus foot. Because of the cavus posture and mild ankle instability,
physical therapy and a cavus foot orthotic are indicated. Although MRI can be of benefit
in the diagnosis of osteochondral lesions of the talus, it is of little value in this patient
based on the absence of ankle joint line symptoms.NSAIDs and lace-up bracing can be
helpful with regard to the ankle, but do not address the lateral column overload. Orthotic
management with arch support and medial posting is contraindicated with a cavus foot
deformity. Temporary immobilization would be indicated in the event of a suspected
stress fracture, in which case the lateral midfoot symptoms would be more constant in
nature. PREFERRED RESPONSE: 5

Question 95 A 49-year-old woman underwent a successful right ankle fusion. She now
reports an altered gait. In an attempt to improve her gait, what is the most appropriate
device?
1. Arizona brace
2. Rocker-bottom sole
3. Double upright drop-lock brace
4. Non-articulated ankle-foot orthosis
5. Carbon fiber insert with a Morton’s extension
DISCUSSION: The plantar flexion-dorsiflexion motion of the ankle can be partially
mimicked with a rocker-bottom shoe adaptation. A carbon fiber insert would not provide
any improvement in her gait. An ankle-foot orthosis, Arizona brace, and double upright
drop-lock brace would immobilize the ankle, which is already achieved with the ankle
fusion. PREFERRED RESPONSE: 2

Question 96 A 31-year-old woman underwent a left Kidner procedure 3 months ago.


She now has pain overlying the medial column of the foot. She withdraws the foot
when touching of the medial foot is attempted.Examination reveals allodynia, pain,
hyperalgesia, and edema of the medial foot. What is the most likely diagnosis?
1. Shingles
2. Cellulitis
3. Charcot foot
4. Osteomyelitis
5. Reflex sympathetic dystrophy

116
59
2012- -Foot and Ankle --Self-Assessment Examination by Dr.Dhahirortho
DISCUSSION: Patients with reflex sympathetic dystrophy (RSD) have a history of trauma,
minor rather than major (eg, Colles fracture), in about 50% to 65% of cases. The condition
may also follow a surgical procedure. Patients usually have symptoms and signs of RSD
including: pain, described as burning, throbbing, shooting, or aching; hyperalgesia;
allodynia; and hyperpathia. There are trophic changes within 10 days of onset of RSD in
30% of the extremities affected, including stiffness and edema and atrophy of hair, nails,
and/or skin. Finally there can be autonomic dysfunction, such as abnormal sweating,
either in excess or anhydrosis, heat and cold insensitivity, or redness or bluish
discoloration of the extremities.Shingles, also called herpes zoster or zoster, is a painful
skin rash caused by the varicella zoster virus (VZV). VZV is the same virus that causes
chickenpox. After a person recovers from chickenpox, the virus stays in the body. Usually
the virus does not cause any problems; however, the virus can reappear years later,
causing shingles. Charcot arthropathy is a progressive condition of the musculoskeletal
system that is characterized by joint dislocations, pathologic fractures, and debilitating
deformities. This disorder results in progressive destruction of bone and soft tissues at
weight-bearing joints; in its most severe form,it may cause significant disruption of the
bony architecture. In patients with diabetes, the incidence of acute Charcot arthropathy
of the foot and ankle ranges from 0.15% to 2.5%. Acute Charcot arthropathy almost
always appears with signs of inflammation. Profound unilateral swelling, an increase in
local skin temperature (generally, an increase of 3° to 7° above the nonaffected foot’s
skin temperature), erythema,joint effusion, and bone resorption in an insensate foot are
present. These characteristics, in the presence of intact skin and a loss of protective
sensation, are often pathognomonic of acute Charcot arthropathy.Cellulitis is an infection
of the skin. Examination would reveal erythema, edema, and pain. Osteomyelitis is an
infection of the bone. Examination may reveal edema, drainage, and pain. PREF RESP: 5

Question 97 With respect to the clinical photograph shown in Figure 97, what artery
provides the most blood supply to the area of the planned incision?

1. Lateral tarsal
2. Lateral calcaneal
3. Lateral malleolar
4. Common peroneal
5. Artery of the tarsal sinus

117
60
2012- -Foot and Ankle --Self-Assessment Examination by Dr.Dhahirortho
DISCUSSION: The photograph shows the planned incision for an extensile lateral
approach to the calcaneus. The lateral calcaneal artery, usually a branch of the peroneal
artery, provides blood supply to the majority of the full-thickness flap of an extensile
lateral approach. Other branches also contribute,including the lateral malleolar and
lateral tarsal arteries, although to a lesser degree.

PREFERRED RESPONSE: 2

Question 98 A 39-year-old woman sustains a grade III inversion ankle sprain and is
treated with boot immobilization,crutches, and physical therapy, through her primary
care physician. The patient is referred for evaluation 3 weeks later because of
persistent pain and inability to bear weight. Examination reveals mild residual swelling
and exquisite tenderness to light touch overlying the anterolateral ankle and
dorsolateral foot.Passive range of motion also reproduces extreme pain. Radiographs
are negative for fracture and an MRI scan shows a tear of the anterior talofibular
ligament. What is the most appropriate management?

1. Cast immobilization
2. Anticonvulsant therapy
3. Brostrom ligament reconstruction
4. Ankle arthroscopy with débridement
5. Proprioceptive physical therapy and a lace-up ankle brace

DISCUSSION: The patient has most likely developed a stretch neurapraxia of the
superficial peroneal nerve. Localized pain, particularly to light touch which has not
improved after 3 weeks (in this instance) raises the likelihood of a neurologic etiology.
Anticonvulsant medication, such as gabapentin or pregabalin, can be invaluable in
dissipating and often resolving neuritic symptoms. Antidepressant medication, such as
amitriptyline, could alternatively be used. Proprioceptive physical therapy is likely to
worsen the neuritic symptoms, but may be indicated once the neuritic symptoms
subside. Cast immobilization can be used acutely in promoting dissipation of swelling and
offering soft-tissue support;however, further immobilization in this instance is likely to be
of limited benefit. Surgical intervention is not indicated in the acute management of
ankle sprains.

PREFERRED RESPONSE: 2

118
61
2012- -Foot and Ankle --Self-Assessment Examination by Dr.Dhahirortho
Question 99 Which of the following factors predisposes patients undergoing lateral
ankle reconstruction to surgical failure?
1. Low functional demand
2. Anatomic reconstruction
3. Cavovarus foot deformity
4. Planovalgus foot deformity
5. History of acute instability (preoperative)

DISCUSSION: Bony malalignment such as hindfoot varus, first ray plantar flexion, or
cavovarus foot deformity renders patients more susceptible to chronic lateral ankle
sprains and can cause failure of isolated lateral ankle reconstruction. Planovalgus foot
deformity principally strains the medial ankle ligamentous structures. Low functional
demand patients, as well as patients with a preoperative history of acute traumatic ankle
instability and patients undergoing anatomic reconstruction all have decreased rates of
surgical failure with lateral ankle ligament reconstruction. PREFERRED RESPONSE: 3

Question 100 A 63-year-old man has long-standing type 2 diabetes. He has had
associated ongoing neuropathy for approximately 5 years. He now reports a red, hot,
swollen right foot for the past 2 days. You place him supine in your office with the foot
elevated for 30 minutes. You return to see that the redness has dissipated. What is the
most likely diagnosis?
1. Cellulitis
2. Osteomyelitis
3. Charcot arthropathy
4. Sjogren syndrome
5. Reynaud syndrome

DISCUSSION: Charcot arthropathy is a progressive condition of the musculoskeletal


system that is characterized by joint dislocations, pathologic fractures, and debilitating
deformities. This disorder results in progressive destruction of bone and soft tissues at
weight-bearing joints; in its most severe form, it may cause significant disruption of the
bony architecture. In patients with diabetes, the incidence of acute Charcot arthropathy
of the foot and ankle ranges from 0.15% to 2.5%. Acute Charcot arthropathy almost
always appears with signs of inflammation. Profound unilateral swelling, an increase in
local skin temperature (generally, an increase of 3° to 7° above the nonaffected foot’s
skin temperature), erythema,joint effusion, and bone resorption in an insensate foot are
present. These characteristics, in the presence of intact skin and a loss of protective
119
62
2012- -Foot and Ankle --Self-Assessment Examination by Dr.Dhahirortho
sensation, are often pathognomonic of acute Charcot arthropathy.Cellulitis is an infection
of the skin and osteomyelitis is an infection of the bone; examination of the limb would
remain unchanged after 30 minutes of elevation. Reynaud and Sjogren syndromes are
rheumatologic conditions that do not have these symptoms. PREFERRED RESPONSE: 3

CLINICAL SITUATION FOR QUESTIONS 101 THROUGH 103


A 27-year-old woman has had pain in her right ankle for 2 years. Examination reveals a
slightly warmjoint, without erythema. Ankle range of motion is limited by pain.
Radiographs are unremarkable.Because management consisting of immobilization,
nonsteroidal anti-inflammatory drugs, and physiotherapy has failed to provide relief,
MRI scans are obtained and shown in Figures 101a and 101b.An intraoperative image
and the histology are shown in Figures 101c and 101d.

Question 101 What is the best descriptor for the lesion shown in Figures 101a and
101b?
1. Normal anatomic variant
2. Inflammatory
3. Dedifferentiated malignancy
4. Well-differentiated malignancy
5. Locally aggressive
PREFERRED RESPONSE: 5

Question 102The orthopaedic surgeon treats the lesion arthroscopically. An


intraoperative image is shown in Figure101c. What is the most appropriate immediate
treatment?
1. Careful removal of the instrumentation without disrupting the lesion
2. Obtain a tissue biopsy, removal of the instrumentation, and wound closure
3. Obtain a tissue biopsy and perform an extensive arthroscopic débridement
4. Place a supramalleolar tourniquet to prevent metastasis of tumor cells
5. Transilluminate the lesion with the arthroscope and place sutures in the overlying
skin forfuture brachytherapy catheter placement
PREFERRED RESPONSE: 3
120
63
2012- -Foot and Ankle --Self-Assessment Examination by Dr.Dhahirortho
Question 103 The histology of the lesion is shown in Figure 101d. What is the most
likely complication after treatment of this lesion?
1. Arthrofibrosis
2. Local recurrence
3. Chondrolysis
4. Disseminated infection
5. Metastasis
PREFERRED RESPONSE: 2

DISCUSSION FOR QUESTIONS 101 THROUGH 103:


The MRI scans show a well-circumscribed inhomogenous mass at the anterior ankle joint
consistent with a diagnosis of pigmented villonodular synovitis (PVNS), not an anatomic
variant. An inflammatory mass,such as the pannus of rheumatoid arthritis, would reveal
moderate signal intensity with relatively uniform signal throughout. The mass is well
encapsulated, respecting tissue boundaries and not showing invasive characteristics as
malignancies would. The arthroscopic image is also consistent with the reddish-brown
frond-like tissue of PVNS. PVNS of the ankle can be treated arthroscopically. If this mode
of treatment is chosen, a tissue sample should be sent to pathology for microscopic
analysis; débridement should be performed on the remaining tissue. Removal of the
instrumentation without débridement will not provide treatment and will result in
disease progression. Tourniquet placement does not provide therapeutic benefit.
Brachytherapy has not been described for the treatment of PVNS of the ankle. The
histologic image shows multiple multinucleated giant cells, hemosiderin, and very few
mitotic figures – consistent with a diagnosis of PVNS. A common complication of PVNS
treatment is local recurrence. Arthrofibrosis and chondrolysis are not seen with proper
surgical care of these patients. Because this is not an infectious lesion, disseminated
infection after treatment is highly unlikely. PVNS rarely metastasizes.

RESPONSES FOR QUESTIONS 104 THROUGH 106


1. Toe is fused too straight (plantar flexed)
2. Toe is fused in too much valgus
3. Toe is fused in too much dorsiflexion
4. There is a nonunion of the fusion
5. Excessive shortening of the first metatarsal during preparation for fusion
What is the most likely diagnosis for each patient?

121
64
2012- -Foot and Ankle --Self-Assessment Examination by Dr.Dhahirortho
Question 104 A 32-year-old woman has had increasing pain in her great toe when she
tries to run and finds it impossible to wear shoes with a heel. She has noticed an
increasing callous build-up on the lateral border of her foot.She is pain-free with flat
shoes and can walk without pain when walking slow.
PREFERRED RESPONSE: 1

Question 105 A man reports a painful soft corn on the medial side of the second toe,
and a corresponding callus on the lateral side of the great toe. He reports some relief if
he places a soft spacer between the toes. Thesymptoms are not aggravated or relieved
by any type of footwear.

PREFERRED RESPONSE: 2

Question 106 A woman who underwent fusion of her great toe 15 months ago now is
unhappy with the result. The foot is asymptomatic in the morning but by evening is
more swollen and painful, especially if she is very active. She reports increased pain
when wearing shoes with heels and feels best in flat, stiff-soled shoes.As she increases
her activity level, the symptoms are getting worse rather than better.

PREFERRED RESPONSE: 4
DISCUSSION FOR QUESTIONS 104 THROUGH 106:
If the toe is fused too straight, the patient cannot roll over the forefoot, thus making it
nearly impossible to run with a normal gait. One way to compensate is to “avoid” the
medial side of the foot by walking on the outside border. Completely flat shoes, especially
shoes with a stiff sole, are the most comfortable. If the toe is fused in too much valgus,
there is excessive pressure on the second toe. This can cause a soft corn or even
ulceration between the great and second toe. With a nonunion of a fusion, there is
usually a slow build-up of scar tissue that makes the joint look bigger over time. In
addition, it is fairly typical for a nonunion to feel reasonable after rest but become painful
and swollen with activity. The normal postsurgical swelling and pain should be completely
resolved after 1 year. Stiff-soled shoes limit motion across the joint and usually provide
symptomatic relief.

122
65
2012- -Foot and Ankle --Self-Assessment Examination by Dr.Dhahirortho
CLINICAL SITUATION FOR QUESTIONS 107 AND 108

Figures 107a and 107b are the coronal and axial CT scans of a 19-year-old woman who
sustained a twisting ankle injury that was previously diagnosed as an ankle sprain. She
reports a feeling of giving out in the ankle and continues to have pain despite rest and
immobilization. She denies any history of problems with the ankle.

Question 107What structure attaches at the fracture site shown in Figures 107a and
107b?
1. Anterior talofibular ligament (ATFL)
2. Anterior inferior tibiofibular ligament (AITFL)
3. Deltoid ligament
4. Calcaneofibular ligament (CFL)
5. Superficial peroneal retinaculum (SPR)
PREFERRED RESPONSE: 2

Question 108 Because of the ongoing pain and instability and the demonstration of
radiographic instability when the ankle is stressed, what surgical procedure should be
performed to restore stability to the ankle joint based on the CT findings?
1. Brostrom procedure
2. Syndesmosis repair or stabilization
3. Allograft lateral ligament reconstruction
4. Excision of loose body/fracture fragment
5. Repair of the SPR with possible fibular groove
123 deepening
66
2012- -Foot and Ankle --Self-Assessment Examination by Dr.Dhahirortho
PREFERRED RESPONSE: 2

DISCUSSION FOR QUESTIONS 107 AND 108:


The fracture at the insertion of the AITFL into the fibula represents a syndesmosis injury.
In some cases, a direct repair of the fracture will stabilize the syndesmosis, but in most
cases this injury should most likely be reinforced by placing a screw or suture tensioning
device across the syndesmosis for additional support.A Brostrom or allograft
reconstruction is indicated for an ankle sprain involving the ATFL or CFL.Simply excising
the fragment will leave the patient with an incompetent syndesmosis. Repairing the SPR
with or without a groove deepening procedure is indicated if there is evidence of
subluxated or dislocated peroneal tendons, which is not demonstrated on the CT scans.
The bone has been avulsed off the fibula by the portion of the AITFL that attaches to the
fibula, therefore indicating that there is a syndesmosis injury. Allograft lateral ligament
reconstruction and excision of loose body/fracture fragment are incorrect procedures
based on location. The deltoid is a medial structure and this fracture is lateral. The ATFL
and CFL attach at the inferior margin of the fibula near the lateral process of the talus and
calcaneus. A SPR avulsion would present as an avulsion off the lateral wall of the fibula,
not superior and not into the syndesmotic space as shown on the CT scans.

124
67
1

125
2

126
Question 1A 56-year-old man has a chief complaint of leg weakness and inability to
walk. Examination reveals 5out of 5 motor strength in all lower extremity muscle
groups tested and normal sensation to light touch in
both lower extremities. The patient is slow in getting up from a seated position and has
an unsteady widebased gait. An MRI scan of the lumbar spine is shown in Figure 1.
What is the next most appropriate course of action?
1. Electromyography and nerve conduction velocity studies of
bilateral lower extremities
2. Multilevel lumbar laminectomy
3. MRI of the thoracic and cervical spine
4. MRI of the brain
5. Epidural steroid injections
DISCUSSION: The patient is having gait problems suspicious for spinal
cord compression. MRI of the thoracic and cervical spine should be
performed to evaluate for spinal cord compression. Reports of leg
weakness in the absence of discrete motor weakness on manual testing, and the
appearance of an unsteady wide-based gait are more consistent with myelopathy as a
cause of the gait difficulty rather than lumbar stenosis. Although the MRI scan of the
lumbar spine shows multilevel spinal stenosis that is mild to moderate, it does not clearly
explain the patient’s signs and symptoms. Electromyography and nerve conduction
velocity studies of the lower extremities are unlikely to add significantly to the
diagnosis.Epidural steroid injections are not indicated. Lumbar decompression is unlikely
to help the patient because the source of the patient’s problem does not originate in the
lumbar spine. MRI of the brain could be considered as a secondary imaging study if the
cervical and thoracic MRI scans fail to identify an obvious cause for gait instability.
PREFERRED RESPONSE: 3
Question 2 In the evaluation of somatosensory-evoked potential waveforms for
intraoperative neuromonitoring for spinal surgery, the minimum criteria for
determining potentially significant changes include
1. 10% decrease in amplitude, 50% decrease in latency.
2. 10% decrease in amplitude, 50% increase in latency.
3. 0% loss of amplitude, transient increase in latency.
4. 50% decrease in amplitude, 10% increase in latency.
5. 50% decrease in amplitude, 10% decrease in latency.
3

127
DISCUSSION: The established criteria for interpreting a significant change are 50%
decrease in signal amplitude, 10% latency increase, and/or a complete loss of potential.
Intraoperative spinal cord monitoring during spinal surgery generally consists of a
combination of monitoring modalities.Somatosensory-evoked potentials in combination
with intraoperative electromyography can provide adequate coverage of sensory and
motor components of spinal cord and nerve root function. Significant changes in evoked
potential waveform characteristics can reflect dysfunction of the ascending
somatosensory system.

PREFERRED RESPONSE: 4

Question 3 A 44-year-old man was involved in a low speed rear-end motor vehicle
accident 4 weeks ago. He predominantly reports pain in the back of the neck, with
occasional radiation into the trapezius region
bilaterally. He denies any extremity pain. The pain has not changed in intensity, but is
worse with neck range of motion. Cervical spine radiographs were negative for acute
osseous trauma or instability. What is the next most appropriate step in management?

1. Continued observation
2. Cervical epidural injections
3. Nonsteroidal anti-inflammatory drugs (NSAIDs), activity modification, and physical
therapy
4. Cervical facet blocks
5. Cervical MRI

DISCUSSION: The patient was involved in a low speed rear-end collision and sustained a
whiplashtype injury, with management most often being nonsurgical. After 4 weeks of
persistent pain, continued observation is not reasonable. Studies have shown that
treatment including NSAIDs, activity modification and a brief duration of physical therapy
allows for improved outcomes after whiplash-type injuries when compared with
observation alone. An MRI scan of the cervical spine is not indicated at this time and
represents an unnecessary expense. Cervical epidural and facet injections are not
indicated in the treatment of patients with whiplash injuries.

PREFERRED RESPONSE: 3

128
Question 4 A patient with a grade 2 L5-S1 isthmic spondylolisthesis reports low back
pain and bilateral lower extremity pain. Nonsurgical management has failed to provide
relief, and the patient is now a candidate for surgical intervention. The surgeon elects
to proceed with L5-S1 laminectomy and posterior instrumented fusion after reduction
of the spondylolisthesis. If a postoperative neurologic deficit develops, what structure
has most likely been affected?
1. L4 nerve root
2. L5 nerve root
3. S1 nerve root
4. Genitofemoral nerve
5. Cauda equina

DISCUSSION: The L5 nerve root is especially vulnerable and prone to injury after the
reduction of spondylolisthesis in patients with mid-and high-grade isthmic
spondylolisthesis. The genitofemoral nerve is more commonly injured during anterior
retroperitoneal approaches to the lumbar spine. Injury to the cauda equina often leads to
bowel and bladder dysfunction and lower extremity weakness and is uncommon after
reduction maneuvers.
PREFERRED RESPONSE: 2

Question 5Patients who are smokers and are undergoing spinal surgery have been
found to be at increased risk forinfection, pseudarthrosis, and reoperation. Smoking
has also been found to be associated with which ofthe following?
1. Longer hospital stay
2. Airway compromise following anterior cervical corpectomy
3. Lower patient satisfaction
4. Lower functional outcome scores
5. Wound hematoma

DISCUSSION: Smoking is associated with lower functional outcome scores and increased
risks for pseudarthrosis, postoperative wound infection, and reoperation rates. In a
prospective study evaluating the risks of airway compromise following anterior cervical
surgery, smoking was not a significant factor.Increased hospital days following spinal
surgery has not been independently associated with smoking.The increase risk for the
development of a wound hematoma has not been associated with smoking.
PR RE: 4
5

129
Question 6A 63-year-old man has a feeling of generalized clumsiness in his arms and
hands, difficulty buttoning his shirt, and gradually worsening gait instability. During
examination, his neck is gently passively flexed to end range while he is seated. The
patient describes an electric shock-like sensation that radiates down the
spine and into the extremities. This describes which of the following?
1. A positive Lhermitte sign
2. A positive Spurling sign
3. A positive Jackson sign
4. A positive Lasegue sign
5. A positive Hoffmann sign
DISCUSSION: What is now referred to as Lhermitte sign was first described by Marie and
Chatelin in 1917 to describe “transient ‘pins and needles’ sensations traveling the spine
and limbs on flexion of the head” in some patients with head injuries. A positive test is
indicated by the presence of electric-like sensation down the spine or extremities. It is
associated with cervical spinal cord pathology from a wide variety of etiologies, including
multiple sclerosis. Recent studies suggest that it has a low sensitivity and high specificity.
Spurling’s sign is elicited by axial compression of the tilted head. Jackson’s sign is elicited
by hyperextension of the cervical spine. Lasegue sign refers to the straight leg raise with
dorsiflexion of the ankle for lumbar radiculopathy. Hoffmann sign is a pathologic reflex of
the upper extremity, that is felt to be an indicator of cervical myelopathy. PR RES: 1
Question 7 Figures 7a through 7d show the radiographs and MRI scans of a 69-year-old
woman with neck and upper extremity pain and progressive deformity of the cervical
spine. What is the most likely diagnosis?

1. Postlaminectomy kyphosis
2. Ankylosing spondylitis
3. Occipitocervical dissociation
4. C3-4 pseudarthrosis
5. Klippel-Feil syndrome
6

130
DISCUSSION: Laminectomy without fusion for the treatment of cervical spondylotic
myelopathy currently plays a minor role in the management of this disorder because of
its many disadvantages. The actual incidence of postlaminectomy kyphosis is unknown,
but is estimated to be between 11% and 47%.It can result in recurrent myelopathy if the
spinal cord becomes draped over the kyphosis. In addition to the neurologic sequelae,
the kyphosis itself can be a source of neck pain and deformity. Spondylolisthesis can
develop, contributing to further cord compression. In this case, the patient had
undergone a previous C4-5 anterior cervical diskectomy and fusion followed by a
posterior laminectomy from C2 through C7, without fusion. This has resulted in severe
kyphosis (i.e. postlaminectomy kyphosis) with grade II-III spondylolisthesis at C3-4 and a
grade I spondylolisthesis at C2-3. While ankylosing spondylitis can also result in a chin-on-
chest deformity secondary to ankylosis, there is no evidence of marginal syndesmophytes
in the imaging studies to suggest this diagnosis. The occiput is hyperextended on C1 on
the lateral upright radiograph to compensate for the kyphosis in an attempt to maintain
horizontal gaze.This results in an unusual appearing relationship on the imaging studies.
However, there is no widening of the distance between C1 and the occiput and no
evidence of soft-tissue injury on the MRI scans to suggest an acute injury. C3-4
demonstrates an unstable spondylolisthesis and was never intended to be included in the
C4-5 fusion. Klippel-Feil syndrome is the failure of segmentation of the cervical spine. The
classic triad includes congenital fusion, low hairline, and a web neck. PRE RESP: 1

Question 8 A 46-year-old man has a recurrent disk herniation on the left side at L4-5
and has had persistent radicular symptoms for 3 months without neurologic deficit. His
previous surgery was performed 13 months ago.What is the best method of surgical
treatment?
1. Posterior lumbar interbody fusion
2. Anterior lumbar interbody fusion
3. Revision diskectomy
4. Artificial disk replacement
5. Interspinous process spacer

DISCUSSION: The patient is a candidate for a revision diskectomy. Outcomes of revision


diskectomy are nearly as good as those reported for primary diskectomy. Reduction in leg
pain is expected. An indication for fusion would be a concomitant instability at the
planned surgical level. Though controversial, artificial disk replacement might be
considered if there is a significant component of diskogenic pain without instability.
7

131
However, disk replacement is not the first choice of treatment in this case. Interspinous
process spacers are not currently indicated in this situation. PREFERRED RESPONSE: 3

Question 9Of the following, what is the most reliable method of assessing spinal
fusion?
1. Radiographs
2. MRI
3. Flexion/extension radiographs
4. CT
5. CT myelography
DISCUSSION: Despite the ease of attainment, radiographs only accurately diagnose failed
arthrodesis in 60% to 80% of uninstrumented cases and these numbers are even lower in
cases with posterior instrumentation. The role of dynamic radiographs remains unclear
because of the paucity of normative data values after lumbar spine fusion. CT scans
provide excellent bony detail and their images are not affected by metal components as
in MRI. Post-myelogram CT is useful for identifying neurologic compression. PREF RES: 4

Question 10 A 42-year-old woman reports neck stiffness, upper extremity pain,


clumsiness, weakness, and instability of gait. Examination reveals 4+ of 5 strength in
the upper extremities and 3+ biceps, brachioradialis, and patellar reflexes with a
positive Hoffman sign bilaterally. MRI and CT scans are shown in Figures 10a and
10b. Based on the history and imaging findings, what is the most likely diagnosis?

1. Diffuse idiopathic skeletal hyperostosis


2. Ankylosing spondylitis
3. Ossification of the posterior longitudinal ligament
4. Rheumatoid arthritis
5. Degenerative cervical stenosis

DISCUSSION: The sagittal T2-weighted MRI scan


shows moderate-severe multilevel cervical
stenosis.The cord compression is noted to be not only
at the disk levels but also at the midvertebral body levels,and the posterior longitudinal
ligament appears to be thickened. The CT scan confirms that the posterior longitudinal
ligament is indeed thickened and ossified, compatible with a diagnosis of ossification of
the posterior longitudinal ligament. This diagnosis is most common in individuals of
8

132
Japanese descent and has a genetic linkage. The anterior osteophytes are smaller than
those seen in diffuse idiopathic skeletal hyperostosis and are not syndesmotic. Patients
with ankylosing spondylitis typically have non-marginal syndesmophytes. Patients with
rheumatoid arthritis may have evidence of instability at C1-C2 on flexionextension
radiographs and subaxial subluxations. PREFERRED RESPONSE: 3

Question 11 An 18-year-old man who sustained a lumbar fracture-dislocation with an


associated complete spinal cord injury 6 weeks ago underwent instrumented posterior
thoracolumbar fusion a few days after the injury. While at a rehabilitation facility,
routine postoperative surveillance radiographs are obtained (Figures 11a
through 11d). What is the most appropriate next step in management?

1. Observation alone with continued rehabilitation of the spinal cord injury


2. Thoracolumbosacral orthosis bracing for 3 months
3. Revision and extension of the posterior instrumentation and fusion
4. Anterior lumbar corpectomy and fusion
5. Anterior/posterior lumbar decompression and fusion

DISCUSSION: The patient has sustained a traumatic spondylolisthesis at the level below
the caudal instrumented level, likely not appreciated at the index surgery. Surveillance
radiographs indicate that there is significant translation in the lumbar spine on sitting,
indicating an unstable injury. The lack of significant bony involvement indicates that the
injury is predominantly through the anterior and posterior ligamentous complexes, and
thus is unlikely to stabilize with nonsurgical management. Because the patient reduces
almost completely on lying supine, the most appropriate course of action is extension of
the posterior fusion to include the level of the instability. Because the patient has a
complete spinal cord injury below the level of the thoracic fracture, decompression is not
indicated. PREFERRED RESPONSE: 3
9

133
Question 12 An 80-year-old man with a history of chronic obstructive pulmonary
disease (COPD) and dementia is involved in a fall from standing height, striking his
forehead. He is seen in the emergency department with predominantly mechanical
neck pain but no obvious neurologic deficits. Radiographs reveal a nondisplaced type II
odontoid fracture. What is the most appropriate treatment?

1. Immobilization in a rigid cervical orthosis for 6 to 8 weeks


2. Posterior occipital-cervical fusion with iliac crest bone graft
3. Open reduction and internal fixation of the odontoid process with an anterior
odontoid screw
4. Resection of the odontoid process through a transoral approach
5. Halo skeletal fixation

DISCUSSION: The treatment options for a type II odontoid fracture include halo
immobilization,odontoid screw fixation, and posterior atlantoaxial arthrodesis. However,
surgical care at this time without attempting nonsurgical management is not warranted;
therefore, the most appropriate management at this time is immobilization in a rigid
cervical orthosis for 6 to 8 weeks. Halo vest fixation can lead to high healing rates but is
generally contraindicated in elderly patients, especially one with COPD and dementia.
Posterior surgical fusion techniques provide high fusion rates, but do so at the expense of
loss of cervical rotation and surgical complications. Resection of a nondisplaced odontoid
fracture without cord compression via a transoral approach is not necessary. PRE RES: 1

Question 13What is the predominant type of collagen found in the nucleus pulposus of
the intervertebral disk?
1. Type I
2. Type II
3. Type V
4. Type VI
5. Type XII
10

134
DISCUSSION: Types I and II collagen are the predominant types of collagen found in the
intervertebral disk. Type I collagen is present in the highest concentration in the annulus
fibrosus and type II collagen in the nucleus pulposus. Type V collagen is present in small
concentration in the annulus fibrosus. Type VI collagen is a non-fibrillar, short-helix
collagen found in both the annulus and nucleus. Type XII is present in the annulus
fibrosus only. PREFERRED RESPONSE: 2

Figure 14What clinical scenario is most consistent with the MR image of the L4-L5 disk
level shown in Figure 14?
1. Left L4 nerve root radiculopathy
2. Right L5 nerve root radiculopathy
3. Bowel and bladder dysfunction
4. Arachnoiditis
5. Symptomatic pseudomeningocele

DISCUSSION: The MRI scan reveals a foraminal disk herniation originating from the L4-L5
disk space that has migrated into the foramen compressing the left L4 nerve root. There
is no evidence of compression of the right L5 nerve root. Bowel and bladder dysfunction
are not associated with L4-mediated nerve function. There is no evidence of arachnoiditis
or pseudomeningocele. PREFERRED RESPONSE: 1

Question 15 A 23-year-old man is evaluated in the emergency department after a


diving accident. Radiographs reveal bilateral jumped facets at C6-7. Examination
reveals no motor function below the C7 level. There is some maintained sensation in
the lower extremities. What is the patient’s current grade on the ASIA (American Spinal
Injury Association) impairment scale?
1. ASIA A
2. ASIA B
3. ASIA C
4. ASIA D
5. ASIA E

DISCUSSION: The American Spinal Injury Association (ASIA) provides a standard method
of measurement of spinal cord injury. The ASIA impairment scale is based on a
11

135
comprehensive motor and sensory examination. An ASIA A grade is ascribed to a patient
with an injury with no motor or sensory preservation below the injury. An ASIA B grade is
defined as no motor preservation below the level of injury but some sensory preservation
below the injury level. An ASIA C grade is defined as a motor function grade of less than 3
below the injury level. An ASIA D grade is defined as a motor function grade of greater
than 3 below the injury level. An ASIA E grade is defined as a normal neurologic
examination. PREFERRED RESPONSE: 2

Question 16 A 56-year-old man has had a 2-year history of slowly progressive neck pain
and bilateral arm aching.Over the past year, he has noticed intermittent, diffuse
numbness in both hands, with decreased grip strength and mild hand clumsiness. He
denies any problems with balance. Examination shows a widebased gait, intrinsic
wasting, and a postive Hoffman’s sign bilaterally. An MRI scan of the cervical spine
is shown in Figure 16. What is the most appropriate treatment?

1. Anterior diskectomy without fusion at C4-C5


2. Epidural injections
3. Anterior diskectomy and fusion at C4-C5 and C5-C6
4. Multilevel laminectomy and fusion
5. Multilevel posterior foraminotomies

DISCUSSION: The patient has classic symptoms of


myelopathy with upper motor neuron signs on
examination. His symptoms have been present for
years, and are getting worse. The cervical spine MRI
scan shows spinal stenosis with multilevel spondylosis
causing spinal cord compression at multiple levels. With
the longstanding duration of the patient’s signs and symptoms, combined with
involvement of multiple levels in the cervical spine, posterior multilevel laminectomy and
fusion is the best treatment option. Two-level anterior diskectomy and fusion would
address the two areas of most severe narrowing,but it would fail to decompress the
other stenotic areas which also require decompression. Posterior cervical
foraminotomies would only address radicular symptoms, which are not present in this
patient, and would not succeed in decompression of the spinal cord. Cervical epidural
injections are not indicated for myelopathy symptoms, and may in fact place this patient
at risk for neurologic deterioration. PREFERRED RESPONSE: 4
12

136
Question 17Of the following signs or findings, which one is most consistent with the
diagnosis of cervical radiculopathy?
1. Spurling sign
2. Hoffman sign
3. Clonus
4. Inverted brachioradialis reflex
5. Babinski sign

DISCUSSION: The Spurling sign is elicited by extending the neck and having the patient
rotate his or her head toward the side of the symptoms; reproduction of symptoms,
including those of radicular pain,suggests cervical nerve root compression as a
contributing factor. The remaining signs and clinical findings are seen with cervical
myelopathy. The Hoffman sign is elicited by flicking the terminal phalanx of the third or
fourth finger; a positive response is seen as reflex flexion of the terminal phalanx of the
thumb. The inverted brachioradialis reflex is seen when the brachioradialis tendon is
tapped and a diminished brachioradialis reflex is noted but reflex contraction of the
finger flexors is seen. PREFERRED RESPONSE: 1

Question 18 A 42-year-old woman has a 3-week history of acute lower back pain with
radiation into the left lower extremity. There is no history of trauma and no systemic
symptoms are noted. Examination reveals a positive straight leg test at 25 degrees on
the left side. Motor testing reveals mild weakness of the gluteus maximus and
weakness of the gastrocnemius at 3/5. Sensory examination reveals decreased
sensation along the lateral aspect of the foot. Knee reflex is intact; however, the ankle
reflex is absent. MRI scans show a posterolateral disk herniation. The diagnosis at this
time is consistent with a herniated nucleus pulposus at what level?
1. L1-2
2. L2-3
3. L3-4
4. L4-5
5. L5-S1

DISCUSSION: The patient’s history and examination findings are consistent with a lumbar
disk herniation at the L5-S1 level. Weakness of the gastrocnemius and gluteus maximus
are consistent with an S1 lumbar radiculopathy. Nerve root tension signs are also

13

137
consistent with a disk herniation at L5-S1, which typically affects the traversing S1 nerve
root. PREFERRED RESPONSE: 5

Question 19Back injuries occur in approximately 2% of the work force every year,
resulting in workers’ compensation costs of more than $20 billion. What percentage of
workers, with symptoms severe enough to require work absence, return to work within
12 weeks?
1. 80% to 90%
2. 70% to 80%
3. 60% to 70%
4. 50% to 60%
5. 40% to 50%

DISCUSSION: In adults, 70% to 85% will experience an episode of low back pain at some
period during their life. Most recover quickly and without residual functional deficits. Of
those patients with symptoms severe enough to require absence from work, 60% to 70%
return within 6 weeks and 80% to 90% return within 12 weeks. After 12 weeks of
symptoms, return to work is much slower. PREFERRED RESPONSE: 1

Question 20In the treatment of fungal osteomyelitis of the spine, what is the preferred
agent?
1. Cefazolin
2. Vancomycin
3. Combination treatment with isoniazid, ethambutol, rifampin, and pyrazimide
4. Trimethoprim-sulfamethoxazole
5. Amphotericin B

DISCUSSION: Amphotericin B is a broad-spectrum antifungal that is commonly used as


the firstline agent for treatment of fungal infections of the spine. Cefazolin, vancomycin,
and trimethoprimsulfamethoxazole are better suited for the treatment of bacterial
infections. Combination treatment with
isoniazid, ethambutol, rifampin, and pyrazimide is used for the treatment of tuberculous
infections.
PREFERRED RESPONSE: 5

14

138
Question 21 Which of the following indicates resolution of a postoperative wound
infection?
1. C-reactive protein (CRP) has normalized and erythrocyte sedimentation rate (ESR) is
improving
2. CRP and ESR remain elevated
3. CRP and ESR are below normal
4. CRP has improved to the same degree as the ESR has improved
5. CRP remains elevated after the ESR has normalized
DISCUSSION: Khan and associates in a retrospective review found that CRP was more
responsive and normalized with resolution of infection, whereas the ESR can remain
elevated in the presence of a normal CRP. Since the normal range of ESR and CRP usually
begins at 0, a level that is below normal is not likely to be found. PREFERRED RESPONSE: 1

Question 22Two weeks after being treated for a nondisplaced type II odontoid fracture
with a halo fixator, a 78-yearold man reports left anterior pin site pain. On
examination, the left anterior pin fixation site is tender andfocal erythema with mild
loosening is noted. The patient denies fevers, chills, or any other constitutional
symptoms. What is the most appropriate course of action at this time?
1. Removal of the halo and soft collar placement
2. Retightening of the pin and treatment with oral antibiotics
3. Removal of the left anterior pin
4. Treatment with oral antibiotics
5. Surgical treatment of the type II odontoid fracture with posterior C1-C2 fusion

DISCUSSION: One of the most common complications of halo device fixation is pin
loosening. In adults, pin loosening can occur in up to 36% of patients and most commonly
occurs in the anterior pins. If there is no infection present, a loose pin can be managed by
simply retightening the screw one to two turns. Infection at the pin site is the second
most common complication of the halo device, occurring in about 20% of patients.
Infection can be either superficial or deep. Superficial infections can be treated with oral
antibiotics and wound care at the pin site. Pin loosening does not always occur with pin
site infection. The infection can be treated with or without removing the pin. If a deep
infection develops, the pin should be removed. If an abscess develops, parenteral
antibiotics and drainage of the abscess may be necessary. A new pin may be placed in a
new position. Other complications from halo fixation include skin breakdown and
intracranial penetration. PREFERRED RESPONSE: 2
15

139
Question 23Which of the following areas of the vertebral segment has the highest ratio
of cortical to cancellous bone?
1. Thoracic vertebral bodies
2. Lumbar vertebral bodies
3. Sacrum
4. Pedicles of the lower lumbar spine
5. Pedicles of the thoracic spine

DISCUSSION: The weight-bearing potential of bone is influenced by the ratio of cortical to


cancellous bone. The area of the spinal anatomy that has the highest ratio is the pedicles
of the thoracic spine. This is followed by the lumbar pedicles. The vertebral bodies have a
lower ratio than the pedicles, with the sacrum having the very lowest ratio.

PREF RES: 5

Question 24A 36-year-old man has a 2-day history of acute lower back pain with severe
radicular symptoms in the left lower extremity. The patient has a positive straight leg
test at 40 degrees on the left side and mild decreased sensation on the dorsum of the
left foot. What is the most appropriate management at this time?
1. Urgent admission to the hospital for surgical intervention
2. Immediate MRI of the lumbar spine as an outpatient
3. Anti-inflammatory medications and activity modification
4. Caudal epidural steroid injection
5. Electromyography

DISCUSSION: In the absence of any severe progressive neurologic deficits or other red
flags, the most appropriate management for an acute lumbar disk herniation is
nonsurgical care. Conservative treatments such as limited bed rest, anti-inflammatory
medications, and judicious use of pain medications are appropriate in this clinical
situation. Up to 90% of patients will experience a resolution of symptoms without the
need for surgical intervention within a 3-month window. In the acute setting, with no
neurologic deficits, immediate MRI of the lumbar spine is neither beneficial nor
warranted. Likewise,without signs of an acute deficit, emergent surgical intervention and
caudal epidural steroid injections are not needed.

PREFERRED RESPONSE: 3

16

140
Question 25What factor is associated with the highest risk for in-hospital complications
for patients undergoing a lumbar fusion for degenerative spondylolisthesis?
1. Hospital size
2. Gender
3. Race
4. Age
5. One comorbidity

DISCUSSION: Age and having three or more comorbidities is associated with a higher rate
of complications in patients undergoing a lumbar fusion for lumbar degenerative
spondylolisthesis. Race,gender, and hospital size have not been found to be associated
with higher complication rates. PREFERRED RESPONSE: 4

Question 26 A 57-year-old man has had a 2-week history of neck pain. He has no history
of radiating symptoms, and has no complaints of numbness or paresthesias. There was
no trauma associated with the onset of the pain. Figure 26 shows the MRI scan initially
obtained by his family physician. What should the patient be told regarding the
prevalence of the MRI findings in his age group?

1. Less than 10%


2. 20% to 25%
3. 50% to 60%
4. 75% to 80%
5. Greater than 95%

DISCUSSION: The MRI findings reveal age-related degenerative changes in the cervical
spine, which is a very common finding in the adult population. Boden and associates
evaluated cervical spine MRI findings on 63 asymptomatic subjects, and found that the
prevalence of having at least one degenerative disk was approximately 57% in those
older than age 40 years.

PREFERRED RESPONSE: 3

17

141
Question 27Which of the following injury patterns is most likely to result from a
hyperextension mechanism to the cervical spine?
1. Unilateral facet dislocation
2. Bilateral facet dislocation
3. Cervical burst fracture
4. Quadrangular fracture
5. Central cord syndrome

DISCUSSION: Central cord syndrome typically results from a hyperextension injury


mechanism in an elderly patient with underlying cervical spondylosis and stenosis.
Unilateral and bilateral facet dislocations occur in association with flexion-distraction
injuries. Cervical burst fractures are seen with pure axial loading of the spine.
Quadrangular fracture patterns are observed with flexion-compression loads. PR RES: 5

Question 28 Figures 28a and 28b show the posteroanterior and lateral radiographs of a
38-year-old woman with adult idiopathic scoliosis. She reports symptoms of long-
standing lower back pain, progressive loss of height,and the inability to stand upright at
the end of the day. What radiographic finding has been found to most closely correlate
with symptoms of lower back pain?

1. Thoracic scoliosis
2. Thoracic hypokyphosis
3. Lumbar disk degeneration
4. Thoracolumbar kyphosis
5. Lumbar hyperlordosis

DISCUSSION: Adult idiopathic


scoliosis and adult “de-novo”
scoliosis can present with a number
of symptoms that relate to
associated degenerative findings
such as stenosis or spondylolisthesis. In the absence of these associated conditions,
increased levels of pain in patients with scoliosis has been found to most closely correlate
with sagittal imbalance. Thoracolumbar and lumbar curves and thoracolumbar kyphosis
have both been found to closely correlate with increased symptoms and lower health-
related quality of life (HRQL) outcome scores. Thoracic scoliosis, thoracic hypokyphosis,
18

142
lumbar hyperlordosis,and lumbar disk degeneration have not been found to correlate
with increased symptoms. PREFERRED RESPONSE: 4

Question 29 A 75-year-old woman is undergoing a T10-S1 lumbar decompression and


fusion for severe degenerative scoliosis. During the deformity corrective maneuver,
intraoperative neuromonitoring revealed a sustained 80% decrease in somatosensory-
evoked potential (SSEP) amplitudes. Appropriate lead placement and functioning has
been reconfirmed by the neuromonitoring technician. The anesthesiologist has ruled
out any anesthetic-related or hemodynamic issues. What is the next appropriate step in
management?
1. Completion of the surgical procedure with continued monitoring
2. Reversal of the corrective maneuver and consideration of a wake-up test
3. Administration of high dose corticosteroids intraoperatively
4. Removal of all instrumentation
5. Discontinue monitoring
DISCUSSION: The most appropriate management is discontinuation of the spinal
instrumentation procedure, including releasing any distractive forces. Given the ongoing
changes, proceeding with the procedure and/or resetting the baseline amplitudes is
inadvisable. If the SSEPS amplitudes fail to return in a timely fashion, it is strongly
recommended to consider a wake-up test. The purpose of intraoperative
neuromonitoring is to provide a real-time assessment of the functional integrity of the
central and peripheral nervous systems during surgery to prevent iatrogenic injury.
Sustained decreased SSEP amplitudes of greater than 50% and transcranial electric
Motor-Evoked Potentials (tceMEP) amplitudes of greater than 75% are indicative of a
possible significant intraoperative neurologic complication. Although intraoperative
corticosteroids might be helpful, especially in the event of a continued SSEP change, the
most important next step would be to release the distractive forces and reevaluate the
patient’s neurologic status. PREFERRED RESPONSE: 2
Question 30 Which of the following is most commonly associated with the use of
bisphosphonates in the setting of
metastatic breast cancer?
1. Reduction of skeletal-related events by 30% to 40%
2. Jaw osteonecrosis in 15% of patients
3. Pain improvement in 20% of patients
4. Acceleration of hypercalcemic symptoms and signs
5. Accelerated bone destruction
19

143
DISCUSSION: Bisphosphonate therapy in breast cancer patients is indicated for the
correction of hypercalcemia and the prevention of cancer treatment-induced bone loss.
Bisphosphonates reduce metastatic bone pain in at least 50% of patients and can reduce
the frequency of skeletal-related events by 30% to 40%. Osteonecrosis of the jaw could
occur in up to 2.5% of breast cancer patients during longterm bisphosphonate therapy.
PREFERRED RESPONSE: 1

Question 31 Figures 31a through 31d. His symptoms are not improved with forward
flexion of the lumbar spine. His lower extremity pain is relieved when he sits or ceases
activity. Which of the following tests would be most helpful in establishing a diagnosis?

1. Selective nerve root blocks


2. Electromyography and a nerve conduction velocity study of the lower extremities
3. Ankle-brachial index (ABI)
4. Bilateral hip radiographs
5. Post-myelography CT scan
DISCUSSION: The differential diagnosis of degenerative lumbar stenosis is extensive.
Vascular and neurogenic claudication frequently coexist in the older population.
Therefore, it is important to determine the specific etiology of a patient’s lower extremity
claudication prior to any surgical intervention.Vascular claudication is relieved with
cessation of activity, whereas neurogenic claudication requires that the patient sit down
or flex the lumbar spine forward to increase the canal diameter. Because this patient
does not experience improvement in his symptoms with sitting or forward flexion, it is
likely that he is experiencing vascular claudication. The ankle-brachial index (ABI) is the
ratio of the blood pressure in the lower legs to the blood pressure in the arms. Compared
with the arm, lower blood pressure in the leg is a sign of peripheral vascular disease. The
ABI is calculated by dividing the systolic blood pressure in the arteries at the ankle and
foot by the higher of the two systolic blood pressures in the arms. An ABI value between
20

144
0.40 to 0.80 is moderately decreased and such patients often experience symptoms such
as intermittent claudication. Selective nerve root blocks prove to be more useful in
identifying specific level(s) of involvement in patients experiencing radicular pain and
paresthesias. Their utility is less helpful in lower extremity claudication.
Electrophysiologic studies are rarely useful, except in identifying the presence and source
of a peripheral neuropathy. About 80% of patients with symptomatic lumbar stenosis will
demonstrate electromyographic changes. Osteoarthritis of the hip may be associated
with buttock, groin, hip, and thigh pain. Decreased range of motion and hip joint pain,
especially in internal rotation and abduction, are common findings in patients with
degenerative arthritis of the hip. While post-myelography CT has been found superior to
MRI as a single study for the preoperative planning of decompression for lumbar spinal
stenosis, it will not assist in differentiating vascular from neurogenic claudication. P R: 3

Question 32For a patient with a type II odontoid fracture, which of the following factors
best predicts the development of a nonunion with nonsurgical management?
1. Frontal oblique pattern
2. Magnitude of fracture displacement
3. Degree of posterior angulation
4. Age
5. Patient history of diabetes mellitus

DISCUSSION: All five factors have been found to be associated with nonunion for type II
odontoid fractures. Of these, initial fracture displacement of greater than 6 mm has the
greatest association with the development of fracture nonunion. PREFERRED RESP: 2

Question 33 Figures 33a and 33b show the standing posteroanterior and lateral
radiographs of a 59-year-old woman with adult idiopathic scoliosis. She underwent a
prior decompressive laminectomy and fusion at L4-S1 to address lumbar stenosis. She
now reports progressive lower back pain and a feeling of being shifted to the right. If
surgical intervention is considered, what is the most important goal in improving her
healthrelated quality of life (HRQL) outcomes?
1. Correction of the thoracolumbar curve
2. Sagittal balance
3. Coronal balance
4. Correction of the thoracic curve
5. Shoulder balance
21

145
DISCUSSION: Sagittal balance is the most reliable predictor of clinical symptoms and
HRQL outcomes on the SRS 29, SF-12, and Oswestry Disability Index. Coronal balance,
shoulder balance,curve magnitude, and degree of curve correction are less critical in
determining clinical symptoms and outcomes. PREFERRED RESPONSE: 2

Question 34 Pedicle subtraction osteotomies (PSO) are commonly performed in the


lumbar spine to treat sagittal imbalance. What is the most common complication
following a PSO in the lumbar spine?
1. Pseudarthrosis
2. Nerve root injury
3. Spinal cord injury
4. Aortic injury
5. Dural tear

DISCUSSION: The rate of pseudarthrosis at 5-year follow-up is 29%, with most occurring
at the thoracolumbar junction cephalad to the site of the PSO. The rate of postoperative
neurologic deficits is 11%, with 2.8% resulting in permanent deficits. Spinal cord injury is
rare because the PSO is typically performed in the lumbar spine below the conus.
PREFERRED RESPONSE: 1

Question 35 What is the most commonly involved level for brachial plexus stretch
injuries or “stingers” in younger athletes involved in collision sports?
1. C3-4
2. C4-5
3. C5-6
4. C6-7
5. C7-T1

DISCUSSION: The most commonly involved level for brachial plexus traction injuries in
young athletes is C5-6, ostensibly due to the greater mobility in the midcervical spine.
“Stingers” in older athletes may be due to foraminal stenosis in combination with
extension and nerve root compression. PREFERRED RESPONSE: 3

22

146
Question 36 What is the most common non-anesthetic-related reversible cause of
sustained changes in intraoperative neurophysiologic monitoring signals during spinal
surgery?
1. Pedicle screw misplacement
2. Patient positioning
3. Spinal cord ischemia
4. Retractor placement
5. Hypertension
DISCUSSION: Patient positioning that results in local nerve compression, plexus traction,
or improper neck alignment is the most common non-anesthetic-related cause of
changes in intraoperative neurophysiologic monitoring data during spinal surgery. Pedicle
screw malpositioning, spinal cord ischemia, and retractor placement are all less common
causes. Hypotension, not hypertension, can be a cause of intraoperative neurophysiologic
changes. PREFERRED RESPONSE: 2

Question 37 Figure 37 shows the standing lateral radiograph of a 62-year-old woman


who reports lower back pain and the inability to stand upright. What permanent
anatomic pelvic parameter should be measured and considered when determining the
amount of lumbar lordosis correction that will be necessary to obtain sagittal balance?

1. Pelvic tilt
2. Pelvic incidence
3. Sacral slope
4. Acetabular version
5. Femoral version

DISCUSSION: Pelvic incidence (PI) is the anatomic angle


between the sacral end plate and a line connecting the center
of the femoral heads. Increased pelvic incidence has been
found to correlate with the incidence and severity of
spondylolisthesis. Patients with increased PI require increased
lumbar lordosis to restore sagittal balance. Pelvic tilt (PT) and
sacral slope (SS) have also been found to correlate with lumbar
lordosis; however, both PT and SS can change depending on pelvic rotation. PI is the only
permanent pelvic parameter that is unaffected by pelvic rotation. Acetabular and femoral
version have not been found to be associated with lumbar lordosis. PREFERRED RES: 2
23

147
Question 38 A 29-year-old woman is seen in the emergency department with a 24-hour
history of severe back and leg pain precipitated by weight-lifting. The patient reports
bilateral leg pain and is unable to urinate. She has dense anesthesia in the perineal
region on examination. A MRI scan is shown in Figure 38. The patient is taken to
surgery urgently. What is her prognosis for recovery?

1. Improvement in her pain and sensory symptoms following surgery but may have
residual bladder dysfunction
2. Decreased pain following surgery; sensory deficits and bladder function are not likely
to improve
3. No change in her symptoms following surgery
4. Complete resolution of pain and will have normal sensation and bladder function
following surgery
5. Improvement in her pain and complete return of bladder function following surgery;
sensation may not return
DISCUSSION: The patient with cauda equina syndrome should be taken to surgery
urgently to provide the best chance of symptom resolution. However, many studies
indicate that patients with cauda equine syndrome do not return to a completely normal
status even following urgent surgery. Whereas pain is typically relieved after surgery,
other deficits, especially bladder and sexual dysfunction, may persist.Particularly in light
of the patient’s severe saddle anesthesia, she may have a poor prognosis for recovery of
normal bladder function. PREFERRED RESPONSE: 1
Question 39 What complication is associated with the use of epidural morphine and
steroid paste after laminectomy?
1. Surgical site infection
2. Arachnoiditis
3. Urinary retention
4. Disk space infections
5. Nerve irritation
24

148
DISCUSSION: Kramer and associates conducted a retrospective review during an
“epidemic” period to identify the risk factors associated with a sudden increase in the
rate of surgical site infections. They found in a multivariate analysis that the use of
morphine nerve paste resulted in a 7.6-fold increase in postoperative surgical wound
débridement, and an 11% rate of surgical site complications. There is no evidence in the
literature verifying the incidence of postoperative urinary retention and arachnoiditis.
PREFERRED RESPONSE: 1

Question 40 Giant cell tumors of the spine can metastasize to other areas of the body.
They most likely metastasize to which of the following areas?
1. Brain
2. Lung
3. Liver
4. Kidney
5. Colon

DISCUSSION: Giant cell tumors of the spine can metastasize to other areas of the body.
Donthineni and associates identified seven cases (three females and four males) of lung
metastases from a total of 51 cases of giant cell tumors of the spine (13.7%). Four of the
seven patients had a spine recurrence after previous treatments and the rest developed
recurrences later. The treatments for the lung nodules consisted of metastectomy in two
and chemotherapy in six patients. At the latest follow-up (ranging from 18 to 126
months), two had died of the disease, two had no evidence of the disease, and three
were alive with disease. This series shows a higher metastatic rate from spine giant cell
tumor as compared with those from the extremities, but the overall behavior and
treatment outcomes of the lung metastases are similar. Tunn and associates also showed
a predilection for pulmonary metastatic lesions. PREFERRED RESPONSE: 2

Question 41 A positive straight leg raising sign is when pain radiates distal to the knee
when the affected leg is passively elevated. The increased tension generally occurs
between
1. 0 degrees to 30 degrees.
2. 30 degrees to 70 degrees.
3. 45 degrees to 90 degrees.
4. 60 degrees to 90 degrees.
5. 90 degrees.
25

149
DISCUSSION: A positive straight leg raising sign causes pain radiating distal to the knee
when the nerve is under tension. This increased tension generally occurs between 30
degrees and 70 degrees of passive elevation. PREFERRED RESPONSE: 2

Question 42 Figures 42a through 42c show the MRI scans of a 56-year-old woman with
progressively worsening low back and bilateral lower extremity pain. Based on these
images, what muscle or muscle group would be expected to be weak on physical
examination?

1. Quadriceps
2. Hamstrings
3. Hip adductors
4. Extensor hallucis longus
5. Gastrocnemius

DISCUSSION: Whereas subjective complaints of leg pain are common among patients
seeking surgical treatment for spondylolisthesis, documented neurologic deficit or
radiculopathy is seen less frequently.Subjective decreases to light touch over the dorsum
of the foot and mild weakness of the extensor halluces longus are the most common
neurologic abnormalities, correlating with L5 nerve root irritation as seen with L5-S1
spondylolisthesis. Many patients with spondylolisthesis report hamstring tightness;
however,these structures are not usually weak. Quadriceps and tibialis anterior weakness
is seen with L4 nerve root irritation. The gastrocnemius is generally weak in S1 nerve root
syndromes. PREFERRED RESPONSE: 4

Question 43 Which of the following is the best method of treatment for a dominant
vertebral artery injury during an occipitocervical fusion?
1. Packing the area with Surgicel
2. Stenting
3. Proximal and distal ligation
4. Direct repair
5. Screw placement to control bleeding

26

150
DISCUSSION: In dominant vertebral artery injury (usually left sided), direct repair is the
best method of treatment despite the technical demands of the exposure and
microvascular repair. Direct repair can minimize immediate and delayed risks of injury
and ischemia. Aggressive packing can result in delayed hemorrhage or fistula formation.
Ligation-associated morbidities such as cerebellar infarction, cranial nerve palsies,
hemiplegia, and even a mortality rate of 12% have been reported. Screw placement is
usually sufficient to control bleeding, but should be considered as a last resort in cases of
dominant vertebral artery injury.

PREFERRED RESPONSE: 4

Question 44 On an axial CT image, which of the following dimensions is considered to


be indicative of a critical amount of lumbar spinal stenosis?

1. AP canal diameter of less than 20 mm


2. AP canal diameter of less than 50 mm
3. Cross-sectional area of less than 100 mm2
4. Cross-sectional area of less than 50 mm2
5. Circumferential area of less than 25 mm

DISCUSSION: Lumbar central stenosis is defined by an AP canal diameter of less than 10


mm or a crosssectional area of less than 100 mm2 as measured on CT.

PREFERRED RES: 3

Question 45 Which of the following materials has the highest modulus of elasticity?
1. Cortical bone
2. Cobalt-chrome
3. Ceramic
4. Titanium
5. Stainless steel

DISCUSSION: Young’s modulus of elasticity is a measure of the stiffness of a material and


its ability to resist deformation. This is the slope of the stress/strain curve in the elastic
range. The highest modulus is ceramic, followed by: cobalt-chrome alloy, stainless steel,
titanium, and then cortical bone. PREFERRED RESPONSE: 3

27

151
Question 46 Morphogenic factors such as bone morphogenetic protein-2, bone
morphogenetic protein-7, and transforming growth factor-beta have been postulated
to play a role in reversing the process of intervertebral disk degeneration. These
molecules work via what mechanism?
1. Increase cell proliferation leading to increased cell numbers within the disk and
enhanced matrix synthesis
2. Inhibit the catabolic effects of matrix metalloproteinases
3. Increase the chondrogenic phenotype of disk cells and increase matrix synthesis
4. Promote increased end plate vascularity and nutritional diffusion within the disk
5. Block receptors for tumor necrosis factor-alpha and interleukin-1

DISCUSSION: Bone morphogenetic protein-2, bone morphogenetic protein-7, and


transforming growth factor-beta are morphogens that have been shown to alter the
phenotype of target cells without increasing cellular proliferation. Within the
intervertebral disk, these factors have the potential to increase the chondrogenic
phenotype among disk cells, and this results in the increased production of disk
matrix.Mitogenic molecules, such as insulin-like growth factor-1 and fibroblast growth
factor, function to increase cellular proliferation. PREFERRED RESPONSE: 3

Question 47 A 75-year-old woman reports persistent severe low back pain after lifting a
bag of groceries 8 weeks ago.She denies the presence of any lower extremity pain,
weakness, or other symptoms. AP and lateral lumbar spine radiographs are shown in
Figures 47a and 47b. For further evaluation, what imaging study should
be obtained next?

1. Lumbar spine flexion and extension


radiographs
2. Lumbar spine CT
3. Lumbar spine MRI
4. Whole body bone scan
5. Tagged white blood cell scan

DISCUSSION: The radiographs show a


L1 vertebral compression fracture. MRI
scans of the lumbar spine are indicated
to help determine the age of the
28

152
fracture via evaluation of the fat-suppressed T2-weighted images. If those images show
edema (bright signal) within the fracture, it can be assumed to be an acute or subacute
fracture. Also, MRI scans may help further characterize the fracture as a benign
osteoporosis-related fracture versus a pathologic fracture. Lastly, MRI scans allow for
evaluation of the absence, presence, and degree of spinal stenosis. Although a lumbar
spine CT scan is also a good option as a next step, given that the patient has a known
fracture as seen on the radiographs, MRI scans will likely provide more relevant
information in this case. A CT scan may assist in the evaluation of the posterior vertebral
body wall and posterior elements in patients with burst fractures. A three-phase bone
scan is a good alternative for the evaluation of the acuity of vertebral compression
fractures in patients who have a contraindication to MRI scans. Flexion-extension
radiographs are most commonly used to evaluate for instability and will not help
determine the age of this fracture. Tagged white blood cell scans are performed to
evaluate for infection, which is highly unlikely in this patient. PRE RES: 3

Question 48 Posterior lumbar spine arthrodesis may be associated with adjacent


segment degeneration cephalad or caudad to the fusion segment. Which of the
following is the predicted rate of symptomatic degeneration at an adjacent segment
warranting either decompression and/or arthrodesis at 5 to 10 years after lumbar
fusion?
1. 5.5% at 5 years and 75.5% at 10 years
2. 10.1% at 5 years and 65.2% at 10 years
3. 16.5% at 5 years and 36.1% at 10 years
4. 26.5% at 5 years and 56.1% at 10 years
5. 49.2% at 5 years and 15.1% at 10 years

DISCUSSION: Ghiselli and associates described a rate of symptomatic degeneration at an


adjacent segment warranting either decompression or arthrodesis to be 16.5% at 5 years
and 36.1% at 10 years based on a Kaplan-Meier analysis. PREFERRED RESPONSE: 3
Question 49 Halo treatment for preadolescent children typically requires the use of
which of the following?
1. 4 to 6 pins with an insertional torque of 1 to 5 in-lb
2. 4 to 6 pins with an insertional torque of 4 to 6 in-lb
3. 4 to 6 pins with an insertional torque of 6 to 8 in-lb
4. 8 to 12 pins with an insertional torque of 1 to 5 in-lb
5. 8 to 12 pins with an insertional torque of 4 to 6 in-lb
29

153
DISCUSSION: The complication rate with halo vest treatment in children is reported to be
as high as 68% in contrast to a 36% complication rate in adults. These complications
include not only pin tract infections,but also skull penetration. Multiple pins allow for the
early removal of pins without fixation consequences should pin site infections begin to
develop. Moreover, there is significant variability in the insertional torque applied by a
variety halo pin torque wrenches, including those from the same manufacturer.
Consequently, the use of a large number of pins (8 to 12) placed a very low insertional
torque (1 to 5 in-lb) in children is recommended. A CT scan of the head should also be
considered to assess for the thickest areas of the skull suitable for pin application. PR: 4

Question 50 Which of the following factors is most likely to contribute to


pseudarthrosis in a patient who has undergone a single-level anterior decompression
and fusion procedure for the treatment of cervical radiculopathy?
1. Sagittal alignment
2. History of diabetes mellitus and tobacco use
3. Performance of an uninstrumented fusion (ie, no plate and screws)
4. Use of allograft (instead of autograft)
5. Fusion at the C3-C4 level

DISCUSSION: Various factors affect the pseudarthrosis rate in patients who undergo
anterior cervical decompression and fusion. Patient factors, including history of smoking
and history of diabetes mellitus,have been shown to significantly increase pseudarthrosis
rates. The literature has been mixed with regard to fusion rates for allograft versus
autograft, especially for one-level fusions; in that category,there is minimal, if any,
difference. Similarly, several authors have shown higher rates of fusion with
uninstrumented single-level rather than instrumented anterior cervical decompressions
and fusions. The level (ie, cranial or caudal) of fusion and sagittal alignment have not
been correlated with fusion rates. PREFERRED RESPONSE: 2

Question 51 A 60-year-old woman has an 8-month history of left-sided arm and neck
pain. Her symptoms developed insidiously. The patient reports burning dysesthesias in
the left forearm and middle finger. Examination reveals weakness of the left triceps
and wrist flexors. An axial MR scan at the level of the C6-7 disk is shown in Figure 51.
The patient wishes to avoid surgery if possible and asks about epidural steroid
injections. What is the likelihood of long-term relief of her symptoms following a
steroid injection?
30

154
1. 5% to 10%
2. 10% to 20%
3. 30% to 40%
4. 50% to 70%
5. 80% to 90%

DISCUSSION: The patient has a C7 radiculopathy in


the left upper extremity. Several studies indicate
that relief of radicular symptoms for periods of
longer than 1 year can be achieved in 50% to 70% of
patients following administration of epidural steroids. PREFERRED RESPONSE: 4

Question 52 A 60-year-old woman has progressive neck pain, upper extremity pain, and
paresthesias. A lateral cervical spine radiograph and an MRI scan are shown in Figures
52a and 52b. What is the most likely underlying diagnosis?

1. Osteomyelitis
2. Ankylosing spondylitis
3. Age-related degenerative changes
4. Rheumatoid arthritis
5. Previous cervical decompression

31

155
DISCUSSION: The radiograph and sagittal T2-weighted MRI scan show multilevel
degenerative changes and subaxial subluxations with anterolisthesis at C3-C4 and C4-C5
and retrolisthesis at C5-C6. In addition, there is evidence of midcervical kyphosis. Such
findings are often seen in patients with rheumatoid arthritis. Patients with osteomyelitis
typically show increased signal intensity in the disks and vertebral bodies. Patients with
ankylosing spondylitis typically show ankylosis of the disks and vertebral
bodies. Age-related degenerative changes typically manifest as degenerative disk disease
with occasional single-level spondylolisthesis, but not typically multilevel
spondylolisthesis, as seen in this patient. The spinous processes are intact; these changes
do not appear to be postoperative. PREFERRED RESPONSE: 4

Question 53 A 66-year-old woman reports chronic mild low back pain. Over the last 3
years, she has noticed worsening buttock and posterior leg pain with standing and
walking. Sitting seems to improve the pain.She also reports numbness in both legs with
walking. An MRI scan and standing radiographs of the lumbar spine are shown in
Figures 53a through 53c. She has undergone two epidural injections with good, but
short-term relief. Further treatment to alleviate this patient’s symptoms should consist
of which of the following?

1. Manual therapy
2. Multilevel laminectomy
3. Facet injections
4. L4-5 laminotomy
5. Laminectomy and fusion
32

156
DISCUSSION: The patient reports symptoms that are classic for neurogenic claudication
secondary to lumbar spinal stenosis. Nonsurgical management has failed to provide
relief, thus a surgical approach is a reasonable treatment option at this point. Studies
have shown significant benefit in patients with lumbar stenosis who choose to undergo
surgical treatment. Manual or manipulative therapy is unlikely to provide relief. Facet
injections are not effective for neurologic symptoms. An L4-5 laminotomy will not
adequately address the patient’s pathology. In the absence of instability on imaging
studies, arthrodesis is not indicated.

PREFERRED RESPONSE: 2

Question 54 When evaluating a patient with a lumbar burst fracture, the integrity of
the posterior ligamentous complex must be evaluated. Which of the following is a
complete and accurate list of the components of the posterior ligamentous complex?

1. Supraspinous ligament, interspinous ligament, ligamentum flavum, and the facet


joint capsules
2. Supraspinous ligament, interspinous ligament, ligamentum flavum, facet joint
capsules, anddisks
3. Supraspinous ligament, interspinous ligament, and ligamentum flavum
4. Supraspinous ligament, interspinous ligament, ligamentum flavum, and the facet
joints
5. Supraspinous ligament, interspinous ligament, ligamentum flavum, facet joint
capsules, facet joints, and the posterior longitudinal ligament

DISCUSSION: The components of the posterior ligamentous complex are the


supraspinous ligament,interspinous ligament, ligamentum flavum, and the facet joint
capsules. Recent studies have emphasized the importance of the posterior ligamentous
complex. The Thoracolumbar Injury Classification and Severity Score (TLICS) is a
classification system that allows for efficient and effective classification of thoracolumbar
spine injuries. It evaluates three characteristics of the injury: 1) morphology, 2) integrity
of the posterior ligamentous complex, and 3) neurologic status of the patient. A
composite injury severity score is calculated from an evaluation of these characteristics of
the injury, and the score can be used to guide treatment.

PREFERRED RESPONSE: 1

33

157
Question 55 In the treatment of chondrosarcomas of the spine and sacrum, which of
the following is a significant factor associated with a worse prognosis with respect to
local control and/or survival?
1. Lower histologic tumor grade of the lesion
2. Increasing patient age
3. Negative excisional margins
4. Initial biopsy done at the tumor center
5. Spine location as opposed to pelvic location of the tumor

DISCUSSION: Bergh and associates examined 69 patients with chondrosarcoma and


looked at prognostic factors affecting local recurrence and survival. There were 61
conventional chondrosarcomas, grades1-3 (with 13 arising in a preexisting
osteochondroma) and eight grade 4 chondrosarcomas (seven dedifferentiated and one
mesenchymal). The overall local recurrence rate was 27%, and the estimated overall 5-
and 10-year survival rates were 72% and 67%, respectively. In contrast, the observed
local recurrence rate was 3% (1 patient) in 31 patients whose conventional
chondrosarcomas were resected with adequate surgical margins; 90% of these patients
survived and most of them (26 of 31 or 84%)were continuously disease free. Significant
factors associated with a worse prognosis with respect to local control and/or survival
were high histologic tumor grade, increasing patient age, primary surgery outside of a
tumor center, incisional biopsy versus a noninvasive diagnostic procedure, and
inadequate surgical margins. Lee and associates also looked at factors affecting outcome
and found that predictors of metastasis and death included local recurrence, a pelvic
location of the tumor, a tumor that was more than 100 cubic centimeters in size, a ploidic
abnormality (aneuploidy coupled with a high mean DNA index), a histologic grade of 3,
and a dedifferentiated type of tumor (p < 0.001). PREFERRED RESPONSE: 2

Question 56 During the application of halo skeletal fixation, the most appropriate
position for the placement of the anterior halo pins is approximately 1 cm above the
superior orbital rim and
1. lateral placement, directly within the temporalis muscle.
2. within the lateral third of the superior orbital rim.
3. lateral to the superior orbital rim.
4. medial third of the superior orbital rim.
5. lateral between the temporalis muscle and zygomatic temporal nerve.

34

158
DISCUSSION: Halo fixation is the most rigid form of cervical orthosis but complications
can arise from improper placement of the fixation pins. A relatively safe zone for anterior
pin placement is located 1 cm above and within the lateral third of the superior orbital
rim. This position avoids the supraorbital and supratrochlear nerves over the medial one
third of the orbit. The more lateral positions in the temporal fossa have very thin bone
and can interfere with the muscles of mastication. PREFERRED RESPONSE: 2

Question 57 A 28-year-old man has had a 2-week history of right posterior leg pain,
with numbness and tingling in the same distribution. He denies any problems with
bowel or bladder function. Examination shows intact motor strength in his bilateral
lower extremities, with numbness to light touch in the lateral border of his right foot.
Over the past 2 weeks, his leg pain has improved significantly. MRI scans are shown in
Figures 57a and 57b. What is the most appropriate course of management?

1. Referral to a pain clinic for epidural injections


2. Activity modification and anti-inflammatory medications
3. Urgent hospital admission followed by hemilaminotomy and diskectomy
4. Hemilaminotomy and diskectomy scheduled as an elective procedure
5. Posterior laminectomy and fusion

DISCUSSION: The patient has an L5-S1 disk herniation, which has a favorable prognosis
without surgical intervention. Most acute lumbar disk herniations resolve with
nonsurgical management. The most appropriate course of initial treatment should be
analgesics and activity modification, followed by rehabilitation as the symptoms allow.
Although the MRI scan indicates a large disk herniation, he has no symptoms or signs that
35

159
would warrant urgent surgical decompression. Planned elective discectomy should be
considered only if nonsurgical management fails to provide relief. Epidural injections
could be considered if the initial course of treatment fails to give the patient significant
relief. Posterior laminectomy and fusion is not indicated without the presence of
instability. PREFERRED RESPONSE: 2
Question 58 An 80-year-old man who was involved in a fall from ground height is
evaluated in the emergency department for head lacerations and mild neck pain.
Examination reveals only mild tenderness of the posterior neck region with some
limitation of motion. Neurologic examination is normal. Radiographs of the cervical
spine are shown in Figures 58a and 58b. What is the next most appropriate step in
management for this patient?

1. Admission for 24-hour observation for possible intracranial hematoma


2. Discharge with a soft collar and follow-up in 24 hours
3. CT of the cervical spine
4. Repeat radiographs of the cervical spine, including flexion and extension views
5. Hard cervical collar for 6 weeks

DISCUSSION: The patient has radiographic findings compatible with diffuse idiopathic
skeletal hyperostosis (DISH) of the cervical spine. Characteristics of DISH include flowing,
non-marginal osteophytes at four or more levels. Patients with DISH develop a significant
loss of flexibility of the spine. The spine acts more as a long bone with minimal force
needed to create unstable fractures. Any minor trauma in patients with DISH should be

36

160
worked up aggressively to rule out occult fracture. In this patient, radiographs fail to
clearly rule out a fracture; therefore, CT of the cervical spine is indicated.Without a
suspicion of history of a head injury, admission specifically for a possible intracranial
hematoma is not warranted. The more concerning injury in a patient with DISH is occult
neck fracture. Treatment with a soft or hard collar is not advised until a fracture is ruled
out. Repeat radiographs are unlikely to show any occult fractures, and flexion and
extension views would not be advised in a patient with a suspected vertebral fracture.
PREFERRED RESPONSE: 3

Question 59 A 40-year-old woman is admitted to the hospital with a history of new-


onset right lower extremity weakness resulting in frequent falls. She reports that a few
weeks prior to the onset of the lower extremity symptoms, she experienced an episode
of acute back pain, which has since resolved. Examination reveals 1-2/5 strength in the
right hip flexors, abductors, and quadriceps. There is 0-1/5 strength noted in the right
ankle plantar and dorsiflexors. Deep tendon reflexes are hyperactive in the lower
extremities and there is clonus on dorsiflexion of the right ankle. The patient has a
positive Babinski sign on the right side. The patient denies neck or arm symptoms, and
the upper extremity neurologic examination is within normal limits. Figures 59a and
59b show the MRI scans of the patient’s cervical and lumbar spine. What is the
next most appropriate step in management?

1. Multilevel anterior cervical diskectomy and fusion


2. Cervical laminectomy
3. Lumbar laminectomy
4. Lumbar laminectomy and fusion
5. Thoracic MRI
37

161
DISCUSSION: Symptoms of thoracic disk herniation may present in one of three distinct
patterns:axial pain, radicular pain, or myelopathy. This patient demonstrates obvious
upper motor neuron signs,including lower extremity hyperflexia, clonus, and a positive
Babinski sign; therefore, the next most appropriate step is a MRI scan of the thoracic
spine. Thoracic disk herniations presenting with axial pain usually involve the mid to
lower thoracic region. The pain is usually worsened with activity and improved with rest.
Radicular pain syndromes are usually described as starting in the back and radiating
anteriorly in a band-like pattern to the anterior chest wall. The pain and paresthesias
generally follow a dermatomal distribution. The MRI scan of this patient’s cervical spine
demonstrates spondylotic changes;however, she denied neck or upper extremity
symptoms and more importantly, has a normal neurologic examination. There are
degenerative changes involving the lumbar spine, but these cannot explain the upper
motor neuron findings. PREFERRED RESPONSE: 5

Question 60Which of the angles depicted in Figure 60 has been found to most closely
correlate with a patient’s lumbar lordosis, thoracic kyphosis, and overall sagittal
alignment?
1. A
2. B
3. C
4. D
5. A and D

DISCUSSION: Angle A represents pelvic incidence (PI), a


constant anatomic relationship between the pelvis and
sacrum. Angle B represents pelvic tilt, and angle C represents
sacral slope. Pelvic tilt and sacral slope can change depending
on the rotation of the pelvis. Pelvic incidence has been found
directly correlate with the magnitude of lumbar lordosis and
thoracic kyphosis because it determines the angle at the base
of the spine (the lumbosacral junction). To obtain sagittal balance, the remainder of the
spine compensates, resulting in the degree of lumbar lordosis and thoracic kyphosis to
maintain an upright posture. Thus, PI must be considered in the evaluation of sagittal
balance and potential reconstructive procedures. Angle D represents the T1 angle.
PREFERRED RESPONSE: 1

38

162
Question 61 An 83-year-old man is seen in the emergency department for evaluation of
neck pain after a motor vehicle accident. The patient has no neurologic deficits. The
patient has a history significant for late stage ankylosing spondylitis with
cervicothoracic kyphosis. What is the most appropriate method of immobilization for
the cervical spine while diagnostic testing is performed?
1. Soft collar
2. Hard cervical collar with head immobilization to a backboard
3. Halo fixation with the neck in extension
4. Maintenance of flexed positioning of the spine
5. No immobilization necessary

DISCUSSION: In patients with trauma in the setting of ankylosing spondylitis and


cervicothoracic kyphosis, it is highly recommended that the neck be maintained in the
pre-morbid flexed position until definitive management can be performed. Ankylosing
spondylitis is a chronic inflammatory disease that is characterized by ossification of the
spinal column with an associated progressive kyphotic deformity of the spine. The
deformity therefore becomes the native position for the patient with ankylosing
spondylitis.Extension of the injured spine in a patient with ankylosing spondylitis can lead
to neurologic injury and/or displacement of a previously aligned fracture. PREF RES: 4

Question 62 A 38-year-old man reports right upper extremity pain that radiates from
his neck to his anterior arm,dorsoradial forearm, and into the index finger. Examination
reveals weakness of the biceps muscle group and loss of his brachioradialis reflex on
that side. At which level is he most likely to have a right-sided cervical disk protrusion
on an MRI scan?
1. C4-C5
2. C5-C6
3. C6-C7
4. C6 vertebral body
5. Far lateral C6-C7
DISCUSSION: The patient has a typical right C6 radiculopathy based on his history and
physical examination. A posterolateral disk protrusion at the C5-C6 level is mostly likely to
cause a C6 radiculopathy because the C6 nerve roots exit just above the C6 pedicle and
therefore would be compressed by a right-sided C5-C6 disk protrusion. In contrast to the
lumbar spine, far lateral disk protrusions are not typically described in the cervical spine.
PREFERRED RESPONSE: 2
39

163
Question 63 Figures 63a and 63b show the radiographs of a 38-year-old man who
reports low back and bilateral lower extremity pain. The spondylolisthesis is best
classified as which of the following?

1. Pathologic
2. Isthmic
3. Acquired
4. Degenerative
5. Dysplastic

DISCUSSION: Spondylolisthesis can be classified into five types. Type I, dysplastic, occurs
at the lumbosacral junction as a result of congenital abnormalities of the upper sacrum
and/or the arch of L5.Type II, isthmic, refers to those involving a lesion in the pars
interarticularis. Type IIA, lytic, represents fatigue fractures of the pars. Type IIB describes
those with elongated, but intact pars. Type IIC describes those that are a result of an
acute fracture of the pars. Type III, degenerative spondylolisthesis, results from long-
standing intersegmental disease. Type IV, traumatic, refers to those resulting from
fractures in regions other than the pars, such as the pedicles. Type V, pathologic, refers to
spondylolisthesis resulting from generalized or local bone disease. The radiographs
demonstrate type II, isthmic spondylolisthesis. PREFERRED RESPONSE: 2

Question 64 Which of the flowing trajectories is preferred for placement of C1 lateral


mass screws?
1. 7.5 degrees lateral and 5 degrees cephalad
2. 7.5 degrees medial and 10 degrees cephalad
3. 10 degrees medial and 22 degrees cephalad
4. 10 degrees lateral and 22 degrees cephalad
5. 20 degrees medial and 22 degrees caudal
40

164
DISCUSSION: The C1 lateral mass can safely accommodate screw fixation. Trajectory of 10
degrees medial and 22 degrees cephalad was safely applied in a series of 50 patients.
Postoperative CT scans confirmed the safe trajectory. The benefit of lateral mass screws
is that they can be safely placed despite the existence of an anomalous vertebral artery
that could preclude the safe placement of transarticular screws.
PREFERRED RESPONSE: 3

Question 65 Which of the following tumors is considered highly vascular?

1. Thyroid
2. Breast
3. Colon
4. Prostate
5. Lung

DISCUSSION: Preoperative embolization is often used for highly vascular lesions such as
renal andthyroid metastases. PREFERRED RESPONSE: 1

Question 66 Which of the following is a significant risk factor for airway complications
after anterior cervical surgery?

1. Smoking history
2. Pulmonary disease
3. Absence of drainage from surgical drains
4. Surgical time of more than 5 hours
5. Myelopathy

DISCUSSION: In a study of 311 patients undergoing anterior cervical surgery only, a


surgical time of more than 5 hours and exposure of four or more vertebral bodies
involving C4 or higher were found to be risk factors for postoperative airway
complications. Surprisingly, preoperative pulmonary status, smoking history, absence of
drainage, and myelopathy were not associated with airway complications.

PRE RES: 4

41

165
Question 67 A 50-year-old man fell from a height of 10 feet and sustained an axial
loading injury to the cervical spine.He reports neck pain and right upper extremity
weakness and has weakness in the lower extremities.An MRI scan is shown Figure 67.
What imaging study should be obtained next to further evaluate this patient?

1. Cervical spine flexion and extension radiographs


2. AP radiograph of the cervical spine
3. CT of the cervical, thoracic, and lumbar spine
4. Electromyography and nerve conduction velocity studies of
the upper extremities
5. MRI of the lumbar spine

DISCUSSION: The MRI scan shows a C7 burst fracture. A CT scan


of the cervical spine will allow for optimal evaluation of this C7
burst fracture. Specifically, it will provide additional osseous
detail and will assist with the detection of additional fractures,
including those of the posterior elements. Additional CT imaging of the thoracic and
lumbar spine is required to rule out concommitant injuries (which may be present in 10%
to 15% of patients). Anteroposterior and lateral cervical spine radiographs would be a
good option for further evaluation but are not included in the available choices here.
Cervical spine flexion and extension radiographs should not be obtained in a patient who
is known to have a relatively unstable spine and a neurologic deficit. Electromyography
and nerve conduction velocity studies are best used to evaluate for cervical radiculopathy
secondary to degenerative abnormalities and are usually not indicated in the acute
trauma setting. PREFERRED RESPONSE: 3

Question 68 Which of the following actions increases radiation exposure to patients


and personnel when using fluoroscopy?
1. The use of lead glasses, thyroid shield, and a lead apron with a equivalent lead
thickness of 0.25 mm
2. Orienting the cathode ray tube beneath the patient with the image intensifier
receptor as close to the patient as possible
3. Limiting the beam on time to only what is clinically important
4. The use of continuous fluoroscopy to ensure proper placement of implants
5. Orienting the beam in the opposite direction of the working team and keeping the
team outside a 6-foot radius from the fluoroscopy machine
42

166
DISCUSSION: Continuous fluoroscopy and cineradiography exposes the patient and
personnel to markedly increased levels of direct and scatter radiation exposure.
Continuous fluoroscopy should be limited to only what is absolutely needed for safe
completion of the procedure. By orienting the cathode ray tube beneath the patient and
placing the image intensifier as close as clinically possible to the patient,scatter radiation
exposure to the personnel is minimized. PREFERRED RESPONSE: 4

Question 69 A 47-year-old man is seen in consultation in the ICU after being admitted
and treated emergently for a dissecting aortic aneurysm. Current examination reveals
generalized weakness of the lower extremities with a significant decrease in pain and
temperature sensation from approximately the waist down.Proprioception is
maintained. What is the most likely diagnosis at this time?
1. Anterior cord syndrome
2. Central cord syndrome
3. Brown-Sequard syndrome
4. Posterior cord syndrome
5. Spinal shock

DISCUSSION: Incomplete cord syndromes include anterior cord syndrome, Brown-


Sequard syndrome,central cord syndrome, and posterior cord syndrome. The anterior
cord syndrome involves a variable loss of motor function and pain and/or temperature
sensation, with preservation of proprioception as seen in this patient. The Brown-
Sequard syndrome involves an ipsilateral loss of proprioception and motor function, with
contralateral loss of pain and temperature sensation. The posterior cord syndrome is a
rare injury and is characterized by preservation of motor function, sense of pain and light
touch, with loss of proprioception and temperature sensation below the level of the
lesion. The central cord syndrome is characterized with greater motor weakness in the
upper extremities than in the lower extremities. The pattern of motor weakness shows
greater distal involvement in the affected extremity than proximal muscle weakness.
Spinal shock is the period of time, usually 24 hours, after a spinal injury characterized
by absent reflexes, flaccidity, and loss of sensation below the level of the injury.
PREFERRED RESPONSE: 1

43

167
Question 70 A 78-year-old woman reports a 1-week history of severe low back pain.
She denies any trauma or recent falls. She is neurologically intact, and is able to
ambulate, although she does require the use of a walker.Radiographs of the lumbar
spine show a T11 compression fracture with a 20% loss of anterior column height. What
is the most appropriate management at this time?
1. Bed rest until symptoms resolve
2. Analgesics and progressive rehabilitation
3. Anterior thoracic corpectomy and arthrodesis with instrumentation
4. Posterior thoracic decompression and fusion
5. Vertebral cement augmentation

DISCUSSION: The patient has sustained a thoracic compression fracture, which is very
common in elderly patients, and can occur with minimal to no trauma. There is
approximately a 20% loss of anterior vertebral body height. The patient is neurologically
stable, and is able to ambulate with an assistive device. Initial treatment should consist of
progressive mobilization with analgesics as needed. Fractures treated in this manner have
a high rate of success, and surgical treatment is often not necessary. In the absence of
neurologic impairment or impending structural instability, surgical decompression and
fusion is not indicated. Bed rest is contraindicated. Cement augmentation is a reasonable
treatment option when a patient fails nonsurgical management, although recent studies
have called into question its efficacy when compared with placebo. PREFERRED RES: 2

Question 71 What is the most common physical finding in myelopathic patients?


1. Babinski sign
2. Hoffman sign
3. Hyperreflexia
4. Clonus
5. Ataxic gait

DISCUSSION: The Hoffman sign is the most common finding, occurring in 80% of
myelopathic patients,and it is more common with increasing severity of the myelopathy.
The prevalence of hyperreflexia has been shown to be no different from the prevalence
in individuals without myelopathy. Sustained clonus and Babinski signs have been shown
to occur in only one third of the patients with myelopathy. PREFERRED RESPONSE: 2

44

168
Question 72 What is the most common presenting problem in patients with cauda
equina syndrome?
1. Urinary retention
2. Urinary incontinence
3. Saddle numbness
4. Lower extremity numbness and weakness
5. Back and leg pain
DISCUSSION: In one recent retrospective cohort study of 42 patients with cauda equina
syndrome, 83% had low back pain at presentation, 90% had radicular lower extremity
pain, 60% had urinary retention, and 55% had urinary incontinence. Objective findings at
presentation included 55% with leg weakness, 62% with sensory deficit, 62% with absent
ankle jerk reflexes, 76% with perianal sensory deficit, and 50% with decreased rectal
tone. PREFERRED RESPONSE: 5

Question 73 A 61-year-old woman has a history of a left thigh melanoma that was
widely resected approximately 12 years ago. The patient also has a history of
nephrolithiasis and has just undergone an abdominal CT scan for evaluation of her
kidneys. The patient was incidentally found to have a lytic lesion of the sacrum. A
radiograph is shown in Figure 73a. CT scans are shown in Figures 73b through 73d.
Open biopsy was performed and the biopsy specimen is shown in Figure 73e. What is
the most appropriate management?

45

169
1. En bloc excision of the sacrum and lesion
2. Both radiation therapy and chemotherapy
3. Embolization of the vascular supply to the lesion
4. Chemotherapy as the initial treatment, followed by surgical resection
5. Bisphosphonate therapy

DISCUSSION: The patient is asymptomatic from the sacral lesion, but has a history of a
malignant lesion,so the suspicion is high that the sacral lesion is a recurrence or a
metastatic lesion. The radiograph and CT scans show a lytic lesion within the osseous
margins of the sacrum, and the histologic section shows no malignant cells. The diagnosis
is Paget’s disease, which is typically treated medically. Bisphosphonate treatment is
typical, but is currently controversial as to whether it helps more than just controlling the
local symptoms. Radiographic features vary but can reveal cortical thickening, coarse
trabeculae, and sclerotic or enlarged vertebral bodies. The sacrum is typically involved.
Histologically there is “mosaic” appearing bone with numerous random intersecting lines,
overly active osteoclasts and/or osteoblasts, and fibrous tissue replacement of marrow.
The specimen shows disordered appearance of the bone and the multiple intersecting
lines. PREFERRED RESPONSE: 5

Question 74 A 42-year-old man reports a 3-day history of worsening lower back pain.
He denies any history of recent trauma or infections. He also reports difficulty urinating
and fecal incontinence in the last 24 hours.Examination reveals generalized lower
extremity weakness, diminished sensation in a saddle distribution,and loss of rectal
tone. What is the most appropriate management at this time?
1. Immediate MRI of the lumbar spine and possible acute surgical intervention
2. General reassurance, anti-inflammatory medications, and an early home exercise
program
3. Radiographs of the lumbar spine and pain medications with 2 days of bed rest
4. Caudal epidural steroid injection with follow-up in 1 week
5. Outpatient MRI of the lumbar spine with follow-up in 1 week

DISCUSSION: Cauda equina syndrome is a medical emergency that must be quickly


diagnosed and treated to avoid long-term complications. Cauda equina syndrome
typically presents with low back pain,unilateral or bilateral sciatica, saddle sensory
disturbances, bladder and bowel dysfunction, and variable lower extremity motor and
sensory loss. Although a number of pathologies can cause cauda equine syndrome, disk
46

170
herniation is the most common cause of acute onset cauda equina syndrome. Cauda
equina syndrome should be evaluated on an emergent basis and admission for work-up is
appropriate.Emergent MRI to evaluate the level of spinal compression and acute
decompression surgery is the most appropriate treatment in this situation. Nonsurgical
management consisting of medications, bed rest,and a home exercise program are not
appropriate. Whereas radiographs could be useful in a patient with traumatic onset of
symptoms, MRI is the best study for evaluation of the spinal canal. Office follow-up and
outpatient diagnositc testing are also inappropriate in this scenario. PREFERRED RES: 1

Question 75 Smoking has been associated with lower fusion rates in both cervical and
lumbar fusion. Which of the following statements best describes an explanation for
these findings?
1. Nicotine impairs osteoblast activity, thus interfering with bone remodeling.
2. The effects of smoking on bone healing are multifactorial and not yet fully
understood.
3. The vasoconstrictive and platelet-activating properties of nicotine inhibit fracture
healing.
4. Nicotine inhibits the function of fibroblasts, red blood cells, and macrophages.
5. Hydrogen cyanide inhibits oxidative metabolism at the cellular level.

DISCUSSION: Tobacco smoking is now the leading avoidable cause of morbidity and
mortality in the United States. The musculoskeletal effects of smoking have been
implicated in osteoporosis, low back pain, degenerative disk disease, poor wound
healing, and delayed fusion and fracture healing. A number of studies have demonstrated
the relationship between smoking and development of pseudarthrosis.Numerous studies
have been performed to offer an explanation of the mechanism mediating this
effect.Whereas all of the above have been postulated as explanations, more recent
studies have demonstrated that nicotine delivered via a transdermal patch significantly
enhanced posterior spinal fusion in rabbits. Thus it appears that the effects of smoking on
fracture healing are multifactorial and not yet fully understood. PREFERRED RESPONSE: 2
Question 76 Which of the following is the least common primary tumor of the spine?
1. Osteosarcoma
2. Hemangioma
3. Osteoid osteoma
4. Eosinophilic granuloma
5. Giant cell tumor
47

171
DISCUSSION: Osteoid osteoma, aneurysmal bone cysts, hemangiomas, eosinophilic
granuloma, and giant cell tumors are more commonly seen in the spine as opposed to
chondrosarcomas, Ewing’s sarcoma,and osteosarcomas which are uncommon in the
spine. Osteosarcoma is the second most common primary tumor of bone, but spinal
involvement is rare, accounting for 3% of all osteosarcomas. PREFERRED RESPONSE: 1

Question 77 In approaching the lateral lumbar spine through the psoas, the
lumbosacral plexus is in danger of being injured. The location of the plexus is best
described as which of the following?
1. More dorsal in the upper lumbar spine and more ventral in the lower lumbar spine
2. More ventral in the upper lumbar spine and more dorsal in the lower lumbar spine
3. Equally dorsal (junction of anterior two thirds and posterior one third) in the psoas
along the lumbar spine
4. Equally ventral (junction of anterior one third and posterior two thirds)in the psoas
along the lumbar spine
5. In the midportion of the psoas along the lumbar spine

DISCUSSION: Studies looking at the location of the lumbar plexus in the psoas muscle
dissected to show the position relative to the disk spaces for lateral lumbar approaches
demonstrate that the plexus is mostly posterior/dorsal at the L1-2 disk spaces. As one
travels down to the L4-5 disk spaces, the plexus is more ventral. It does not stay at the
same position within the psoas from the L1-2 to the L4-5 disk spaces. PREFERRED RES: 1

Question 78 Which of the following tumors rarely metastasizes to the spine?


1. Colon
2. Breast
3. Lung
4. Renal
5. Thyroid

DISCUSSION: Metastatic tumors are the most common tumors found in the spine. Up to
75% of all bony metastases occur in patients with carcinoma of the breast, lung, kidney,
prostate, thyroid, and multiple myeloma. Carcinoma of the colon rarely metastasizes to
the spine. PREFERRED RESPONSE: 1

48

172
Question 79 Which of the following would be associated with the spinal deformity
shown in Figures 79a and 79b?

1. Improved gait
2. Deformity progression
3. Delayed satiety
4. No further risk of fracture
5. Improved lung function
DISCUSSION: The images delineate progressive osteoporotic collapse. As outlined by
Kado and associates, Schlaich and associates, and Gold and associates, the progression of
spinal deformity and the functional consequences of vertebral compression fractures are
persistent even in those patients who are pain free. Vertebral compression fractures are
associated with deteriorating gait, early satiety, further future fracture risk, and
deteriorating lung function. PREFERRED RESPONSE: 2
Question 80 When compared with individuals with diffuse idiopathic scoliosis (DISH),
patients with ankylosing spondylitis have which of the following rates of neurologic
injury, treatment complications, and mortality?
1. Higher rates of neurologic injury, higher rates of treatment complications, and higher
mortality following spine trauma
2. Lower rates of neurologic injury, higher rates of treatment complications, and lower
mortality following spine trauma
3. Similar rates of neurologic injury, treatment complications, and mortality following
spine trauma
4. Higher rates of neurologic injury, higher rates of treatment complications, and similar
mortality following spine trauma
5. Higher rates of neurologic injury, higher rates of treatment complications, and lower
mortality following spine trauma

49

173
DISCUSSION: All patients with hyperostotic conditions of the spine are at a high risk of
neurologic injury following even minor trauma. However, several studies have shown that
patients with ankylosing spondylitis have higher rates of neurologic injury and
complications related to treatment compared with individuals with DISH. A recent meta-
analysis was unable to show a significantly increased risk of mortality for patients with
ankylosing spondylitis relative to those with DISH (3-month mortality estimated at 17% to
20%). PREFERRED RESPONSE: 4

Question 81 Figures 81a through 81c show the MRI scans of a 53-year-old man who has
experienced a long history of progressively worsening right-sided back pain with
radiation to the buttocks and right lower extremity.Examination reveals weakness and
hyperreflexia in the right lower extremity. He reports intermittent episodes of urinary
incontinence. What is the most appropriate surgical approach?

1. Costotransversectomy
2. Laminectomy
3. Transpedicular
4. Lateral extracavitary
5. Transthoracic
DISCUSSION: The presence of stenosis in the thoracic region with its related clinical
manifestations has only recently been appreciated. The pathogenesis of thoracic stenosis
is similar to that found in the cervical or lumbar spine. Two distinct clinical syndromes of
thoracic stenosis have been identified,the most common being associated with
degenerative changes of the spine. Clinical manifestations include development of

50

174
unilateral or bilateral symptoms of pseudoclaudication. Focal radicular pain or
paresthesias may also be present. The neurologic examination initially may be normal,
but as the degree of neural compression progresses, posterior column dysfunction and
long tract signs appear. If allowed to progress untreated, the patient may develop
significant difficulty with gait and bowel/bladder function.Thoracic spinal cord stenosis
secondary to congenital narrowing of the spinal canal is associated with a more abrupt
onset of symptoms. The typical clinical manifestations of myelopathy may commence
following minor or moderate trauma. Radicular symptoms are rare in congenital thoracic
stenosis. One indication for thoracic laminectomy is a patient in whom imaging has
demonstrated evidence of spinal canal stenosis secondary to hypertrophy of the
posterior elements. A laminectomy should not be the primary approach when stenosis
results from a significant ventral epidural osteophyte or herniated disk;these lesions are
more effectively and safely managed by a posterolateral (transpedicular, transfacetal, or
costotransversectomy) or an anterior approach. PREFERRED RESPONSE: 2

Question 82 A 22-year-old man has an acute spinal cord injury after a diving accident.
Preliminary radiographs reveal bilateral jumped facets at C6-7. Neurologic examination
shows an incomplete spinal cord injury consistent with an ASIA B impairment grade.
The patient is otherwise hemodynamically stable with no other injuries. Attempts at
closed high weight reduction with tong traction have so far been unsuccessful. What is
the most appropriate management at this time?

1. Continue a high weight closed reduction of the fracture-dislocation


2. Urgent surgical intervention for reduction and decompression
3. High-dose steroids for 48 hours before surgical stabilization
4. Halo fixation
5. Closed reduction under general anesthesia

DISCUSSION: Although there are no current standards for the timing of surgical
intervention for acute spinal cord injuries there is increasing data, including animal
studies, suggesting that early decompression and stabilization of an acute spinal cord
injury can be beneficial. Continuing attempts at closed reduction is not advised given the
failure of attempted high weight reduction. In light of the neurologic deficit,waiting 48
hours with or without steroid treatment is not recommended. Likewise, halo fixation
without reduction of the dislocation should not be considered for definitive treatment.
Closed reductions should not be performed under general anesthesia. PREFERRED RES: 2
51

175
Question 83 Treatment consisting of halo vest immobilization is most likely to fail with
which of the following cervical injuries?
1. C1 lateral mass fracture
2. C2 pars fracture
3. C4 burst fracture
4. C5 burst fracture
5. C6-C7 facet fracture-dislocation

DISCUSSION: Facet joint fracture or dislocation is associated with an increased risk of loss
of alignment with halo vest immobilization. The recently published study by van
Middendorp and associates confirms the findings of prior studies that facet fracture-
subluxations or dislocations are difficult to immobilize with a halo vest due to a limited
ability to maintain reduction and alignment. C2 pars fractures, burst fractures,and C1
lateral mass fractures can be managed with halo vest immobilization. PREFERRED RES: 5

Question 84 A 57-year-old woman with rheumatoid arthritis and a history of chronic


low back pain is seen in the emergency department reporting a 2-month history of
increasing low back pain. She denies any extremity pain. Examination reveals full lower
extremity strength and normal sensation. She has a history of intermittent, low-grade
fevers over the past few weeks. Vital signs reveal a temperature of 100.2 F, blood
pressure of 135/70 mm Hg, a heart rate of 95/min, and respirations of 18. A lumbar
spine MRI scan is shown in Figure 84. What is the most appropriate course of action at
this time?

1. Intravenous administration of broad-spectrum


antibiotics
2. Oral administration of broad-spectrum antibiotics
3. Urgent anterior partial corpectomies and débridement
and arthrodesis
4. Urgent posterior laminectomy and diskectomy for
débridement
5. Tissue biopsy followed by intravenous antibiotics and
close patient monitoring

52

176
DISCUSSION: The patient has infectious diskitis in the lumbar spine. This is more common
in those patients who are immunocompromised, such as IV drug abusers, alcoholics, and
patients on immunosuppressive agents such as patients with rheumatoid arthritis. The
time from onset of symptoms to accurate diagnosis can vary from weeks to months. The
patient is neurologically intact, and has no clinical or laboratory evidence of
overwhelming sepsis. Thus, the most appropriate initial course of treatment should
consist of obtaining a tissue biopsy of the area for accurate pathogen
identification,followed by intravenous antibiotic administration. Urgent surgery via any
approach is not indicated without evidence of neurologic compromise or sepsis. The
administration of antibiotics, whether by an oral or intravenous route prior to biopsy, will
hinder accurate identification of the offending organism,which can adversely affect
treatment with pathogen-specific antibiotics. Surgery would be performed if the patient
deteriorated neurologically, or failed medical management of the infection.P RE: 5

Question 85A 69-year-old man has nonpainful weakness in the upper and lower
extremities. He also notes progressive instability in his gait and increasing difficulty
ambulating, as well as manipulating small objects with his hands. MRI scans of his
cervical spine are shown in Figures 85a and 85b. When would be the most appropriate
time to proceed with surgical treatment?

1. When the patient is medically stable for surgery


2. When the MRI scans show multisegmental high-intensity intramedullary signal
changes on T2-weighted sequences
3. When he reaches a Nurick grade of IV for his preoperative neurologic function
4. When he reports neck and/or extremity pain that becomes intolerable or not
controlled by medication
5. When he develops bowel or bladder incontinence

53

177
DISCUSSION: The natural history of cervical myelopathy is one of slow deterioration over
time, typically in a stepwise fashion with a variable period of stable neurologic function.
More recent studies suggest that surgery should be performed as soon as possible when
cervical spondylotic myelopathy has been diagnosed. Both anterior and posterior are
effective and there is no statistical difference between their outcomes. Surgical outcome
is related to the patient’s age, disease course, the presence of osseous spinal stenosis,
preoperative comorbidities, the preoperative spinal cord functional score, and the
presence of high-signal abnormalities on T2-weighted images. To improve the operative
result, all the influencing factors should be considered. Patients with focal high-intensity
intramedullary signal changes on T2-weighted images have better clinical outcomes
following surgery than do patients with demonstrable multisegmental high-intensity
intramedullary signal changes on T2-weighted sequences. The transverse area and shape
of the spinal cord at the involved segment may also be predictive of surgical outcome.
With progressive compression, the cross section of the spinal cord changes from a
boomerang shape to a teardrop shape to a triangular shape. In patients with a Nurick
grade of I, there are signs of cord involvement, but gait remains normal. With a Nurick
grade of II, there are mild gait abnormalities,not affecting the patient’s employment
status. With a Nurick grade of III, gait abnormalities prevent employment, but the patient
remains able to ambulate without assistance. In Nurick grade IV, the patient is only able
to ambulate with assistance. In Nurick grade V, the patient is chair-bound or bedridden.
Clearly, it is desirable to operate when the patient is functioning with a Nurick grade of I
or II. Whereas many patients presenting with cervical spondylotic myelopathy also report
axial neck pain and radicular symptoms in the upper extremities, this is not always the
case. Surgical intervention will generally be effective in eliminating this pain; however,
the pain is not the determining factor for performing surgery.Surgery is performed to
preserve and restore function. Changes in bowel and bladder function can occur
in extremely severe cases of myelopathy, but this is quite rare. PREFERRED RESPONSE: 1

Question 86 Which of the following statements best describes the instantaneous axis of
rotation (IAR) for thefunctional spinal unit?
1. The IAR is calculated by movement around six different axes.
2. The IAR consists of potential movements or 12 degrees of freedom.
3. The IAR is fixed and consistent and is the axis about which each vertebral segment
rotates.
4. The IAR can be derived from AP and lateral radiographs.
5. Degenerative disease, fractures, and ligamentous injuries can affect the IAR.
54

178
DISCUSSION: The instantaneous axis of rotation is the axis about which each vertebral
segment rotates,but is theoretical depending on how it is defined, and varies depending
on multiple factors. It is not a fixed point but can move depending on the position of the
spine, and it is affected by degenerative conditions,fractures, injuries, and other anatomic
changes of the spine. There are three axes of movement with 6 degrees of freedom
(rotation and translation movements about each axis). PREFERRED RESPONSE: 5

Question 87 Metal-on-metal lumbar disk arthroplasty devices may generate cobalt and
chromium ions into the serum of patients after implantation into the lumbar spine.
Which of the following statements best represents the levels of the serum ion levels in
these patients?
1. The serum ion levels are not measureable in these patients.
2. The serum ion levels are measureable, but are of negligible value in these patients.
3. The serum ion levels measured equal the values measured in the local tissues in total
hip arthroplasty metal-on-metal prostheses.
4. The serum ion levels measured are much lower in terms of their level to the values
measured in total hip arthroplasty metal-on-metal prostheses.
5. The serum ion levels measured are similar in terms of their level to the values
measured in total hip arthroplasty metal-on-metal prostheses.

DISCUSSION: According to two studies looking at patients with a cobalt-chrome metal-on-


metal lumbar disk arthroplasty, serum ion levels in these patients were similar to values
measured in patients with total hip arthroplasty metal-on-metal prostheses. PRE RES: 5

Question 88 A 78-year-old woman has a history of chronic low back pain. She denies
any extremity problems. Her pain is worse in the morning, and gets better, although it
does not go away, as the day goes on. An MRI scan of the lumbar spine is shown in
Figure 88. She denies any acute worsening of her symptoms,although in general, her
symptoms are slowly worsening. She takes nonsteroidal anti-inflammatory drugs as
needed for her pain, but otherwise takes no other medications. What
is the next most appropriate step in management?
1. DEXA scan
2. Brace treatment with a Jewett hyperextension brace
3. Anterior lumbar corpectomy and arthrodesis with instrumentation
4. Posterior lumbar decompression and fusion
5. Vertebral cement augmentation
55

179
DISCUSSION: The patient has MRI findings throughout her lumbar spine consistent with
old compression fractures. Given the imaging findings and advanced age, she is at high
risk for osteoporosis and subsequent fragility fractures. Management should consist of a
DEXA scan to evaluate her degree of osteoporosis and begin medical treatment as
appropriate. Because acute fracture is unlikely, and she has no neurologic compromise,
neither bracing nor surgical treatment is indicated.

PREFERRED RESPONSE: 1

Question 89 What is the incidence of the radiographic finding shown in Figure 89 in the
adult population in North America?

1. Less than 2%
2. 4% to 6%
3. 8% to 10%
4. 12% to 14%
5. Greater than 15%

DISCUSSION: The pars interarticularis is a thin


bicortical region of the posterior arch of the
lumbar vertebra, where the lamina and inferior
articular process intersect with the pedicle and
superior articular process. A fibrocartilaginous
cleft in this region is referred to as a spondylolysis
or a pars defect. This finding is present in 4% to
6% of the North American adult population.
Buetler and associates conducted a prospective study of 500 children and followed them
through young adulthood. They found the incidence of spondylolysis to be 4.4% in
children aged 6 years. An additional eight subjects developed
spondylolysis between the ages of 12 and 25 years for an adult incidence of 6%. Ninety
percent of the pars defects occurred at the L5-S1 level, with two thirds of the defects
occurring in males.

PREFERRED RESPONSE: 2

56

180
Question 90 A 78-year-old man is seen in the emergency room 3 hours after a fall from
a standing position. The patient sustained a mild scalp laceration and the injury shown
in Figure 90. He reports severe neck pain and is unable to move his hands and legs.
Examination reveals absent motor function in the wrist flexors,triceps, and fingers. He
cannot move his lower extremities during motor testing. The patient has some
sensation in the lower extremities. Bulbocavernosus reflex is absent. Based on
examination findings and the imaging findings, what is the most definitive treatment
option?

1. Closed reduction and immobilization in a halo-thoracic vest


2. Halo application and cervical traction for 6 weeks, followed by 8 weeks of
immobilization in a halo-thoracic vest
3. Open reduction, decompression, and fusion with anterior-posterior stabilization
4. Open reduction, anterior decompression, and fusion
5. Uninstrumented posterior fusion spanning the injured segment

DISCUSSION: The patient has a hyperostotic condition of the cervical spine, most likely
ankylosing spondylitis. Because of a rigid and osteoporotic spine, relatively minor falls can
result in unstable spinal injuries with significant instability and a high risk for neurologic
sequelae. The patient has an unstable injury at C6 with an incomplete spinal cord injury,
necessitating urgent decompression and stabilization.Studies have shown that, in

57

181
patients with ankylosing spondylitis, stand-alone anterior stabilization results in a high
failure rate. Halo-thoracic vests carry a high risk of septic and pulmonary issues, especially
in the elderly. Uninstrumented fusion will provide insufficient stability in such patients.
PREFERRED RESPONSE: 3

Question 91 A 35-year-old man who has had a 6-month history of low back pain and
tenderness now reports worsening pain and stiffness in the hips and entire back. An AP
radiograph of the pelvis demonstrates fusion of the sacroiliac joints bilaterally. What is
the next most appropriate step in management?
1. Anesthetic injections in both sacroiliac joints
2. Sacroiliac fusion with plate fixation
3. Anti-inflammatory medications, physical therapy, and HLA-B27 testing
4. Patient reassurance and follow-up as needed
5. Immediate bilateral sacroiliac joint aspiration and culture

DISCUSSION: The patient has a classic presentation of early ankylosing spondylitis.


Sacroiliac joint fusion is the earliest radiographic finding and is typically followed by
cephalad spinal progression. Early treatment of ankylosing spondylitis consists of
nonsteroidal anti-inflammatory drugs and physical therapy to preserve spinal motion.
HLA-B27 testing is positive in most (about 95%) patients; however, it is not
pathognomonic because it can be positive with other conditions. Considering the
progressive nature of this disease, further work-up in a patient with potential ankylosing
spondylitis is not warranted. Sacroiliac joint anesthetic injections and sacroiliac fusion are
not recommended treatments for early ankylosing spondylitis. Aspiration of the sacroiliac
joints can be done if sacroiliac joint infection is suspected;however, in the absence of
fever or other constitutional symptoms, infection is unlikely. PREFERRED RESPONSE: 3

Question 92 Which of the following is the most common site for compression and burst
fractures?

1. L2 level
2. L3 level
3. L4 level
4. L5 level
5. S1 level

58

182
DISCUSSION: Thoracolumbar compression and burst fractures occur most frequently at
the thoracolumbar junction (between T12 and L2) and represent more than 50% of all
thoracic and lumbar fractures. The thoracolumbar junction represents a transition from
the stiff thoracic spine to the more flexible lumbar spine. PREFERRED RESPONSE: 1

Question 93Figure 93 shows the axial T2-weighted MRI scan of the lumbar spine of a
70-year-old man. The arrow points to which of the following structures?
1. Lamina
2. Facet joint
3. Lumbar synovial cyst
4. Ligamentum flavum
5. Epidural space

DISCUSSION: The ligamenta flava (singular,


ligamentum flavum, Latin for yellow ligament)
are ligaments that connect the laminae of
adjacent vertebra, all the way from the axis to
the first segment of the sacrum. In T2-
weighted sequencing, ligamentous structures
possess a low signal intensity. The
ligamentum in this patient is markedly
thickened, resulting in severe spinal stenosis. The epidural space lies ventral and medial
to the ligamentum flavum and should possess a high signal intensity secondary to the
presence of cerebrospinal fluid. However, in the case of high-grade stenosis, there may
be little if any cerebrospinal fluid present, making the epidural space and central canal
difficult to identify. A lumbar synovial cyst should also have high signal intensity because
of the presence of synovial fluid. PREFERRED RESPONSE: 4

Question 94The findings in Brown-Séquard syndrome include loss of which of the


following?
1. Greater loss of upper extremity motor function than lower extremity function
2. Ipsilateral motor function and ipsilateral pain and temperature sensation
3. Ipsilateral motor function and contralateral pain and temperature sensation
4. Contralateral motor function and ipsilateral pain and temperature sensation
5. Lower of extremity proprioception and balance

59

183
DISCUSSION: Brown-Séquard syndrome is most commonly seen after penetrating injuries
to the spinal cord and results in ipsilateral loss of motor function and contralateral loss of
pain and temperature sensation. Patients with central cord syndrome have greater
weakness in the upper extremities than in the lower extremities. Loss of proprioception is
typically seen in patients with posterior cord syndrome. PREFERRED RESPONSE: 3

Question 95 A 35-year-old rock climber sustains an L1 burst fracture from a 30-foot fall
while climbing. He sustained no other fractures or serious injuries. He is neurologically
intact and has minimal posterior tenderness without increased spinous process
separation on examination. Radiographs reveal kyphosis of 20 degrees between T12
and L2 with 30% vertebral height loss. A CT scan shows 55% canal compromise. What is
the most appropriate management?
1. Bed rest for 6 weeks, followed by a thoracolumbosacral orthosis (TLSO) until the
fracture is healed
2. TLSO and or body cast for 3 to 6 months with mobilization when comfortable
3. Posterior spinal fusion and instrumentation
4. Anterior decompression with spinal fusion and instrumentation
5. Anterior decompression and anterior-posterior spinal fusion and instrumentation

DISCUSSION: Wood and associates have shown that the use of a TLSO or a body jacket
was equally effective as surgery for the treatment of thoracolumbar burst fractures
without neurologic deficit. The only difference in any of the measured parameters
(including pain, functional outcome, residual canal compromise, and kyphosis) was a
decreased complication rate in the nonsurgical group compared with the surgical group.
The maximum time to mobilization in the nonsurgical group was 5 days. PRE RES: 2

Question 96 A 75-year-old woman who sustained a fall now reports neck pain and
upper extremity weakness.Examination reveals 4 of 5 strength in the upper extremities
and 5 of 5 strength in the lower extremities.Radiographs show multilevel degenerative
disk disease. An MRI scan is shown in Figure 96. Her clinical presentation is most
compatible with which of the following?
1. Brachial plexus injury
2. Anterior cord syndrome
3. Posterior cord syndrome
4. Central cord syndrome
5. Brown-Séquard syndrome
60

184
DISCUSSION: The MRI scan shows advanced multilevel degenerative changes and
moderate to severe stenosis at C3-C4 and C4-C5 with associated cord signal change. The
patient has greater weakness in the upper extremities than in the lower extremities. This
pattern is most compatible with central cord syndrome. Patients with brachial plexus
injury will have unilateral weakness. Patients with anterior cord syndrome will have
greater weakness in the legs than in the arms, and those with Brown-Séquard syndrome
will have ipsilateral motor deficits and contralateral pain and temperature deficits.
PREFERRED RESPONSE: 4

Question 97 A patient who underwent an L5-S1 hemilaminotomy and partial


diskectomy for radiculopathy 3 weeks ago now reports increasing leg and back pain
with radicular signs. An axial T2-weighted MRI scan is shown in Figure 97a, an axial T1-
weighted MRI scan is shown in Figure 97b, and a contrast enhanced T1-weighted MRI
scan is shown in Figure 97c. What is the most appropriate management for the
patient’s symptoms?

1. Irrigation and débridement of deep wound infection


2. CT-guided needle biopsy and IV antibiotics
3. Revision laminotomy and diskectomy
4. L4-L5 anterior débridement and fusion
5. Open repair of the L4-L5 pseudomeningocele

DISCUSSION: The MRI scans show a recurrent disk herniation. There is no increase fluid
signal or enhancement to suggest infection or any other pathologic process. There is no
infection; therefore, IV antibiotics and débridement are not indicated. Similarly, a
pseudomeningocele is not present. In addition,with progressive weakness, physical
therapy is not appropriate. A revision diskectomy is useful for recurrent radiculopathy.
PREFERRED RESPONSE: 3
61

185
Question 98 Thoracic disk herniations most typically occur at what level of the thoracic
spine?
1. Upper third
2. Junction of upper third and middle third
3. Middle third
4. Junction of middle third and lower third
5. Lower third

DISCUSSION: Most thoracic disk herniations occur in the lower (caudal) third of the
thoracic spine. This predilection may be related to the unique anatomic and
biomechanical environment of that region. The 11th and 12th ribs do not join the rib cage
anteriorly and do not form a true articulation with the transverse processes posteriorly.
Furthermore, flexion and torsional forces tend to concentrate between T10 and L1.
PREFERRED RESPONSE: 5

Question 99An acute posterolateral disk herniation at the L4-5 level will most likely
affect what nerve root?
1. L3
2. L4
3. L5
4. S1
5. S2

DISCUSSION: Posterolateral disk herniations will generally compress the transversing


nerve root.Therefore, a posterolateral disk herniation at the L4-5 level will typically
compromise the L5 nerve root.Far lateral (extraforaminal) disk herniations generally
compromise the exiting nerve root. Therefore, an extraforaminal herniation at the L4-5
level will typically compromise the exiting L4 nerve root. PREFERRED RESPONSE: 3

Question 100Which of the following groups of tumors is most likely to metastasize to


the spine?
1. Uterine, thyroid, renal, lung
2. Breast, prostate, lung, kidney, thyroid
3. Breast, prostate, lung, kidney, skin
4. Breast, prostate, other osseous, skin, thyroid
5. Breast, prostate, hemangioma, lung, kidney
62

186
DISCUSSION: The primary tumors most likely to metastasize to the vertebral column are
breast (16%to 37% of breast cancer patients develop spine metastases), prostate (9% to
15%), lung (12% to 15%),kidney (3% to 6%), and thyroid (4%). Symptomatic lesions
typically are found in the thoracic (68% to 70%), lumbosacral (16% to 22%), and cervical
(8% to 15%) spine. Hemangiomas are benign tumors of the spine and are typically seen in
the vertebral body as lesions that are bright on T1- and T2-weighted images; they do not
typically metastasize to the spine. PREFERRED RESPONSE: 2

RESPONSES FOR QUESTIONS 101 THROUGH 103


1. C4
2. C5
3. C6
4. C7
5. C8
For each patient, please select the correct nerve root involvement.

Question 101 A 35-year-old woman with severe shoulder pain has 3+/5 weakness of the
left deltoid and decreased sensation along the lateral arm. There is an asymmetric
decrease of the bicep reflex. For each patient, please select the correct nerve root
involvement.

PREFERRED RESPONSE: 2

Question 102A 75-year-old man has a chronic history of decreased right-sided grip
strength. Upper extremity reflexes are decreased bilaterally but symmetric. Significant
wasting of the interosseous muscles is noted in the right hand.

PREFERRED RESPONSE: 5

Question 103A 40-year-old woman has an acute history of lateral neck pain and
numbness. There is mild scapular winging on the left side. All reflexes are normal and
symmetric in the upper extremity

PREFERRED RESPONSE: 1

63

187
DISCUSSION FOR QUESTIONS 101 THROUGH 103:
Compression of the C4 nerve root can cause sensory deficits in the lower neck and upper
shoulder region.The C4 contributes innervation to the diaphragm. C5 nerve root lesions
cause pain in the upper shoulder and arm region but not below the elbow. Sensory loss is
typically in an oval patch over the lateral upper arm. The most commonly affected
muscles are the deltoid and infraspinatus and supraspinatus muscles.A C6 nerve root
lesion causes pain in the lateral forearm with the classic numbness in the thumb and
index finger. Motor deficits include the brachialis, biceps, and brachioradialis (the wrist
extensors). A C7 nerve root lesion generates pain in the posterior arm and scapula region
with radiation to the dorsal forearm.The sensory examination often reveals involvement
of the second and third fingers. A C8 radiculopathy creates pain in the medial forearm
and into the axilla occasionally. Sensory loss is usually in the medial forearm and fifth
digit. C8 motor weakness occurs with the long flexors of the thumb and index
finger.There is often intrinsic finger weakness as well if there is also T1 nerve root
involvement. A C6 nerve root lesion affects the bicep reflex, a C7 nerve root lesion affects
the triceps, and a C8 lesion can affect the finger flexor reflex.
CLINICAL SITUATION FOR QUESTIONS 104 THROUGH 106

Figures 104a and 104b are the radiographs of a 40-year-old woman who has had pain
and swelling for the past 3 months over the distal thigh area near her right knee. A
biopsy specimen is shown in Figure 104c.
Question 104 What is the most likely diagnosis?
1. Fibrous dysplasia
2. Aneurysmal bone cyst
3. Osteosarcoma
4. Giant cell tumor
5. Osteomyelitis

64

188
PREFERRED RESPONSE: 4
Question 105Workup of this patient should specifically include which of the following
studies?
1. Westergren sedimentation rate
2. Platelet function tests
3. CT scan of the chest
4. Serum electrophoresis
5. Prostate-specific antigen

PREFERRED RESPONSE: 3
Question 106 Which of the following represents the most appropriate treatment at this
time?
1. Amputation of the leg
2. Distal femoral replacement
3. Chemotherapy and radiation therapy with non-weight-bearing until the lesion heals
4. Curettage, with local phenol and cementation
5. Hormonal therapy

PREFERRED RESPONSE: 4
DISCUSSION FOR QUESTIONS 104 THROUGH 106:
This is a classic presentation of a giant cell tumor. It is a benign aggressive bone tumor
consisting of distinct undifferentiated mononuclear cells. Most occur in patients 30 years
to 50 years of age (90% are older than age 20 years) and affect woman more than men.
Pain and swelling for 2 to 3 months is classic.The most common location is the distal
femur. The biopsy specimen shows multinucleated giant cells amid a background of
mononuclear cells. This benign tumor may rarely metastasize to the lung (less than 2% of
the time). A CT scan of the chest is required to rule out any further lesions. Curettage,
with local adjuvants, such as phenol, cryotherapy, or argon beam, and cementation
represents the best treatment option at this time.
CLINICAL SITUATION FOR QUESTIONS 107 THROUGH 109
An otherwise healthy 65-year-old man reports progressive and unrelenting back pain for
the last 6 weeks that is nonmechanical in nature. He denies leg pain, weakness, bowel or
bladder symptoms, and any significant past medical history. His pain is increased at night.
Examination reveals pain with percussion throughout his thoracic spine. He is
neurologically intact. AP and lateral radiographs of the thoracic and lumbar areas show
diffuse osteopenia with no fractures.
65

189
Question 107 What is the most appropriate test to order at this time?
1. PET scan
2. CT scan of the thoracic spine
3. Upright scoliosis series
4. MRI with gadolinium of the thoracic spine
5. Total body bone scan

PREFERRED RESPONSE: 4
Question 108 If the scan shows metastatic, noncontiguous lesions throughout the
thoracic spine without epidural spinal cord compression with no known primary lesion,
what is the next step in establishing a diagnosis?
1. Open biopsy of the spine lesion
2. Image-guided biopsy
3. Bone marrow aspirate of the ilium
4. CT scan of the chest, abdomen, and pelvis
5. Urine electrolytes

PREFERRED RESPONSE: 4
Question 109 Diagnostic work-up concluded that the patient has metastatic lymphoma.
He has no neurologic deficit.Based on these findings, what is the most appropriate
management for this spinal condition?
1. Cement augmentation
2. En bloc resection
3. Chemotherapy
4. Radiation therapy
5. Intralesional excision

PREFERRED RESPONSE: 4

DISCUSSION FOR QUESTIONS 107 THROUGH 109:


Patients with no prior diagnosis of cancer may present with vertebral lesions. After
obtaining initial radiographs, the next most appropriate step is to obtain an MRI with
gadolinium. To determine the primary site of the disease, a CT of the chest, abdomen,
and pelvis is the next step in establishing a diagnosis. Radiation therapy is the first line of
treatment in spinal metastatic disease from lymphoma because it is a highly
radiosensitive tumor.
66

190

You might also like